Sunteți pe pagina 1din 259

Physics

BOARD ANSWER PAPER : MARCH 2016


PHYSICS
SECTION – I
Q.1. Attempt any SIX :
i. a. Consider a particle performing circular motion in anticlockwise sense with centre O
and radius r as shown in the figure.

b. Let,  = angular velocity of the particle

v = linear velocity of the particle

r = radius vector of the particle


O  v [½]
r

c. Linear displacement in vector form is given by,


  
s =   r [½]
Dividing both side by t,
 
s  
=  r ….(1)
t t
d. Taking limiting value in equation (1)
 
s  
lim = lim  r
t  0 t t  0 t
 
ds d 
 =  r [½]
dt dt

ds 
But, = v = linear velocity,
dt

d 
=  = angular velocity
dt
  
 v =   r [½]
ii. Expression for critical velocity:
m
vC
h
r

M
Earth R

1
Board Answer Paper : March 2016
a. Let,
M = mass of the earth
R = radius of the earth
h = height of the satellite from the earth’s surface
m = mass of the satellite
vc = critical velocity of the satellite in the given orbit
r = (R + h) = radius of the circular orbit [1]
b. For the circular motion of the satellite, the necessary centripetal force is given as,
mvc2
FCP = ….(1)
r
c. The gravitational force of attraction between the earth and the satellite is given by,
GMm
FG = .…(2)
r2
d. Gravitational force provides the centripetal force necessary for the circular motion of
the satellite.
 FCP = FG
mv c2 GMm
 = ….[From equations (1) and (2)] [½]
r r2
GM
 vc2 =
r
GM
 vc = .…(3)
r
e. But, r = R + h
GM
 vc = ….(4) [½]
(R  h)
Also, GM = gh (R + h)2
where, gh is acceleration due to gravity at height ‘h’ above earth’s surface.
 vc = gh  R  h  ….(5)
Equations (4) and (5) represent critical velocities of satellite orbiting at a certain height
above the earth’s surface.
iii. a. Let,
M = mass of the body
v = linear velocity of the body
 = angular velocity of the body
I = moment of inertia of the body
K = radius of gyration
b. Translational kinetic energy of body,
1
(K.E.)translational = Mv2 [½]
2
Similarly,
Rotational kinetic energy of rolling body is
1
(KE)rotational = I2 [½]
2
c. Total K.E of rolling body = (K.E) translational + (K.E) rotational
1 1
(K.E)rolling = Mv2 + I2
2 2
But I = MK2 [½]
and v = r [½]
2
Physics

1 1  v2 
 (K.E)rolling = Mv2 + MK2  2 
2 2 r 
1  K2 
 (K.E)rolling = Mv2 1 + 2 
2  r 

iv. a. Emissive power: The emissive power of a body at a given temperature is defined as the
quantity of radiant energy emitted by the body per unit time per unit surface area of the
body at that temperature. [1]
b. Coefficient of emission of a body: The ratio of the emissive power of a body at a given
temperature to the emissive power of a perfectly black body at the same temperature is
called coefficient of emission (emissivity) of the body. [1]
v. Solution:
Given: r = 5 cm = 0.05 m
90
n = 90 r.p.m. = r.p.s.
60
g = 9.8 m/s2
To find: Coefficient of static friction (s)
r2
Formula: s = [½]
g
Calculation:
2    90
Since,  = 2n = = 3 rad/s
60
0.05  (3) 2
 s = [½]
9.8
0.45  (3.14) 2
=
9.8
= {antilog [(1.6532) + 2(0.4969) – (0.9912)]}
= {antilog(1.6558)}
= 0.4527
The coefficient of static friction between the coin and the turntable is 0.4527. [1]
vi. Solution:
Given: Fundamental frequency for closed pipe = Third overtone of open pipe.
l 
To find: Ratio of lengths of air columns in both the pipes  c 
 lo 
Formulae: a. Fundamental frequency of pipe closed at one end:
v
nc = [½]
4lc
b. Fundamental frequency of pipe open at both ends:
v
no = [½]
2lo
c. Third overtone of open pipe:
n3 = 4n0
Calculation:
Fundamental frequency of closed pipe (nc) is same as third overtone of open pipe.
 nc = n3
 nc = 4n0 [½]
3
Board Answer Paper : March 2016

v  v 
 = 4 
4lc  2lo 
1 2
 =
4lc l0
lc 1
 =
lo 8
The ratio of lengths of air columns in closed pipe and open pipe is 1:8. [½]
vii. Solution:
Given: 2A = 20 cm  A = 10 cm,
T = 6.28 s, x = 6 cm
To find: Velocity (v)
Formula: v =  A2  x 2 [½]
2 2  3.14
Calculation: Since,  = = = 1 rad/s
T 6.28
 v = (1) (10) 2  (6) 2 [½]
= 100  36 = 8 cm/s
The velocity of the particle at x = 6 cm is 8 cm/s. [1]
viii. Solution:
Given: E = 5T
To find: Diameter of the drop (d) in C.G.S. system
Formula: E = TA
Calculation: A = 4r2
 E = 4r2T
 5T = 4r2T [½]
 4r2 = 5
5 5
 r= =
4 2
 d = 2r = 5 [½]
= 2.236 cm
Diameter of the drop is 2.236 cm. [1]
Q.2. Select and write the most appropriate answer from the given alternatives for each sub-
question:
i. (D)
s
=
r
D
s = vt and r =
2
2vt
 = [1]
D
ii. (B)
1
K
x
and x  W
1
 W [1]
K

4
Physics
iii. (A)
1
Stress  [1]
r2
iv. (D) [1]
v. (B) [1]
vi. (C) [1]
vii. (C)
K.E1 = K.E2
1 1
 I1 12  mv 2
2 2
5  36 20v 2
 
2 2
 90 = 10 v2
 v = 3 m/s [1]
Q.3. A. Linear S.H.M is defined as the linear periodic motion of a body, in which the restoring force
(or acceleration) is always directed towards the mean position and its magnitude is directly
proportional to the displacement from the mean position. [1]
i. Consider a particle ‘P’ moving along the circumference of a circle of radius ‘A’ with
constant angular speed ‘’ in anticlockwise direction.
ii. At any instant t = 0, particle P has its projection at point M as shown in the
figure (a). Particle P is called as reference particle and the circle on which it moves is
called as reference circle.
iii. As reference particle P revolves, its projection moves back and forth about centre O
along the diameter BC. [½]
Y Y

P P 
A A
 t + 
X
X
B O M C X B Mx O A X

t=0 t>0
Y Y
Figure (a) Figure (b) [½]
iv. The x-component of the displacement, velocity and acceleration of P is always same as
the displacement, velocity and acceleration of M.
v. Suppose that the particle P starts from an initial position with initial phase ‘’. In time
‘t’, the angle between OP and X-axis is (t + ) as shown in the figure (b).
x
vi. In figure (b), cos (t + ) =
A
where x = displacement from mean position.
 x = A cos (t + ) ….(1) [½]
Equation (1) represents displacement of projection of P at time ‘t’.
vii. The velocity of particle is the time rate of change of displacement.
dx d
v= = [A cos (t + )]
dt dt
 v =  Asin (t + ) ….(2) [½]
Equation (2) represents velocity of projection of P at time ‘t’.
5
Board Answer Paper : March 2016
viii. The acceleration of particle is the time rate of change of velocity.
dv d
a= = [A sin(t + )]
dt dt
 a = A2 cos (t + )
 a = 2x [ x = A cos (t + )]
 a = 2x ….(3) [½]
Equation (3) represents acceleration of projection of P at time ‘t’.
From equation (3),
ax [ 2 = constant quantity]
ix. As acceleration of projection of P is directly proportional to its displacement and its
direction is opposite to that of displacement, thus projection of particle P performs
simple harmonic motion. But M is projection of particle P performing U.C.M. Hence,
S.H.M is the projection of U.C.M along a diameter of circle. [½]
B. Solution:
Given: 0 = 25 C, 1 = 50 C, 2 = 45 C,
 d 
 dt  = 4 C/min
 1
 d 
To find: Rate of cooling at 2  
 dt  2
d
Formula: = K(  0) [½]
dt
Calculation: Using formula, for 1 = 50 C
4 = K(50 – 25) [½]
4
 K= min1 [½]
25
 For 2 = 45 C
 d  4
  =  (45 – 25) [½]
 dt  2 25
4
=  (20)
25
16
=
5
= 3.2 C/min
The rate of cooling at 45 C is 3.2 C/min. [1]
OR
A. Formation of stationary waves by analytical method:
i. Consider two identical progressive waves of equal amplitude and frequency travelling
along X axis in opposite direction. They are given by,
2
y1 = A sin (vt  x) along positive X-axis .…(1) [½]

2
y2 = A sin (vt + x) along negative X-axis ….(2) [½]

ii. The resultant displacement ‘y’ is given by the principle of superposition of waves,
y = y 1 + y2 ….(3)
2 2
y = A sin (vt  x) + A sin (vt + x)
 
6
Physics
iii. By using,
C D C D
sin C + sin D = 2sin   cos  ,
 2   2 
we get,
 2  vt  x  vt  x    2  vt  x  vt  x  
y = 2A sin    cos   
  2    2 
 2vt   2 
= 2Asin   cos  ( x) 
     
 2x   v
 y = 2Asin 2πnt cos    n   [cos ( ) = cosθ]
     
 2x 
 y = 2Acos   sin 2πnt
  
 2x 
iv. Let R = 2Acos  
  
 y = Rsin (2πnt) ….(4)
But,  = 2n
 y = R sin t ….(5) [½]
Equation (5) represents the equation of S.H.M. Hence, the resultant wave is a S.H.M.
of amplitude R which varies with x.
v. The absence of x in equation (5) shows that the resultant wave is neither travelling
forward nor backward. Therefore it is called as stationary wave.
vi. Amplitude at node is minimum, i.e., 0.
 Rmin = 0
 2 x 
Since R = 2A cos  ,
  
 2 x 
 cos   =0 [½]
  
2x  3 5
 = , , , …….
 2 2 2
 3 5
 x= , , ,……. [½]
4 4 4
vii. At antinodes: R =  2A
 2x 
 cos   = 1 [½]
  
 2x 
   = 0, , 2, 3,…….n
  
 3
 x = 0, ,, ……… [½]
2 2
 Distance between first node and adjacent
 
antinode = xn  xan = 0= [½]
4 4
λ
 The distance between a node and an adjacent antinode is .
4
B. Solution:
Given: N = 48, X = 4
nF = 1.5 nL, i.e. n1 = 1.5 n48 [½]
7
Board Answer Paper : March 2016
To find: a. Frequency of first tuning fork (n1)
b. Frequency of 42nd tuning fork (n42)
Formula: nL = nF  (N  1)X
Calculation: Using formula, for N = 42
n42 = n1  [(42 – 1)  4]
= n1  164 [½]
Also for N = 48,
n48 = n1  [(48 – 1)  4]
= n1  188 [½]
n
n48 = 1 ….(Given)
1.5
n1
 = n1 – 188
1.5
188  1.5
 n1 = = 564 Hz
0.5
Frequency of first fork is 564 Hz [1]
Substituting the value
 n42 = 564 – 164 = 400 Hz
Frequency of 42nd fork is 400 Hz. [½]
Q.4. Attempt any THREE
i. Solution:
Given: m = 600 kg, g = 9.8 m/s2, d = 5000 m, R = 6400 km = 6.4  106 m
To find: Decrease in weight of body (dW)
 d
formula: gd = g 1  
 R
Calculation: Weight of body on the earth surface,
W = mg = 600  9.8 = 5880 N [½]
Since, Wd = mgd
From formula,
 d
Wd = mg 1   [½]
 R
 5  103 
= 5880 1  6 
[½]
 6.4  10 
 6.4  0.005 
= 5880  
 6.4 
= {antilog{log(5880) + [log(6.395)  log(6.4)]}}
= {antilog[3.7694 + (0.8058 – 0.8062)]}
= {antilog(3.7694 + 1.9996)}
= {antilog(3.7690)}
 Wd = 5875 N [1]
 Decrease in weight = W  Wd
= 5880 – 5875
dW = 5N
Decrease in weight is 5 N. [½]
ii. Theorem of parallel axes:
The moment of inertia of a body about any axis is equal to the sum of its moment of inertia
about a parallel axis passing through its centre of mass and the product of its mass and the
square of the perpendicular distance between the two parallel axes.
8
Physics
Mathematically, IO = IC + Mh2
where IO = M.I of the body about any axis passing through centre O.
IC = M.I of the body about parallel axis passing through centre of mass.
h = distance between two parallel axes. [1]
Proof:
a. Consider a rigid body of mass ‘M’ rotating about an axis passing through a point O as
shown in the figure.
Let C be the center of mass of the body, situated at distance h from the axis of rotation.

P(dm)
M
r ro

O C D
h x
IO IC
[½]
b. Consider a small element of mass dm of the body, situated at a point P.
c. Join PO and PC and draw PD perpendicular to OC when produced.
d. M.I of the element dm about the axis through O is OP2 dm
 M.I of the body about the axis through O is given by
IO =  OP 2 dm =  r 2 dm ....(1)
e. M.I of the element dm about the axis through C is CP2 dm
 M.I of the body about the axis through C
IC =  CP 2 dm =  ro2 dm ....(2)
f. From the figure,
OP2 = OD2 + PD2
= (OC + CD)2 + PD2
= OC2 + 2 OC . CD + CD2 + PD2
 CP2 = CD2 + PD2
 OP2 = OC2 + 2 OC . CD + CP2
i.e. r2 = h2 + 2hx + ro2 ....(3) [½]
g. From equation (3) and (1),
 h 
2
IO =  2hx  ro2 dm

 h dm +  2 h.x dm+ r dm
2 2
= o

= h  dm + 2 h  x dm +  r dm
2 2
o

IO = h  dm + 2h  x dm + I
2
C [From equation (2)]
 IO = IC + h  dm + 2 h  x dm
2
....(4)
h. Since  dm = M and  x dm = 0.
algebraic sum of the moments of the masses of its individual particles about the centre
of mass is zero for body in equilibrium. [½]
 Equation (4) becomes,
IO = IC + Mh2
Hence proved. [½]
9
Board Answer Paper : March 2016
iii. Expression for excess pressure inside a bubble:
Po
Pi

a. Free surface of drops or bubbles are spherical in shape.


Let,
Pi = inside pressure of a drop or air bubble
Po = outside pressure of bubble
r = radius of drop or bubble.
b. Let the radius of drop increases from r to r + r so that inside pressure remains constant.
c. Initial area of drop A1 = 4r2, Final surface area of drop A2 = 4(r+r)2 [½]
Increase in surface area,
A = A2  A1 = 4[(r + r)2  r2]
= 4[r2 + 2rr + r2  r2]
= 8rr + 4r2
d. As r is very small, the term containing r2 can be neglected.
 A = 8rr [½]
e. Work done by force of surface tension,
dW = TA = (8rr)T ….(1) [½]
But, dF = (Pi  Po) A [½]
 dW = Fr = (Pi  Po) Ar [½]
From equation (1),
(Pi  Po) Ar = (8rr) T
8πrrT
 Pi  Po = [ A = 4r2]
4πr 2 r
2T
 Pi  Po = ….(2) [½]
r
In case of soap bubble, there are two free surfaces in contact with air.
4T
 Excess pressure, Pi  Po =
r
Equation (2) represents excess pressure inside a drop or air bubble. It is also called Laplace’s
law of spherical membrane.

iv. Solution:
Given: A = 1.5 mm2 = 1.5  10–6 m2, lateral strain = 1.5  10–5,
Ysteel = 2  1011 N/m2,  = 0.291,
g = 9.8 m/s2
To find: Mass attached (M)
lateral strain
Formulae: a. = [½]
longitudinal strain
longitudinal stress
b. Y= [½]
longitudinal strain

10
Physics
Calculation: From formula,
longitudinalstress
Ysteel =
 lateralstrain 
 
  
Mg
=
 lateralstrain 
A 
  
Y  A  lateralstrain
 M = steel [½]
g
2  1011  1.5  106  1.5  105
= [½]
0.291  9.8
4.5
 M=
0.291  9.8
= {antilog{log(4.5)  [log(0.291) + log(9.8)]}}
= {antilog[0.6532  (1.4639 + 0.9912)]}
= {antilog[0.6532 – 0.4551]}
= {antilog(0.1981)}
= 1.578 kg
Mass attached to the wire is 1.578 kg. [1]
SECTION – II
Q.5. Attempt any SIX:
i. The bending of light near the edge of an obstacle or slit and spreading into the region of
geometrical shadow is called diffraction of light. [1]
Two types of diffraction are:
a. Fresnel diffraction:
Diffraction pattern in which source of light and screen are kept at finite distance from the
slit is called fresnel diffraction.
eg: Diffraction of straight edge, small opaque disc, narrow rectangular slit, etc. [½]
b. Fraunhofer diffraction:
Diffraction pattern in which, the source of light and the screen are effectively at infinite
distances from the diffracting system is called Fraunhoffer diffraction.
In this diffraction pattern convex lens is used.
eg: Diffraction due to single slit, double slit, etc. [½]
ii. Construction of cyclotron:

Electromagnet
N Coils

dees

(Diagram and labelling) (1 + 1) [2]


11
Board Answer Paper : March 2016
iii.
No. Paramagnetic substance Ferromagnetic substance
i. It is weakly attracted by a magnet. It is strongly attracted by a magnet.
ii. When kept in a non-uniform magnetic When kept in a non-uniform magnetic
field, it shows moderate tendency to field, it shows strong tendency to
move from weaker to the stronger part move from weaker to the stronger part
of the field. of the field.
iii. When kept in an external magnetic When kept in an external magnetic
field it becomes weakly magnetised, field it becomes strongly magnetised,
and the direction of magnetic moment and the direction of magnetic moment
acquired will be same as that of the acquired will be same as that of the
field. field.
iv. When the external magnetic field is When the external magnetic field is
removed, the paramagnetic substance removed, the ferromagnetic substance
loses its magnetism. retains magnetism permanently.
v. They cannot be converted into When heated above curie temperature,
ferromagnetic substances. they become paramagnetic substances.
vi. Every atom has some magnetic dipole The resultant magnetic dipole moment
moment but resultant dipole moment is greater.
is zero.
vii. They can be temporarily magnetised They can be permanently magnetised.
in external magnetic field.
(Any two correct points of difference) (1  2) [2]
iv. Surface wave propagation:
When the electromagnetic waves (radiowaves) from the transmitting antenna propagate
along the surface of the earth so as to reach the receiving antenna, the wave propagation is
called surface wave propagation (ground wave propagation).
Explanation:
a. Ground waves are the radiowaves which propagate along the surface of the earth.
b. The electromagnetic waves which are vertically polarised, can travel along the surface
of earth. The horizontal component of electric field in contact with earth is short
circuited. The electric field vector of the wave induces charge in the earth.
c. There is loss of power in a signal during its propagation on the surface of the earth due
to partial absorption of energy by ground. Loss of energy is also due to diffraction
effect. The absorption of energy is high for high frequency.
Hence ground wave propagation is suitable for low frequency and medium frequency.
It is used for local broadcasting For examples: ship, communication, radio navigation.
d. Ground wave propagation is possible only when the transmitting and receiving antenna are
close to the earth’s surface. [2]
v. Solution:
Given: G = 500 , Req. = 21 
To find: Shunt Resistance (S)
1 1 1
Formula:   [½]
R eq G S
Calculation: From formula,
1 1 1
= 
S R eq G
G  Req
=
R eq  G

12
Physics
R eq  G
 S=
G  R eq
21  500
= [½]
500  21
10500
=
479
= {antilog[log(10500)  log(479)]}
= {antilog[4.0212  2.6803]}
= {antilog(1.3409)}
 S = 21.93 
The value of shunt resistance is 21.93 . [1]
vi. Solution:
Given: 1 = 1.8  10–5, T1 = 200 K
1  2 = 6  10–6
To find: Required temperature (T2)
Formula: T = constant
Calculation: Given 1  2 = 6  10–6
 2 = 1.8  10–5  0.6  10–5
= 1.2  10–5
From formula,
1T1 = 2T2
T
 T2 = 1 1 [½]
2
1.8  105  200
= [½]
1.2  105
= 300 K
The required temperature is 300 K. [1]
vii. Solution:
Given: M = 2H, dI = 4A, dt = 2.5  10–4 s
To find: The induced e.m.f.(e)
dI
Formula: e=M [½]
dt
Calculation: From formula,
4
e=2 [½]
2.5  104
= 2  4  0.4  104
= 3.2  104 V
The induced e.m.f. in the coil is 3.2  104 V. [1]
viii. Solution:
Given:  = 4.33  10–4 year–1
To find: Half life period (T)
0.693
Formula: T= [½]

Calculation: From formula,
0.693 6.93
T= 4
=  103 [½]
4.33  10 4.33
= {antilog[log(6.93)  log(4.33)]}  103
13
Board Answer Paper : March 2016
= {antilog(0.8407 – 0.6365)}  103
= 1.601  103
= 1601 years
Half-life period of a radioactive element is 1601 years. [1]
Q.6. Select and write the most appropriate answer from the given alternatives for each
sub-question:
i. (D) [1]
ii. (C) [1]
iii. (B) [1]
iv. (A) [1]
v. (D) [1]
vi. (B) [1]
vii. (C) [1]
Q.7. A. Principle of transformer:
It is based on the principle of mutual induction i.e., whenever the magnetic flux linked with a
coil changes, an e.m.f. is induced in the neighbouring coil. [1]
Construction:
i. A transformer consists of two sets of coils P and S insulated from each other. The coil
P is called the primary coil and coil S is called the secondary coil.
ii. The two coils are wound separately on a laminated soft iron core.
iii. The a.c input voltage is applied across the primary and the induced output a.c voltage is
obtained across the secondary, which is used to drive current in the desired circuit.
iv. The two coils are electrically insulated from each other but they are magnetically
linked.
v. To minimise eddy currents, the soft iron core is laminated. [½]
Soft iron core

Input a.c.
voltage  P S
Load

P S

core
[½]

Working:
i. When an alternating voltage is applied to the primary coil the current through the coil
goes on changing. Hence, the magnetic flux through the core also changes.
ii. As this changing magnetic flux is linked with both the coils, an e.m.f is induced in each
coil.
iii. The amount of the magnetic flux linked with the coil depends upon the number of turns
of the coil.
iv. Let, ‘’ be the magnetic flux linked per turn with both the coils at certain instant ‘t’.
v. Let ‘NP and ‘NS’ be the number of turns of primary and secondary coil,
NP = magnetic flux linked with the primary coil at certain instant ‘t’
NS = magnetic flux linked with the secondary coil at certain instant ‘t’
14
Physics
vi. Induced e.m.f produced in the primary and secondary coil is given by,
dP d
eP =  =  NP ….(1)
dt dt
d d
eS =  S =  NS ….(2)
dt dt
vii. Dividing equation (2) by (1),
eS N
 = S ….(3) [1]
eP NP
Equation (3) represents equation of transformer.
N
The ratio S is called turns ratio (transformer ratio) of the transformer.
NP
viii. For an ideal transformer,
Input power = Output power
 ePIP = eSIS
eS I
 = P ….(4)
eP IS
ix. From equation (3) and (4),
eS N I
= S = P [1]
eP NP IS

B. Solution:
Given: Q = 0.2 C = 0.2  10–6 C
A = 40 cm2 = 40  10–4 m2
0 = 8.85  10–12 S.I.units
To find: a. Electric field intensity (E)
b. Mechanical force per unit area (f)

Formulae: a. E=
0 k
1
b. f= 0kE2 [½]
2

Q
c. =
A
Calculation: Using formula (a) and (c),
Q
E= [½]
A 0 k
0.2  106
= ….(k = 1 for air)
40  104  8.85  1012
20  108
=
3.540  1014
= {antilog[log(20)  log(3.54)]}  106
= {antilog(1.3010 – 0.5490)}  106
= {antilog(0.7520)}  106
= 5.649  106 V/m
Electric intensity is 5.649  106 V/m [1]
15
Board Answer Paper : March 2016
From formula (b)
1
f =  8.85  10–12  (5.649  106)2
2
= 4.425  10–12  (5.649)2  1012
= {antilog[log(4.425) + 2log(5.649)]}
= {antilog[0.6459 + (2  0.7520)]}
= {antilog[2.1499]}
 f = 141.2 N/m2
Mechanical force per unit area is 141.2 N/m2 [1]
OR
A. Geiger-Marsden experiment:
a. The experimental arrangement is as shown in the figure.
Screen
Lead Gold
bricks foil


Source of
 particles

Detector
[1]
b. In this experiment, a narrow beam of -particles from radioactive source was incident
on a gold foil.
c. The scattered -particles were detected by the detector fixed on rotating stand. Detector
used had zinc sulphide screen and microscope.
d. -particles produced scintillations on screen which could be observed through
microscope.
e. The whole setup is enclosed in an evacuated chamber.
Observation:
a. Most of the -particles passed undeviated.
b. Only few -particles (about 0.14%) were scattered by more than 1.
c. Some -particles were deflected slightly and very few (1 in 8000) deflected by more
than 90.
d. Some -particles were bounced back with  = 180. [2]
Mass defect:
The difference between the actual mass of the nucleus and the sum of masses of
constituent nucleons is called mass defect.
Let,
M = measured mass of nucleus
A = mass number
Z = atomic number
mp = mass of hydrogen atom
mn = mass of free neutron
(A  Z) = number of neutrons
 Mass defect,
m = [Zmp + (A Z)mn]  M [1]
16
Physics
B. Solution:
Given: 0 = 2.3 eV,  = 6800 Å = 6.8  10–7 m
c = 3  108 m/s, h = 6.63  10–34 Js
To find: a. Threshold frequency (0)
b. Incident frequency ()
Formulae: a. 0 = h0
c
b. = [½]

Calculation: From formula (a),

0 = 0 [½]
h
2.3  1.6  1019
= [½]
6.63  1034
.68  1015
=
6.63
36.8
=  1014
6.63
= {antilog[log(36.8) – log(6.63)]}  1014
= {antilog(1.5658 – 0.8215)}  1014
= {antilog(0.7443)}  1014
= 5.550  1014 Hz
Threshold frequency 0 is 5.55  1014 Hz. [½]
From formula (b)
3  108
=
6.8  107
30  107
=
6.8  107
= {antilog[log(30)  log(6.8)]}  1014
= {antilog(1.4771  0.8325)}  1014
= {antilog(0.6446)}  1014
= 4.412  1014 Hz
Frequency of incident light is 4.412  1014 Hz. [½]
As  < 0, emission of photoelectrons is not possible. [½]
Q.8. Attempt any THREE:
i. Solution:
14
Given: ag = 1.5,  = 3.5  10 Hz,
8
c = 3  10 m/s
To find: a. Change in wavelength of light ()
b. 
Wave number of light 

Formulae: a.    = 1
a
b. ag =
g
c. c =  [1]
17
Board Answer Paper : March 2016
Calculation:
a. Using formula (c),
c 3  108 6
a =  =  106 = 0.8571  106 m
 3.5  10 14
7
 a = 8571 Å [½]
Using formula (b),

a 8571A
g = =
a g 1.5
 g = 5714 Å [½]
  = a  g = 8571  5714
  = 2857 Å
The change in wavelength of light is 2857 Å. [½]
b. 
Now, using formula (a),  =
1
g
1
 = = 1.75  106 m1
5.714  107
The wave number of light is 1.75  106 m1. [½]
ii. Solution:
Given: x10 = 2.09 mm, R = 6400 Å, B = 4800 Å
To find: Change in fringe width (X)
D D
Formulae: a. X= b. x = (2m  1)
d 2d
Calculation: Using formula (b),
D
x10 = (2(10) – 1)
2d
19
2.09 =  XR
2
2.09  2
 XR = [½]
19
4.18
=
19
= 0.22 mm [½]
From formula (a) we can conclude, for same setting,
X
X1 X 2
 
1  2

i.e. XB = B  XR [½]
R
4800Å
=  0.22 mm
6400Å
0.66
=
4
= 0.165 mm [½]
 change in fringe width (X) = XR  XB
= 0.22 – 0.165 = 0.055 mm
The change in fringe width is 0.055 mm. [1]
18
Physics
iii. Kelvin’s method to determine the resistance of a galvanometer:
Construction:
a. In Kelvin’s method, the galvanometer (G) whose resistance is to be determined is
connected in the left gap of a metrebridge and a known resistance R is connected in the
right gap.
b. A jockey (J) is connected directly to the point B and it can slide along the wire.
G R
B
T1 T2
T3
lg D J
A lR
C
Scale
0 10 20 30 40 50 60 70 80 90 100

+  ( )
E K Rh
G : Galvanometer
R : Resistance from resistance box
AC : Metal wire one metre long
Rh : Rheostat
E : Cell
K : Plug key
J : Jockey [1]
c. A cell of e.m.f ‘E’ is connected between points A and C of the wire in series with a
high resistance box.
d. The rheostat is used to adjust the deflection in the galvanometer to half of its maximum
value. Hence, this method is also called half current method or half scale method.
e. First the deflection in the galvanometer is adjusted at half of its original value and the
reading is noted. It acts as null position.
f. The value of R is adjusted, so that the galvanomter gives a fairly large deflection, i.e.,
full scale deflection.
If the jockey is touched to different points on the wire then galvanometer shows
increase or decrease in the deflection.
g. A point D is located on the wire so that when the jockey is touched at that point,
galvanometer shows the same deflection as before. It means that point D and B are at
the same potential, i.e., bridge is balanced. [½]
Working:
a. Let,
lg = length of wire corresponding to left gap
lR = length of wire corresponding to right gap
G = resistance of galvanometer
b. In the balanced condition,
G Resistance of wire of length lg
= [½]
R Resistance of wire of length lR
G  lg l
 = = g
R  lR lR
where,
 = resistance per unit length of wire
lg
 G = R. [1]
lR
19
Board Answer Paper : March 2016
c. Since lg + lR = 100 cm
 lR = (100  lg)
 lg 
 G =R  
 100  lg 
Measuring lg and R, value of G can be determined.
iv. Principle of working of an oscillator:
+ 
d. c. supply

Input Transistor amplifier Output


Vi (A) Vo = AVi

Feedback Network
Vf = Vo
()
[1]
a. A simple oscillator consists of an amplifier and feedback network with frequency
determining components.
b. A frequency-determining network, (resonant tank circuit) which also works as feedback
network and transistor amplifier acts as element.
c. With enough feedback, the oscillations start as soon as the circuit is switched on.
d. With positive feedback, the output current of the amplifier will be in the right phase to
increase the alternating current in the resonant circuit.
e. The oscillations then built up in amplitude until the power losses in the circuit are equal
to the power that the amplifier can develop.
f. The natural frequency of the oscillator is close to the resonant frequency of the resonant
circuit. [1]
V
g. Suppose the voltage gain without feedback of the amplifier is A = o .
Vi
h. The feedback factor  is the fraction of the output voltage fed back to the input,
Vi = Vf = Vo
Vo 1
 A= =
Vi 
 A = 1
i. The condition A = 1, is called Berkhausen’s criterion. It states that the phase shift of
the feedback voltage will be zero or integral multiple of 2 rad, i.e., there will be
positive feedback. [½]
A
j. The voltage gain of complete system is given by, Af = .
1  A
Thus, for the frequency for which A = 1, Af will be infinite, i.e., the circuit will
operate without any external signal voltage, which means the circuit will oscillate at
that frequency. [½]

20
Physics

BOARD ANSWER PAPER : JULY 2016


PHYSICS
SECTION – I
Q.1. Select and write the most appropriate answer from the given alternatives for each
sub-question:
i. (C) [1]
ii. (A)
1
E= I2
2
 2E = (I)
2E
 =L [1]

iii. (A) [1]
iv. (C) [1]
v. (D)
2T cos 
h=
rg
1
 h [1]
r
vi. (A) [1]
vii. (B)
1 2
P= c
3
c1 P1 2 3
 =  = [1]
c2 P2 1 2

Q.2. Attempt any SIX :


i. N N cos 


N sin 
G C
F cos  

F h [1]
F sin 

A
B

W = mg
AC : inclined road surface
AB : horizontal surface
BC : height of road surface
G : centre of gravity of vehicle
W : (mg) weight of vehicle
N : normal reaction exerted on vehicle
 : angle of banking [1]
1
Board Answer Paper : July 2016
ii. Expression for critical velocity:
m
vC
h
r

M [½]
Earth R

a. Let,
M = mass of the earth
R = radius of the earth
h = height of the satellite from the earth’s surface
m = mass of the satellite
vc = critical velocity of the satellite in the given orbit
r = (R + h) = radius of the circular orbit [½]
b. For the circular motion of the satellite, the necessary centripetal force is given as,
mvc2
FCP = ….(1)
r
c. The gravitational force of attraction between the earth and the satellite is given by,
GMm
FG = .…(2)
r2
d. Gravitational force provides the centripetal force necessary for the circular motion of
the satellite.
 FCP = FG
mv c2 GMm
 = ….[From equations (1) and (2)] [½]
r r2
GM
 vc2 =
r
GM
 vc = .…(3)
r
e. But, r = R + h
GM
 vc = ….(4) [½]
(R  h)
Equation (4) represents the expression for critical velocity.
iii.
T Tcos  T
 
r T cos   2r
Tsin  r T sin 
  h
[1]

Water

Rise of liquid in capillary tube


r = radius of capillary tube
h = height of liquid level in the tube
2
Physics
T = surface tension of liquid
 = density of liquid
g = acceleration due to gravity [1]
iv. a. A gas consists of very large number of extremely small particles known as molecules. [½]
b. The intermolecular force of attraction between gas molecules are negligible. [½]
c. Molecules are always in the state of random motion, i.e., they are moving in all possible
directions with all possible velocities. This state is called molecular chaos. [½]
d. Between any two successive collisions, a molecule travels in a straight line with
constant velocity. It is called free path. [½]
v. Given: L = 47 cm
= 47  10–2 m
v = 3.3  102 m/s
To find: Fundamental frequency of air column (n)
v
Formula: n= [½]
2L
Calculation: From formula,
3.3  102
n= [½]
2  47  102
330
=  102
94
= antilog [log(330) – log(94)]  102
= antilog[2.5185 – 1.9731]  102
= antilog[0.5454]  102
= 3.512  102
 n = 351.2 Hz
The fundamental frequency of air column is 351.2 Hz [1]
vi. Given: R = 0.1 m
 = 6  103 kg/m3
To find: M.I of sphere about a tangent to its surface (It)
7
Formula: It = MR2 [½]
5
7
Calculations: It = MR2
5
4
But M = V = R 3
3
74 3  2
 It =  R   R
5 3 
28 5
= R  [½]
15
28
=  3.14  (0.1)5  6  103
15
28  3.14  6
=  102
15
56  3.14
=  102
5
= antilog[log(56) + log(3.14) – log(5)]  10–2
= antilog[1.7482 + 0.4969 – 0.6990]  10–2
3
Board Answer Paper : July 2016
= antilog[1.5461]  10–2
= 35.17  10–2
 It = 0.3517 kg m2
Moment of inertia of solid sphere about a tangent to its surface is 0.3517 kg m2. [1]
vii. Given: T = 10 s
1
P.E = (T.E) [½]
2
To find: Time (t)
1 1
Formula: i. P.E = m2x2 ii. T.E = m2A2
2 2
iii. x = Asint
Calculation: From formula i. and ii.,
1 11 
m2 x 2 =  m2 A 2  … (From given condition)
2 2 2 
A
 x=
2
Using formula iii., x = Asint [½]
A
Asint =
2
1
 sint =
2
 1 
 t = sin–1  
 2
 2  
  t = [½]
 T  4
T 10
 t= =
8 8
 t = 1.25 s
The time in which the potential energy will be half of total energy is 1.25 s. [½]
viii. Given: m = 2 kg
l = 1.5 m
 = 30
g = 9.8 m/s2
To find: Period (T)
l cos 
Formula: T = 2 [½]
g
Calculation: From formula,
1.5  cos30
T = 2  3.14 [½]
9.8
1.5  0.8660
= 6.28 
9.8

= antilog log  6.28    log1.5  log 0.8660  log 9.8  


1
 2 

= antilog 0.7980   0.1761  1.9375  0.9912  


1
 2 

4
Physics

= antilog 0.7980   0.1136  0.9912 


1
 2 

= antilog 0.7980   0.8776  


1
 2 
= antilog[0.3592]
 T = 2.2875 s
Period of revolution is 2.287 s [1]
Q.3. Attempt any THREE
i. Kepler’s first law (Law of orbit):
Every planet revolves around the sun in an elliptical orbit with the sun situated at one of the
focii of the ellipse. [1]
Kepler’s second law (Law of equal areas):
The radius vector drawn from the sun to any planet sweeps out equal areas in equal intervals
of time, i.e., areal velocity of the radius vector is constant. [1]
Kepler’s third law (Law of periods):
The square of the period of revolution of the planet round the sun is directly proportional to
the cube of the semi-major axis of the elliptical orbit. [1]
ii. Expression for torque acting on a rotating body:

m3 


F3 F2
r3 O r m2
2
mn 
rn r1 F1

Fn m1

a. Suppose a rigid body consists of n particles of masses m1, m2, m3, ......, mn which are
situated at distances r1, r2, r3, …, rn respectively, from the axis of rotation as shown in
figure.
b. Each particle revolves with angular acceleration .
c. Let F1, F2, F3, …., Fn be the tangential force acting on particles of masses,
m1, m2, m3, …, mn respectively. [1]
d. Linear acceleration of particles of masses m1, m2,…, mn are given by,
a1 = r1, a2 = r2, a3 = r3, …, an = rn
e. Magnitude of force acting on particle of mass m1 is given by,
F1 = m1a1 = m1r1 [ a = r] [½]
Magnitude of torque on particle of mass m1 is given by,
1 = F1 r1 sin 
But,  = 90 [ Radius vector is ar to tangential force]
 1 = F1 r1 sin 90
= F1r1
= m1a1 r1
 1 = m1r12  [ a1 = r1 ] [½]

5
Board Answer Paper : July 2016
Similarly,
2 = m 2 r22 
3 = m3 r32 ,

.…

.…
n = mn rn2 
f. Total torque acting on the body,
 = 1 + 2 + 3 + …. + n
  = m1r12  + m 2 r22 + m3 r32  + … + m n rn2  [½]
2 2 2
  = (m1r1 + m2r2 + m3r3 +.....+ m r ) 2
n n

 n 
 
 =  m i ri 2  
 i 1 
n
But, m r
i 1
i i
2
=I

  = I [½]
g. If  = 1 rad/s2 then  = I.
Thus, when a torque rotates the body with uniform angular acceleration of 1 rad/s2 then
M.I of the body about a given axis of rotation becomes equal to torque acting on it.
iii. Given: A = 2 mm2 = 2  10–6 m2
F = 10 N
Ysteel = 2  1011 N/m2
 = 0.29
To find: Lateral strain
longitudinalstress
Formula: i. Y =
longitudinalstrain
lateralstrain
ii.  =
longitudinalstrain
F
Calculation: Longitudinal stress =
A
10
=
2  106
= 5  106 N/m2 [½]
Using formula i.,
longitudinalstress
longitudinal strain = [½]
Y
5  10 6
=
2  1011
= 2.5  10–5 [½]
Using formula ii,
lateral strain =   longitudinal strain [½]
= 0.29  2.5  10–5
 lateral strain = 7.25  10–6
Lateral strain produced in the wire is 7.25  10–6. [1]
62  54
iv. Given: 1 = = 58 C in 10 minutes
2
54  48
2 = = 51 C in next 10 minutes
2
6
Physics

 d  62  54 8
  = = = 0.8 C/min
dt
 1 10 10
 d  54  48 6
  = = = 0.6 C/min
 dt  2 2 10
To find: Temperature of surroundings (0)
d
Formula: = K( – 0) [½]
dt
Calculation: From formula,
 d 
  = K(1 – 0)
 dt 1
 0.8 = K(58 – 0) ….(1) [½]
and
 d 
  = K(2 - 0)
 dt  2
 0.6 = K(51 – 0) ….(2) [½]
Dividing equation (1) by (2)
0.8 58  0
 =
0.6 51  0
8 58  0
 = [½]
6 51  0
 8(51 – 0) = 6(58 – 0)
408 – 80 = 348 – 60
 20 = 408 – 348
 20 = 60
 0 = 30C
Temperature of surroundings is 30C [1]
Q.4. A. Formation of stationary waves by analytical method:
i. Consider two identical progressive waves of equal amplitude and frequency travelling
along X axis in opposite direction. They are given by,
2
y1 = A sin (vt  x) along positive X-axis .…(1)

2
y2 = A sin (vt + x) along negative X-axis ….(2) [½]

ii. The resultant displacement ‘y’ is given by the principle of superposition of waves,
y = y 1 + y2 ….(3)
2 2
y = A sin (vt  x) + A sin (vt + x) [½]
 
iii. By using,
CD C D
sin C + sin D = 2sin   cos  ,
 2   2 
we get,
 2  vt  x  vt  x    2  vt  x  vt  x  
y = 2A sin    cos   
   2    2 
 2vt   2 
= 2Asin   cos  ( x) 
     
7
Board Answer Paper : July 2016

 2x   v
 y = 2Asin 2πnt cos    n   [cos ( ) = cosθ]
     
 2x 
 y = 2Acos   sin 2πnt [½]
  
 2x 
iv. Let R = 2Acos  
  
 y = Rsin (2πnt) ….(4) [½]
But,  = 2n
 y = R sin t ….(5)
Equation (5) represents the equation of S.H.M. Hence, the resultant wave is a S.H.M.
of amplitude R which varies with x.
v. The absence of x in equation (5) shows that the resultant wave is neither travelling
forward nor backward. Therefore it is called as stationary wave.
Amplitude at node is minimum, i.e., 0.
 Rmin = 0
 2 x 
Since R = 2A cos  ,
  
 2 x 
 cos   =0 [½]
  
2x  3 5
 = , , , …….
 2 2 2
 3 5
 x= , , ,…….
4 4 4
Distance between two consecutive nodes,
3  
x1  x0 =  = ,
4 4 2
5  3  
x2  x1 =  = and so on.
4 4 2

Thus, distance between two successive nodes is . [½]
2
 2x 
Since R = 2A cos  ,
  
At antinodes: R =  2A
 2x 
 cos   = 1 [½]
  
 2x 
   = 0, , 2, 3,…….n
  
 3
 x = 0, ,  , ………
2 2
 Distance between two consecutive
  
antinodes = x1  x0 = , x2  x1 =  = and so on.
2 2 2
Thus, distance between two successive antinodes is /2.
Hence, nodes and antinodes are equispaced. The distance between a node and an
λ
adjacent antinode is . [½]
4
8
Physics
B. Given: Speed limit, vL = 120 km/hr
10
nA = nA – nA = 0.9 nA [½]
100
Velocity of sound, v = 340 m/s
To find: Velocity of source (vS)
 v 
Formula: nA =  n .…(1) [½]
 v  vS 
 v 
nA =  n ….(2) [½]
 v  vS 
Calculation: Divide equation (1) by (2)
n A v  vS
 = [½]
n A v  vS
1 340  vS
 = [½]
0.9 340  vS
 340 – vS = 0.9(340 + vS)
 340 – vS = 306 + 0.9 vS
 1.9 vS = 34
34 340
 vS = =
1.9 19
= 17.89 m/s
 vS = 64.40 km/hr [½]
 VS < VL
Hence, the policeman is not justified in punishing the car driver.
OR
A. Practical simple pendulum:
Practical simple pendulum is defined as a small heavy sphere (bob), suspended by a light and
inextensible string from a rigid support. [½]
To show motion of the bob of simple pendulum is S.H.M:
O

l
T T
mg sin 
B
A x 
mg cos 
mg mg
i. Consider a simple pendulum of mass ‘m’ and length ‘L’.
L = l + r,
where, l = length of string
r = radius of bob
ii. Let OA be the initial position of pendulum and OB, its instantaneous position when
the string makes an angle  with the vertical.
In displaced position, two forces are acting on the bob:
a. Gravitational force (weight) ‘mg’ in downward direction.
b. Tension T in the string.
9
Board Answer Paper : July 2016
iii. Weight ‘mg’ can be resolved into two rectangular components:
a. Radial component mg cos  along OB and
b. Tangential component mg sin  perpendicular to OB and directed towards
mean position. [1]
iv. mg cos  is balanced by tension T in the string, while mg sin  provides restoring force.
 F =  mg sin  [½]
where, negative sign shows that force and angular displacement are oppositely directed.
Hence, restoring force is proportional to sin  instead of . So, the resulting motion is
not S.H.M.
v. If  is very small then,
x
sin    =
L
x
 F =  mg [½]
L
F x
 =g
m L
ma x
 =g
m L
g
 a= x .…(1)
L
 g 
 ax ....   constant  [½]
 L 
Hence, motion of the bob of simple pendulum is simple harmonic.
Expression for time period:
In S.H.M,
a =  2 x ….(2)
Comparing equations (1) and (2),
g
2 =
L
2
But,  = [½]
T
2
 2  g
   =
 T  L
2 g
 =
T L
L
 T = 2 ….(3) [½]
g
Equation (3) represents time period of simple pendulum.
Thus period of simple pendulum depends on the length of the pendulum and
acceleration due to gravity. [1]

B. Given: E =  2 T
To find: Diameter of drop (d)
Formula: E = TA
Calculation: A = 4r2
From formula,
E = 4r2T
10
Physics

  2 T = 4r2T [½]
 4r2 = 2
2
 r2 =
4
1.414
=
4
r2 = 0.3535
 r = 0.3535
r = 0.5946 m [1]
 d = 2r
= 2(0.5946)
 d = 1.1892 m
Diameter of the drop is 1.1892 m. [½]
SECTION – II
Q.5. Select and write the most appropriate answer from the given alternatives for each
sub-question:
i. (B) [1]
ii. (D) [1]
iii. (C) [1]
iv. (B) [1]
v. (C) [1]
vi. (A) [1]
vii. (B)
e = v lb
5
= 54   1  3  104
18
= 45  10–4 = 4.5 mV [1]
Q.6. Attempt any SIX:
i.
No. Intrinsic semiconductors Extrinsic semiconductors
i. Semiconductor in the pure form The semiconductor, resulting from mixing of
is known as intrinsic impurity in it, is known as extrinsic
semiconductor. semiconductor.
ii. Their conductivity is low. Their conductivity is high.
iii. Its electrical conductivity is a Its electrical conductivity depends upon the
function of temperature alone. temperature as well as on the quantity of
impurity atoms doped in the structure.
iv. The number of free electrons in In these semiconductors, number of free
conduction band is equal to the electrons and number of holes are unequal.
number of holes in valence
band.
v. These are not practically used. These are practically used.
vi. In these, the Fermi energy level In these, the Fermi energy level shifts towards
lies in the middle of valence valence or conduction energy band.
band and conduction band.
(Any two correct points of difference) (1+1) [2]
11
Board Answer Paper : July 2016
ii. Receiving
Antenna

Output
Amplifier IF Stage Detector Amplifier
Received
signal
Block diagram of a receiver
(Diagram and labelling) (1+1) [2]
2
iii. Given: x1 = 6.5 cm = 6.5  10 m,
x2 = 6.65 cm = 6.65  102 m,
 = 5000 Å = 5  107 m
To find: Nature of illumination at the point
Formula: x=n
Calculation: Path difference is given by,
∆x = x2  x1 = 6.65  102  6.5  102
= 0.15  102 m
From formula,
x
n= [1/2]
λ
0.15×10 –2
n= [½]
5×10 –7
= 3000 [½]
λ
 ∆x = 3000 = 6000 
2
λ
As the path difference is even multiple of ,
2
 The point is bright. [1/2]
iv. Magnetic dipole moments in paramagnetic substance:

B

S N

(a): Absence of external field (b): Strong external magnetic field [2]
v. Given: R = 100 , r = 20 , E = 2 V
To find: Reading of voltmeter (V)
Formula: V = E  Ir [1/2]
Calculation: Current through the circuit is given by
E 2 2
I= = = [1/2]
R  r 100 + 20 120
1
 I= A
60
From formula,
 1 
V = 2    20  = 2  0.3333
 60 
 V = 1.667 V
The reading on the voltmeter is 1.667 V. [1]
12
Physics
vi. Given: 1 = 1.2  105, T1 = 300 K,
 2 = 1.8  105
To find: Required temperature (T2)
Formula: χT = constant
Calculation: From formula,
1T1 =  2 T2 [1/2]
1 T1 1.2  105  300
 T2 = = [1/2]
2 1.8 105
 T2 = 200 K
The required temperature is 200 K. [1]
vii. Given: V = 25000 volt
To find: De-broglie wavelength ()
12  27
Formula: = Å [1/2]
V
Calculation: Using formula,
12  27
=  10–10 [1/2]
25000
12  27  1010
=
2  5  104
12  27
= 1
 1012
 2  5 2
  1 
= antilog log 12.27   log  2  5     1012
  2 
  1 
= antilog 1  0888   0  3979     1012
  2 
= antilog  0  8899   10 12

= 7761  10–12 m = 007761  10–10 m


The de-broglie wavelength of electron is 00776  10–10 m [1]
viii.
A Output A
Input Output
B Inputs
B
AND gate
OR gate

Input Output A Output


A Inputs
B
NOT gate NAND gate
(1/2  4) [2]
Q.7. Attempt any THREE: [9]
i. a. Let S1 and S2 be the two coherent monochromatic sources which are separated by short
distance d. They emit light waves of wavelength .
b. Let D = horizontal distance between screen and source.
c. Draw S1M and S2N  AB
OP = perpendicular bisector of slit.
13
Board Answer Paper : July 2016
Since S1P = S2P, the path difference between waves reaching P from S1 and
S2 is zero, therefore there is a bright point at P.
d. Consider a point Q on the screen which is at a distance x from the central point P on the
screen. Light waves from S1 and S2 reach at Q simultaneously by covering path S1Q
and S2Q, where they superimpose. [1/2]
A
Q

x
S1 M
d/2
d O P
x d/2
K
S2 N
D

B [1/2]
e. In  S1MQ,
(S1Q)2 = (S1M)2 + (MQ)2
2
2  2 d
(S1Q) = D +  x   ….(1) [1/2]
 2
f. In  S2NQ,
(S2Q)2 = (S2N)2 + (NQ)2
2
2 2 d
 (S2Q) = D +  x   ….(2) [1/2]
 2
g. Subtract equation (1) from (2),
(S2Q)2  (S1Q)2
 d  
2
d 
2

=  D2   x      D2   x   
  2    2 
   
2 2
 d  d
= D2   x    D2   x  
 2  2
2 2
 d  d
= x   x  
 2  2
 d 2
  d2 
=  x 2   xd    x 2   xd 
 4   4 
2 2
d d
= x 2   xd  x 2   xd
4 4
(S2Q)2  (S1Q)2 = 2xd [1/2]
 (S2Q + S1Q) (S2Q  S1Q) = 2xd
2xd
 S2 Q  S1 Q = ….(3)
S2 Q  S1Q
h. If x << D and d << D then,
S1 Q  S2 Q  D
S2Q + S1Q = 2D
 Equation (3) becomes,
2xd
S2 Q  S1 Q =
2D
14
Physics
xd
 S2 Q  S1 Q =
D
xd
 x = ….(4) [1/2]
D
Equation (4) gives the path difference of two interfering light waves.
ii. Law of radioactive decay:
The number of nuclei undergoing the decay per unit time is proportional to the number of
unchanged nuclei present at that instant. [1]
If ‘N’ is the number of nuclei present at any instant ‘t’, ‘dN’ is the number of nuclei that
disintegrated in short interval of time ‘dt’, then according to decay law,
dN
 N
dt
dN
 =  N [1/2]
dt
where,  is known as decay constant or disintegration constant. The negative sign indicates
disintegration of atoms.
Derivation of relation N = N0 et :
a. Let, N = number of nuclei present at any instant t.
dN = number of nuclei disintegrated in short interval dt.
b. According to decay law,
dN
=  N ….(1)
dt
c. Integrating both sides of equation (1),
dN
 N =   dt [1/2]
 loge N = t + c .…(2)
where, c is constant of integration whose value depends on initial conditions.
d. At t = 0, N = N0
 loge N0 = 0 + c [From equation (1)]
e. Substituting the value of c in equation (1), we get,
loge N = t + loge N0
 loge N  loge N0 = t
 N 
 loge   = t
 N0 
N
 = et
N0
 N = N0 e t [1/2]
Decay curve:

N0

No. of unchanged
nuclei N

(NdN)
[1/2]
O t t + dt time

15
Board Answer Paper : July 2016
iii. Given: ag= 15,  = 5  1014 Hz
To find: a. Change in wavelength ()
b. Wave number of light in glass  ν g 
λa 1 c
Formula: a. ag = b. ν = c. =
λg λ ν
Calculation: Using formula (c),
c 3  108
a = = = 06  10–6 m
ν 5  1014
 a = 6000 Å [1/2]
From formula (a),
λ
g = a [½]
a μg

6000
= = 4000 Å [1/2]
1 5
 Change in wavelength  = a  g
= 6000 – 4000
= 2000 Å
Change in wavelength is 2000 Å [1/2]
1
Wave number of light in glass = ν g = [½]
λg
1
=
4000Å
= 25  106 m–1
Wave number of light in glass is 25  106 m–1 [1/2]
iv. Given:  = 3000 Å = 3  10–7 m, me = 91  10–31 kg,
o = 23 eV, h = 663  10–34 Js, c = 3  108 m/s
To find: Maximum velocity (vmax)
hc
Formula: (K.E)max =  o [1/2]
λ
Calculation: From formula,
 6  63  1034  3  108 
(K.E.)max =  7
  2  3  1  6  1019   J [1/2]
 3  10 
= (663  10–19)  (368  10–19 ) J
= 295  10–19 J [1/2]
Also,
1
m e v 2max = (K.E.)max
2
2(K.E.) max
 vmax = [1/2]
me

2  2  95  1019
= [1/2]
9  1  1031
5  90  1012
=
9 1
16
Physics

590
=  105
9 1
 1 
= antilog  log  590   log  9  1    105
 2 
 1 
= antilog   2  7709  0.9590     105
 2 
 1 
= antilog  1  8119     105
 2 
= antilog  0  9059   10 5

= 8052  105 m/s


The maximum velocity of electron is 8052  105 m/s [1/2]
Q.8. A. Electromagnetic induction:
The phenomenon of producing an induced e.m.f in a conductor or conducting coil due to
changing magnetic flux is called electromagnetic induction. [1]
i. Consider a rectangular loop of conducting wire ‘PQRS’ partly placed in uniform
magnetic field of induction ‘B’ as shown in figure.

      
 B  PP 
F1    I Q
      
F       v
l
      
      
x
 S      R
F2  dx
    
[1/2]
ii. Let ‘l’ be the length of the side PS and ‘x’ be the length of the loop within the field.
 A = lx = area of the loop, which lies inside the field. [1]
iii. The magnetic flux () through the area A at certain time ‘t’ is  = BA = Blx [1/2]
iv. The loop is pulled out of the magnetic field of induction ‘B’ to the right with a uniform
velocity ‘v’.
d d
v. The rate of change of magnetic flux is given by, = (Blx)
dt dt
d  dx 
 = Bl  
dt  dt 

But,   = v
dx
 dt 
d
 = Blv .…(1) [1/2]
dt
vi. Due to change in magnetic flux, induced current is set up in the coil. The direction of
this current is clockwise according to Lenz’s law. Due to this, the sides of the coil
experiences the forces, F1, F2 and F as shown in figure. The directions of these forces is
given by Flemings left hand rule.
vii. The magnitude of force ‘F’ acting on the side PS is given by, F = BIl.
17
Board Answer Paper : July 2016
 
viii. The force F1 and F2 are equal in magnitude and opposite in direction, therefore they

cancel out. The only unbalanced force which opposes the motion of the coil is F .
Hence, work must be done against this force in order to pull the coil.
ix. The work done in time ‘dt’ during the small displacement ‘dx’ is given by,
dW = Fdx
 ve sign shows that F and ‘dx’ are opposite to each other.
 dW =  (BIl) dx ….(2) [1/2]
x. This external work provides the energy needed to maintain the induced current I
through the loop (coil).
xi. If ‘e’ is the e.m.f induced then,
dW
electric power = = eI
dt
 dW = eIdt .…(3) [1/2]
xii. From equations (2) and (3),
eIdt =  BIl dx

e =  Bl  
dx

 dt 
 e =  Blv .…(4) [1/2]
xiii. From equation (1) and (4),
d
e= 
dt
B. Solution:
Given: L = 2 m, R = 10 , RE = 990 ,
E=2V
To find: Potential gradient of wire (K)
V
Formula: K= [1/2]
L
E
Calculation: Since, I = [1/2]
R  RE
ER
Also, V = IR=
R  RE
2  10
=
10  990
20
=
1000
V = 2  102 volt
From formula,
2  102
K=
2
 K = 102 V/m = 0.01 V/m
The potential gradient of wire is 0.01 V/m. [1]
OR
18
Physics
A. Construction:
Steel
chamber
Metallic S
comb

P2 Metallic
C2 sphere
D
I
Insulating
M1 M2 belt
C1
HT
P1 Motor
driven pulley

Target [1]
i. It consists of a large hollow metallic sphere S mounted on two insulating columns M1 and
M2.
ii. There are two pulleys P1 and P2. P2 is mounted at the centre of sphere S while P1 is mounted
near the bottom. A long narrow belt made of insulating material passes over the pulleys. The
belt is driven by an electric motor, connected to the lower pulley P1.
iii. C1 and C2 are two pointed metal combs, having a number of metallic needles, pointing
towards the belt. C1 is called the spray comb and C2 is called the collecting comb. C1 is kept
at a positive potential of about 10,000 V by using a high tension source (HT) while C2 is
connected to the sphere S.
iv. D is an evacuated accelerating tube having an electrode I at its upper end. This electrode is
connected to the sphere S. To prevent the leakage of charge from the spray, the generator is
enclosed in a steel chamber filled with nitrogen or methane at high pressure. [11/2]
Working:
i. When the spray comb is maintained at high positive potential, it produces charges in its
vicinity. The positive charges get sprayed on the belt due to the repulsive action of comb
C1, which are carried upward by the moving belt. A comb C2, called collecting comb is
positioned near the upper end of the belt, such that the pointed ends touch the belt and the
other end is in contact with the inner surface of the metallic sphere S.
ii. The comb C2 collects the positive charge and transfers them to the metallic sphere. The
charge transferred by the comb C2 immediately moves on to the outer surface of the hollow
sphere.
iii. As the belt goes on moving, the accumulation of positive charge on the sphere also keeps on
taking place continuously and its potential rises considerably.
iv. With the increase of charge on the sphere, its leakage due to ionisation of surrounding air also
becomes faster. When the rate of loss of charge due to leakage becomes equal to the rate at
which the charge is transferred to the sphere, the potential of sphere becomes maximum.
v. The projectiles such as protons, deuterons etc. are introduced in the upper part of the
evacuated accelerator tube. They get accelerated in the downward direction along the length
of the tube and acquire very high energy. At the other end, they come out and hit the target
with large kinetic energy and bring about nuclear disintegration. [11/2]
19
Board Answer Paper : July 2016
B. Solution:
Given: G = 25 , Ig = 10 mA = 10  10–3 A,
V = 100 V
To find: Resistance (Rs)
V
Formula: Rs = G [1/2]
Ig
Calculation: From formula,
100
Rs =  25 [1/2]
10  103
= 104 – 25
= 10000 – 25 [1/2]
= 9975  [1]
A resistance of 9975  should be connected in series. [1/2]

20
Physics

BOARD ANSWER PAPER : MARCH 2017


PHYSICS
SECTION – I
Q.1. Select and write the most appropriate answer from the given alternatives for each sub-
question:
i. (C) [1]
ii. (C) [1]
iii. (C) [1]
iv. (D) [1]
v. (B) [1]
vi. (A) [1]
vii. (B) [1]
Q.2. Attempt any SIX :
i. a. Force acting on a particle performing circular motion along the radius of circle and
directed towards the centre of the circle is called centripetal force.
mv 2
It is given by FCP =
r
where, r = radius of circular path. [1]
b. Example: Electron revolves around the nucleus of an atom. The necessary centripetal
force is provided by electrostatic force of attraction between positively charged nucleus
and negatively charged electron.
c. Unit: N in SI system and dyne in CGS system.
d. Dimensions: [M1L1T2] [1]
ii. Expression for r.m.s velocity:
a. Let, P = pressure exerted by one mole of an ideal gas
V = volume of one mole of the gas
T = absolute temperature
b. Pressure exerted by gas is given by,
1 Mc 2
P= [½]
3 V
where M = mass of one mole (molecular weight) of the gas.
1
 PV = Mc2 ....(1)
3
c. But for one mole of an ideal gas, PV = RT [½]
1
 RT = Mc2 [From equation (1)]
3
3RT
 c2 =
M
3RT
 c= ….(2) [½]
M
Equation (2) represents expression for r.m.s velocity of gas molecules.
d. As R and M in equation (2) are constant,
 c T [½]
c1 T1
 =
c2 T2

1
Board Answer Paper : March 2017
iii. Differential equation of linear S.H.M:
a. Let a particle of mass ‘m’ undergo S.H.M about its mean position O. At any instant ‘t’,
displacement of particle be ‘x’ as shown in the following figure.

Equilibrium position
O F = kx
Q P R
A A

b. By definition, F =  kx .…(1) [½]


where, k is force constant
c. The acceleration of the particle is given by,
 dx 
d  2
a=
dv
=  dt  = d x [½]
dt dt dt 2
d. According to Newton’s second law of motion,
F = ma [½]
 d2 x 
 F=m  2  .…(2)
 dt 
e. From equations (1) and (2),
d2x
m =  kx
dt 2
d2x k
 2
=  x
dt m
d2x k
 2
+ x=0 ….(3)
dt m
k
where, = 2 = constant
m
d2x
 2
+ 2x = 0 ….(4) [½]
dt
f. Equations (3) and (4) represent differential equation of linear S.H.M.
iv.


FA
A
P
Air
 Liquid
 FC
FR T

Solid

[Diagram and labelling] (1 + 1) [2]

2
Physics
R
v. Given: W = mg = 350 N, d =
2
To find: Weight (Wd)
 d
Formula: gd = g 1   [½]
 R
Calculation: Since Wd = mgd,
from formula,
 d
Wd = mg 1   [½]
 R
  R 
  2 
 Wd = 350  1    
 R 
 
1
= 350  = 175
2
 Wd = 175 N
The body would weigh 175 N half way down to the centre of the earth. [1]
vi. Solution:
Given: m = 1kg, v = 2 m/s
To find: Total K.E
1  K2 
Formulae: E = MV 2 1  2  [½]
2  R 
Calculation:
2
For solid sphere, K2 =   R2
5
1   2 2  2 1 7
 1   5 R  / R   2  5  MV
2
E= MV 2 [½]
2    
 E = 2.8 J
The total kinetic energy of the solid sphere is 2.8 J. [1]
vii. Solution:
Given: l = 0.9 m, n = 324 Hz
To find: Speed (v)
Formula: v = n [½]
Calculation: In 2nd overtone, 3 loops are formed.
3
 l = 
2
2l
 = [½]
3
From formula,
 2l  2 
v = n   = 324   0.9 
3
   3 
 v = 194.4 m/s
The speed of the transverse wave is 194.4 m/s. [1]
3
Board Answer Paper : March 2017
viii. Solution:
Given: 1 = 25 C, 2 = 15 C,
 d 
 dt  = 0.5 C/min
 1
 d 
To find: Rate of cooling at 2  
 dt 2
d
Formula: = K(  0)
dt
Calculation: Using formula, for 2 = 15 C,
 d / dt 2 2  0
 [½]
 d / dt 1 1  0
 d 
 
 dt  2 15
  [½]
0.5 25
= 0.3 C/min
The rate of cooling when it is 15 C above same surroundings is 0.3 C/min. [1]
Q.3. Attempt any THREE:
i. a. Let,
M = mass of earth
m = mass of satellite
R = radius of earth
vc = critical velocity
b. In one revolution, distance covered by satellite is equal to circumference of its circular
orbit. [½]
c. If T is the time period of satellite, then
Circumference of the orbit
T=
Critical velocity
2r
 T= .…(1) [½]
vc
GM
But, vc = ….(2) [½]
r
d. Substituting equation (2) in (1),
2r
T= [½]
GM
r
r
= 2 r 2 
GM
r3
 T = 2 ….(3) [½]
GM
Thus, period of a satellite revolving around the Earth depends upon mass of the Earth. [½]
ii. Expression for torque acting on a rotating body:
a. Suppose a rigid body consists of n particles of masses m1, m2, m3, ......, mn which are
situated at distances r1, r2, r3, …, rn respectively, from the axis of rotation as shown in
figure.
b. Each particle revolves with angular acceleration .
4
Physics
c. Let F1, F2, F3, …., Fn be the tangential force acting on particles of masses, m1, m2, m3,
…, mn respectively.
d. Linear acceleration of particles of masses m1, m2,…, mn are given by,
a1 = r1, a2 = r2, a3 = r3, …, an = rn

m3 


F3 F2
r3 O r m2
2
mn 
rn r1 F1

Fn m1

[Explanation + Diagram] (½ + ½) [1]


e. Magnitude of force acting on particle of mass m1 is given by,
F1 = m1a1 = m1r1 [ a = r]
Magnitude of torque on particle of mass m1 is given by,
1 = F1 r1 sin  [½]
But,  = 90 [ Radius vector is ar to tangential force]
 1 = F1 r1 sin 90
= F1r1
= m1a1 r1
 1 = m1r12  [ a1 = r1 ]
Similarly,
2 = m 2 r22 
3 = m3 r32 ,
.…
.…

n = mn rn2 
f. Total torque acting on the body,
 = 1 + 2 + 3 + …. + n
  = m1r12  + m 2 r22 + m3 r32  + … + m n rn2 
  = (m1r12 + m2r22 + m3r32+.....+ m n rn2 )
 n 
 
 =  m i ri 2  
 i 1 
n
But, m r
i 1
i i
2
=I

  = I [½]
g. Unit: Nm in SI system. [½]
h. Dimensions: [M1L2T2] [½]
iii. Solution:
Given: E = 2T [½]
To find: Diameter of drop (d)
Formula: E = TA [½]
5
Board Answer Paper : March 2017
Calculation: From formula,
E
A =
T
2T
 A =
T
 A = 2π ....(1) [½]
We know , A = 4r2
Substituting in equation (1),
 2 = 4r2 [½]
 4r2 = 2
2 1
 r2 = =
4 2
r2 = 0.5
 r = 0.5 = 0.71 m
d = 2r
= 2(0.71)
 d = 1.414 m
The diameter of the drop is 1.414 m. [1]
5
iv. Given: v = 36 km h1 = 36  = 10 m/s,  = 10
18
To find: Length of circular track s(l)
v2
Formula: tan  = max [½]
rg
Calculation: From formula,
v2
r  max
g tan 
10 
2

 r = [½]
9.8  tan 10o
= 57.88 m [1]
l = 2πR = 2 3.14  57.88 = 363.7 m
The length of the circular track is 363.7 m. [1]
Q.4. A. Conservation of energy in linear S.H.M.:
i. Suppose a particle of mass m performing linear S.H.M. is at point P which is at a distance x
from the mean position O as shown in figure.
O P
X x X
Negative Mean Positive
extremity position extremity

ii. Kinetic energy of particle at point P is given by,


1
K.E. = m2(A2  x2) [½]
2
iii. Potential energy at point P is given by,
1
P.E. = m2x2 [½]
2
6
Physics
iv. Total energy at point P is given by,
T.E. = K.E. + P.E.
1 1
= m2(A2  x2) + m2x2
2 2
1
= m2(A2  x2 + x2)
2
1
 T.E. = m2A2 ….(1) [½]
2
v. If particle is at mean position:
x=0
1
 K.E. = m2A2
2
1
P.E. = m2(0)2 = 0
2
1
 T.E. = K.E. + P.E. = m2A2 .…(2) [½]
2

vi. If particle is at extreme position:


x=A
K.E. =
1
2
 
m2 A 2  A 2 = 0

1
P.E. = m2A2
2
1
 T.E. = P.E.+ K.E. = m2A2 .…(3) [½]
2
vii. From equations (1), (2) and (3), it is observed that total energy of a particle performing linear
S.H.M. at any point in its path is constant. Hence, total energy of linear S.H.M. remains
conserved. [½]
viii. a. Graph of variation of kinetic energy w. r. t. instantaneous displacement.
K.E

K.Emax

x=A O x=A
x=0
[½]
b. Graph of variation of potential energy w.r.t. instantaneous displacement.

P.E
P.Emax

x=A x=0 x=A


[½]
7
Board Answer Paper : March 2017
B. Solution:
83 83
Given: 1 = m, 2 = m,
170 172
no. of beats = 8/s.
To find: Velocity (v),
Frequency (n1, n2)
Formula: v = n
Calculation: From formula,
v v
 n1 = and n2 = .
1 2
But 1 > 2
 n2 > n1. [½]
 n2  n1 = 8 [½]
 1 1   172 170 
 v    = 8 or v   =8
  2 1   83 83 
8  83
 v= = 332 m/s [1]
2
170
 n1 = 332  = 680 Hz and [½]
83
332  172
n2 = = 688 Hz [½]
83
i. The velocity of sound in air is 332 m/s.
ii. The frequencies of the two notes are 680 Hz and 688 Hz.
OR
A. Formation of stationary waves by analytical method:
i. Consider two identical progressive waves of equal amplitude and frequency travelling
along X axis in opposite direction. They are given by,
2
y1 = A sin (vt  x) along positive X-axis .…(1) [½]

2
y2 = A sin (vt + x) along negative X-axis ….(2) [½]

ii. The resultant displacement ‘y’ is given by the principle of superposition of waves,
y = y 1 + y2 ….(3) [½]
2 2
y = A sin (vt  x) + A sin (vt + x) [½]
 
iii. By using,
C D C D
sin C + sin D = 2sin   cos  ,
 2   2 
we get,
 2  vt  x  vt  x    2  vt  x  vt  x  
y = 2A sin    cos   
   2    2 
 2vt   2 
= 2Asin   cos  ( x)  [½]
     
 2x   v
 y = 2Asin 2πnt cos    n   [cos ( ) = cosθ]
     
 2x 
 y = 2Acos   sin 2πnt
  

8
Physics

 2x 
iv. Let R = 2Acos  
  
 y = Rsin (2πnt) ….(4) [½]
But,  = 2n
 y = R sin t ….(5)
Equation (5) represents the equation of S.H.M. Hence, the resultant wave is a S.H.M.
of amplitude R which varies with x.

A A A A
N N N
N  Node N
A  Antinode
  
2 4 2
Position and displacement of nodes
and antinodes in a stationary wave

[Diagram and labelling] [1]


B. Solution:
Given: L = 4 m, r = 0.5 mm = 0.5  103 m, M = 1kg,
Elastic limit of steel is 2.4  108 N/m2,
Y for steel (Ysteel) = 20  1010 N/m2
To find: Extension in length (l)
Area of cross section (A)
FL
Formulae: i. Y= [½]
Al
F
ii. Elastic limit = [½]
A
Calculation: From formula (i),
FL MgL
l = = 2 [½]
AY r Y
1  9.8  4
 l= [½]
 
2
3.14  0.5  103  20  1010

l = 2.495  104 m
From formula (ii),
Mg
A=
Elatic limit
1 9.8
=
2.4  108
 A = 4.083  108 m2 [1]
4
The extension produced in length is 2.495  10 m and the area of cross section of the
wire should be 4.083  108 m2.
9
Board Answer Paper : March 2017
SECTION – II
Q.5. Select and write the most appropriate answer from the given alternatives for each
sub-question:
i. (C) [1]
ii. (B) [1]
iii. (A) [1]
iv. (B) [1]
v. (C) [1]
vi. (C) [1]
vii. (D) [1]
Q.6. Attempt any SIX:
i.
No. Interference Diffraction
i. Interference is due to superposition of Diffraction is due to waves coming from
waves from different wavefronts. different parts of the same wavefront.
ii. All bright fringes are of equal intensity. Intensity decreases with the order of
bright band.
iii. Minimum intensity may be zero. Minimum intensity is not zero.
iv. Width of the central maximum is same as Width of central maximum is broader
that of other bright fringes i.e fringe width than other maxima and it is double the
is same for all fringes including central fringe width.
maxima.
v. The waves emitted by two coherent The light waves are bend at the corners and
sources travel in straight line. displaced from their straight line path.
vi. The resolving power of an optical The resolving power of an optical
instrument does not depend on the instrument depends on the phenomenon
phenomenon of interference. of diffraction.

[Any two points  1 Mark each]


ii. a. To use a M.C.G as a voltmeter, its resistance should be increased to a desired value and
an arrangement should be provided to measure large potential difference. This is
achieved by connecting a high resistance in series with the M.C.G. [½]

V = (G + Rs)Ig
V = GIg

Ig G Ig Ig
Ig
G

M.C.G Voltmeter [½]

b. Let ‘G’ be the resistance of the galvanometer coil and ‘Ig’ be the maximum current
which can be passed through the galvanometer coil for full-scale deflection.
10
Physics
c. Let ‘V’ be the potential difference to be measured.
Let ‘Rs’ be the resistance connected in series with the galvanometer coil.
d. From Ohm’s law,
V = Ig (G + Rs) [½]
V
G + Rs =
Ig
V
 Rs = G [½]
Ig
Knowing V, Ig and G, value of Rs can be determined.
iii. Advantages of potentiometer over voltmeter:
a. The voltmeter is used to measure terminal P.D of cell while potentiometer is used to
measure small terminal P.D as well as e.m.f of the cell.
b. The accuracy of potentiometer can be easily increased by increasing the length of wire.
c. A small P.D can be measured accurately with the help of potentiometer. The resistance
of voltmeter is high but not infinity to work as an ideal voltmeter.
d. The internal resistance of a cell can be measured with the help of potentiometer.
e. Potential difference across the wire is greater than E1 or E2 or E1 + E2.
[Any two advantages] (1  2) [2]
iv.
Receiving
Antenna

Output
Amplifier IF Stage Detector Amplifier
Received
signal
Block diagram of a receiver
[Diagram and labelling] (1+1) [2]
v. Solution:
Given: N = 100, A = 15 cm2 = 15  104 m2,
B = 0.03 Wb/m2,
C = 15  1010 Nm/degree
To find: Sensitivity (Si)
NAB
Formula: Si = [½]
C
Calculation: From formula,
100  15  104  0.03
Si = [½]
15  1010
 Si = 3  106 div/A
The sensitivity of a moving coil galvanometer is 3  106 div/A. [1]
vi. Given:  = 6t2 + 7t + 1 (in milliweber), t = 2 s
To find: Magnitude of induced e.m.f. (e)
d
Formula: e= (in magnitude) [½]
dt
11
Board Answer Paper : March 2017
Calculation: Using formula,
d
e= (6t2 + 7t + 1)
dt
= 12t + 7 [½]
At t = 2s
|e| = 12  2 + 7
= 31 mV = 31  103 V
The magnitude of induced e.m.f. is 31  103 V. [1]
vii. Solution:
Given: R = 50 , lX = 40 cm = 0.4 m, lR = 60 cm = 0.6 m
To find: Unknown resistance (X)
l
Formula: X=R x [½]
lR
Calculation: From formula,
50  40
X= [½]
60
100
= 
3
= 33.33 Ω
The value of unkown resistance is 33.33 Ω. [1]
10
viii. Given: r2 = 2.14  10 m, n = 2,
v2 = 1.09  106 m/s
To find: Frequency of revolution (2)
Formula: v = r = r(2)
v
 = [½]
2r
Calculation: From formula,
v 1.09  106
2 = 2 = [½]
2r2 2  3.142  2.14  1010
 2 = 8.11  1014 Hz
The frequency of revolution of electron in 2nd Bohr orbit is 8.11  1014 Hz. [1]
Q.7. Attempt any THREE:
i. P-N junction diode as half wave rectifier:
P N Id
_ +
P1 +
S1 D
Vo = Id  RL
d.c out put
a.c input

RL

S2
P2 _
Figure (a): Half wave rectifier

P1P2 : Primary coil of step down transformer


S1S2 : Secondary coil of step down transformer
D : P-N junction diode
RL : Load resistance [1]

12
Physics
Working:
a. A rectifier, which rectifies only one half cycle of each a.c input supply, is called a half
wave rectifier.
b. The circuit diagram is as shown in figure (a).
c. When a.c input is applied to a junction diode, it gets forward biased during one half
cycle and reverse biased during the next opposite half cycle.
d. The a.c supply is fed across the primary coil P1P2 of a step down transformer. The
secondary coil S1S2 of the transformer is connected to the junction diode D and a load
resistance RL. The output d.c. voltage is obtained across the load resistance RL. The
output d.c. voltage is given by Vo = Id  RL.
e. Suppose that during the first half of the input a.c voltage terminal S1 becomes positive
w.r.t. S2. Then, P region of diode D becomes positive w.r.t. N region. This makes the
junction diode forward biased, thus the conventional current flows in the direction of
the arrow-heads through RL.
f. During the negative half cycle the terminal S2 is at positive potential w.r.t. S1. Thus, P
region of diode D is negative w.r.t. N region. This makes the diode reverse biased.
Thus diode does not allow current through it and no current will flow through RL.
g. During the next half cycle, output is again obtained as the junction diode gets forward
biased. Thus, a half wave rectifier gives pulsating unidirectional and intermittent d.c
output voltage V across the load resistance RL. [1½]
h. Graph of a.c input and d.c output voltage with respect to time is shown in figure below.

+V
a.c input
voltage +ve + ve + ve
o time
ve ve
V
+V
d.c output +ve +ve +ve
voltage o time
V
Input and out put waveforms
for half wave rectifier
[½]
ii. Derivation of induced e.m.f due to self induction:
L

+  [½]
E K

a. Consider a coil connected with battery E, plug key K and inductor L carrying current of
magnitude I as shown in figure.
b. Since magnetic flux linked with the coil is directly proportional to the current.
 I
  = LI .…(1)
where, L = constant called coefficient of self induction or self inductance of the coil,
which depends upon the material of the core, number of turns, shape and area of the
coil.
13
Board Answer Paper : March 2017
c. Induced e.m.f in the coil is given by,
d
e=
dt
dI
e=L ….(2)
dt
ve sign in equation (ii) shows that self induced e.m.f opposes the rate of change of
current.
dI dI
 |e| =  L =L
dt dt
dI
 Magnitude of self induced e.m.f is given by, |e| = L
dt
This is required induced e.m.f. [1]
Derivation of induced e.m.f. due to mutual induction:

+
E
 P S G
K
[½]

a. Consider primary coil P and secondary coil S fitted with galvanometer G are placed
very close to each other as shown in figure. The coil P is connected in series with the
source of e.m.f (battery) and key K.
b. When tap key K is pressed current IP passes through the coil P.
Magnetic flux S linked with secondary coil S at any instant is directly proportional to
current IP through primary coil P at that instant.
 S IP
 S = M IP ….(1)
where M is constant called coefficient of mutual induction or mutual inductance of the
coil.
d S
c. e.m.f induced in S at any instant is given by, eS = 
dt
d
=  (MIP) [From equation (1)]
dt
dI
 eS =  M P
dt
 Magnitude of induced e.m.f is given by,
MdIP MdIP
eS =  =
dt dt
eS
 M [1]
dI P / dt

iii. Solution:
Given: l = 1 cm = 102 m, E = 300 Vm1, k = 8
To find: Energy contained in the cube (U)
U
Formula: u=
V
14
Physics
Calculation: Volume of marble,
V = (l3) = (102)3 = 106 m3
1
Energy density, u = 0 kE2 [½]
2
1
=  8.85  1012 8  (300)2 [½]
2
= 3.185  106 J/m3 [½]
From formula,
U = u  V = 3.185  106  106 [½]
 U = 3.185  1012 J
The energy contained in the cube is 3.185  1012 J. [1]
iv. Given: r = 0.53 Å = 0.53  1010 m, f = 9  109 MHz = 9  1015 Hz
To find: Angular momentum (L0)
M0
Formula: L0 = [½]
gyromagneticratio
Calculation: M = IA [½]
1
Since, I = e = fe
T
From formula,
M = feA = fer2 [½]
= 9  1015  1.6  1019    (0.53 1010)2
= 1.6    0.25  1023
 M = 1.270  1023 Am2
Using formula,
1.270  1023
L0 = [½]
8.8  1010
 L0 = 0.1443  1033 kgm2/s
The oribital angular moementum of the electron is 0.1443  1033 kgm2/s. [1]
Q.8. A. Biprism experiment:
This experiment is used in the laboratory to measure the wavelength of monochromatic light.
Apparatus:
Monochromatic source (sodium lamp), optical bench, lens, micrometer, biprism etc.
Experimental arrangement:

S B
L E
S

A A

i. The apparatus is arranged as shown in the figure.


ii. A long rail called optical bench is fitted with a scale, a slit (S), biprism (B), convex lens
(L) and micrometer eye piece (E).
iii. They are mounted vertically on the optical bench with the help of stands such that each
stand can be adjusted in a vertical, lateral as well as longitudinal direction so that
perfect axial alignment can be achieved.
iv. Each stand can be rotated about vertical axis passing through itself. [½]
15
Board Answer Paper : March 2017
Adjustment:
i. Optical bench is leveled with the help of leveling screws. Stands are leveled at same
horizontal plane.
ii. Slit is made narrow, biprism is kept with its refracting edge parallel to the slit.
Procedure:
i. A narrow vertical slit S is illuminated by a source of monochromatic light (S).
The biprism B is placed close to the slit S.
ii. When light from S falls on the refracting edge of the prism then due to refraction, two
virtual images S1 and S2 of the slit S are formed.
iii. These images are in the plane of the slit, quite close to each other and act as two
coherent sources of light.
iv. The waves from these sources interfere with each other and form interference fringes.
v. The eye piece (E) carrying micrometer is kept at large distance from the biprism. The
interference pattern is observed through (E).

S1 Interfering region
d S E

S2
B
Ray diagram of biprism experiment
S1, S2: Virtual images of the source
B: Biprism,
E: Eye-piece

[Ray diagram  1 Mark]


Determination of X:
i. To measure the band width, a vertical cross wire of the micrometer is adjusted on one
bright band and the micrometer reading x1 is noted.
ii. Similarly the cross wire is adjusted on 2nd, 3rd and 4th bright bands and the
corresponding micrometer readings x2, x3, x4, are noted.
iii. The differences (x2  x1), (x3  x2), (x4  x3), etc, give the band widths. The mean of
these band widths is the width X of fringe. [½]

Determination of the distance between source and eyepiece (D):


It can be read directly from scale attached to the optical bench. [½]

Determination of d:
i. Conjugate foci method is used in the determination of d. A convex lens (L) of suitable
focal length is fixed between the biprism and eyepiece on the optical bench.
ii. The convex lens is moved, towards the slit till the magnified images of S1 and S2 are
seen. The distance between them is measured say d1.
iii. Now, move the convex lens away from the slit till the diminished images of S1 and S2
are seen. The distance between them is measured, say d2.
iv. The distance between S1 and S2 is given by the formula, d = d1d 2 [1]

16
Physics
Determination of :
D
Since band width is given by X =
d
Xd
  =
D
Substituting value of d,
X d1d 2
= [½]
D
Putting value of X, d1, d2 and D wavelength can be measured exactly.
B. Solution:

AB = width of incident
B wavefront
Rarer i
medium CD = width of refracted
i C wavefront
A r
Denser r
medium r
D
N1 N2

Given: i = 65, CD = 2AB


To find: Refractive index ()
cosi AB
Formulae: i.  [1]
cos r CD
sin i
ii. = [½]
sin r
Calculation: From formula (i),
cos 65 AB

cos r 2AB
0.4226 1

cos r 2
cos r = 0.845
 r = 3216 [1]
From formula (ii),
sin 65
=
sin (3216)
  = 1.697
The refractive index for the denser medium is 1.697. [½]

17
Board Answer Paper : March 2017
OR
A.

E = 0 n=
n=6
E6 =  0.38
E5 = 0.54 Pfund series n = 5
n=4
E4 = 0.85 Brackett series
E3 = 1.51 n=3
Paschen series

E2 = 3.4 n=2
Balmer series
E(in eV)

E1 = 13.6 n=1
Lyman series Quantum number (n) 

[Diagram and labelling] (1+1) [2]


Paschen series:
i. The spectral lines of this series correspond to the transition of an electron from some
higher energy state to 3rd orbit.
ii. For paschen series, p = 3 and n = 4, 5,...
The wave numbers and the wavelengths of the spectral lines constituting the Paschen
series are given by,
 = 1=R 1  1 
 2 2 
 3 n 
iii. Paschen series lies in the infrared region of the spectrum which is invisible and
contains infinite number of lines.
iv. Wavelengths for n = 4 and 5 are 18750 Å and 12820 Å respectively. [1]
Brackett series:
i. The spectral lines of this series corresponds to the transition of an electron from a higher
energy state to the 4th orbit.
ii. For this series, p = 4 and n = 5, 6, 7,...
The wave numbers and the wavelengths of the spectral lines constituting the Brackett
series are given by,
1  1 1 
 = =R  2 2
  4 n 
iii. This series lies in the near infrared region of the spectrum and contains infinite number of
lines. Wavelengths for n = 5 and 6, are 40518 Å and 26253 Å respectively. [1]
18
Physics
B. Solution:
Given: (W0)P = 2.25 eV = 2.25  1.6  1019 J = 3.6  1019 J,
(W0)C = 2.14 eV = 2.14  1.6  1019 J = 3.424  1019 J,
 = 5180 Å = 5.18  107 m
To find: Will the photoelectric effect occur for either of these elements with  = 5180 Å
Formula: W0 = h0 [½]
Calculation: From formula,
(W0)P = h(0)P
(W0 )P 3.6  1019
 (0)P = =
h 6.63  1034
 (0)P = 5.430  1014 Hz [1]
Similarly,
 W0 C 3.424  1019
(0)C = =
h 6.63  1034
 (0)C = 5.164  1014 Hz [1]
The corresponding frequency is given by,
c 3  108 3
2 = = 7
=  1015
 5.18  10 5.18
 2 = 5.792  1014 Hz
For potassium,
5.792  1014 Hz > 5.430  1014 Hz
i.e., 2 > (0)P
 Photoelectric emission will take place when light of wavelength  is incident on it.
For caesium,
5. 792  1014 Hz > 5.16  1014 Hz
i.e., 2 > (0)C
 Photoelectric emission will take place when light of wavelength  is incident on it.
 For = 5180Å wavelength, both potassium and caesium will exhibit
photoelectric emission. [½]

19
Physics

BOARD ANSWER PAPER : JULY 2017


PHYSICS
SECTION – I
Q.1. Attempt any SIX :
i. Solution:
Given: R = 6.4  106 m,  = 7.26  10–5 rad/s
To find: Change in weight (W)
Formulae: i. g = gp  geq = R2 [½]
ii. W = mg [½]
Calculation: From formula (i) and (ii),
W = m(R2)
= 1  6.4  106  (7.26  10–5)2
= 3373  10–5 N
Change in weight of the mass is 3373  10–5 N. [1]
ii. Solution:
Given: m = 0.3 kg, r = l = 0.5 m, v = 2 m/s
To find: Tension (T)
mv 2
Formulae: Tmax = + mg cos [½]
r
Calculation: From formula ,
0.3  4
= + 0.3  9.8  cos 60 [½]
0.5
= 3.87 N
The tension in the string is 3.87 N. [1]
iii. Solution:
Given: r1 : r2 = 4 : 3
To find: W1 : W2
Formulae: W = 2TdA [½]
Calculation: From formula,
Work done to blow both bubbles,
W1 = 2T  4r12  ; and W2 = 2T  4r22  [½]
2
W1  r1 
   [½]
W2  r2 
2
4
= 
3
= 16 : 9
The ratio of work done to blow the bubbles is 16 : 9. [½]
iv. Solution:
3
Kinetic energy of gas = RT
2
 K.E.  T
[½]
(K.E.) 2 T2
 =
(K.E.)1 T1

1
Board Answer Paper : July 2017
1 T
 = 2 [½]
2 273
 T2 = 136.5 K
At 136.5 K, the average kinetic energy of the gas will be exactly half of its value at
N.T.P. [1]
v. a. Surface tension is defined as the force per unit length acting at right angles to an
imaginary line drawn on the free surface of liquid. [1]
b. The extra energy that a liquid surface holds under isothermal condition is called surface
energy. [1]
vi. Variation of g due to altitude:
a. Let,
M = mass of the earth
R = radius of the earth
h = height at which acceleration due to gravity is to be found.
g = acceleration due to gravity at the surface of the earth.
gh = acceleration due to gravity at height h.
b. On the surface of the earth,
GM
g = 2 ….(1) [½]
R
At height ‘h’ above the earth surface,
GM
gh = ….(2) [½]
(R  h) 2
c. Dividing equation (2) by (1),
g h GM / (R  h) 2
=
g GM / R 2
GM R2
= 
(R  h)2 GM
gh R2
 = ….(3) [½]
g (R  h)2
gh R2
 = 2
g  h
R 1  
2

 R
gh 1
 = 2
g  h
1 
 R 
2
gh  h 
 = 1 ….(4)
g  R 
d. Expanding equation (4) by using binomial expansion and neglecting higher power of
h
gives,
R
g h  2h 
= 1   ( h << R) [½]
g  R 
 2h 
 gh = g  1   ....(5)
 R 
Above equation represents acceleration due to gravity at altitude h.
2
Physics
vii. Physical significance of K:
a. Radius of gyration is a measure of distribution of mass about the given axis of rotation.
b. If the particles of the body are distributed close to the axis of rotation, the radius of
gyration is less. [1]
c. If the particles are distributed away from the axis of rotation, the radius of gyration is
more.
d. The knowledge of mass and radius of gyration of the body about a given axis of
rotation gives the value of its moment of inertia about the same axis, even if we do not
know the actual shape of the body. [1]
viii.
N N cos 


N sin 
G C
F cos  

F h
A  F sin 
B

W = mg
AC : inclined road surface
AB : horizontal surface
BC : height of road surface
G : centre of gravity of vehicle
W : (mg) weight of vehicle
N : normal reaction exerted on vehicle
 : angle of banking

(Diagram and labelling) (1 + 1) [2]


Q.2. Attempt any THREE:
i. Theoretical proof:
a. Consider an ordinary body O and perfectly black body B of same dimension suspended
in a uniform temperature enclosure as shown in the figure.
b. At thermal equilibrium, both the bodies will have same temperature as that of the
enclosure.
c. Let, E = emissive power of ordinary body O
Eb = emissive power of perfectly black body B
a = coefficient of absorption of O
e = emissivity of O
Q = radiant energy incident per unit time per unit area on each body [½]

Uniform
temperature
enclosure
O B
[½]
d. Quantity of heat absorbed per unit area per unit time by body O = aQ.
Quantity of heat energy emitted per unit area per unit time by body O = E.
Since there is no change in temperature
 E = aQ
3
Board Answer Paper : July 2017
E
 Q= .…(1) [½]
a
e. Quantity of heat absorbed per unit area per unit time by perfectly black body,B = Q
The radiant heat energy emitted per unit time per unit area by perfectly black body,
B = Eb
Since there is no change in temperature.
 Eb = Q .…(2) [½]
f. From equations (1) and (2),
E E
= Eb  =a
a Eb
E
But, =e [½]
Eb
 a=e [½]
ii. Expression for strain energy:
a. Let,
L = length of wire
A = area of cross section of wire
r = radius of cross section of wire L
l = elongation of the wire by applying load.
b. If the wire is perfectly elastic then,
Young’s modulus, 0 A
F/A l
Y=
l/L F
F L
= 
A l
YAl
 F= ….(1)
L
c. Let ‘f’ be the restoring force and ‘x’ be its corresponding extension at certain instant
during the process of extension.
YAx
 f= ….(2)
L
d. Let ‘dW’ be the work done for the further small extension ‘dx’.
 dW = fdx [½]
YAx
 dW = dx ….(3) [½]
L
e. The total amount of work done in stretching the wire from 0 to l can be found out by
integrating equation (3).
l l l
YAx YA
W= 
0
dW = 
0
L
dx =
L  xdx
0
[½]

l
YA  x 2 
 W=   [½]
L  2 0
YAl 2
 W = [½]
2L
YAl l
 W = .
L 2
4
Physics
YAl
But, =F
L
1
 W= Fl .…(4) [½]
2
Equation (4) represents the work done by stretching a wire.
iii. Solution:
Given: N = 12, nL = 2 nF , n5 = 90 Hz
To find: Number of beats (x),
Frequency of last fork (nL)
Formula: nL = nF + (N  1) x [½]
Calculation: From formula,
nL = nF + (5  1) x
 nF + 4x = 90 ….(1)
nL = nF + (12  1) x [½]
 nL = nF + 11x ….(2)
nL = 2nF
nF = 11x ….From (2) [½]
Substituting in equation (1),
15x = 90 or x = 6 beat/s [½]
 nF = 11  6 = 66 Hz [½]
 nL = 2  66 = 132 Hz [½]
The frequency of the first and last tuning fork is 66 Hz and 132 Hz respectively.
iv. Solution:
Given: R = 0.2m,  = 8000 Kg/m3
To find: Moment of inertia (I)
Formulae: i. I0 = Ic + MR2 [½]
7
 I0 = MR2
5
ii. Mass (M) = volume  density
Calculation: From formula (ii),
4 
M = V =  R 3   [½]
3 
From formula (i),
7 4 
I =  R 3  R2 [½]
5 3 
28 5
= R 
15
28
=  3.14  (2  101 )5  8000 [½]
15
 I = 15.02 kg m2
M.I. of the uniform solid sphere about a tangent to its surface is 15.02 kg m2. [1]
Q.3. A. Linear S.H.M is defined as the linear periodic motion of a body, in which the restoring force
(or acceleration) is always directed towards the mean position and its magnitude is directly
proportional to the displacement from the mean position. [1]
Displacement-time graph:
i. At extreme position,  = /2
 Displacement, x = A cos t
5
Board Answer Paper : July 2017
Velocity time graph:
i. At extreme position,  = /2
 Velocity of a particle is v =  A sin t [½]
ii. Table:
2
Substituting  = in above equation,
T
Time Phase Velocity
(t) (t) (v)
0 0 0
T/4 /2  A
[½]
T/2  0
3T/4 3/2 A
T 2 0
iii. Graph:

+A
Velocity

0 T T 3T 5T 3T
t
T
4 2 4 4 2
A

[Explanation and graphical variation of velocity and time.] [1]


Acceleration-time graph:
i. At extreme position,  = /2
 Acceleration of a particle is,
a = A2 cos t [½]
ii. Table:
2
Substituting  = in above equation,
T
Time (t) Phase Acceleration
(t) (a)
0 0  A2
T/4 /2 0 [½]
T/2  A2
3T/4 3/2 0
T 2  A2
iii. Graph:
Acceleration

+A2
0 T T 3T 5T 3T
t
T
4 2 4 4 2
A2

Conclusions:
i. Displacement, velocity and acceleration of S.H.M. are periodic functions of time.
ii. The displacement and acceleration curves are sine curves whereas velocity curve is
cosine curve ( = 0).
6
Physics
iii. The phase difference between displacement and acceleration is of  radian.
iv. The phase difference between displacement and velocity and velocity and acceleration
is of /2 radian.
v. The displacement and acceleration is maximum at extreme position whereas velocity is
minimum at the same position.
vi. All curves repeat same path after phase of 2 radian.
[Explanation and graphical variation of velocity and time.] [1]
B. Solution:
Given: L = 1.005 m, g = 9.8 m/s2
To find: Loss in period (T)
L
Formula: T = 2 [½]
g
Calculation: From formula,
1.005
T = 2  3.14 
9.8
1.005
= 6.28
9.8
= 2.012 s
The period of a seconds pendulum is 2 second.
Hence, the given pendulum clock will lose 0.012s in 2.012s (during summer). [½]
 Loss in period per day
24  3600  0.012
T =
2.012
 T = 515.3 s
The clock will gain or lose 515.3 s in one day. [1]
OR

A. i. First mode or fundamental mode:


In this mode of vibration, there is one node at the centre of the pipe and two antinodes,
one at each open end as shown in figure (a).
Let,
v = wave velocity in air A
n = fundamental frequency
 = wavelength L N 
2
L = length of air column
v = n ….(1) A

Also L =
2 Figure (a)
  = 2L ….(2) [Diagram  ½]
From equation (1) and (2),
v = n 2L
v
 n= ….(3) [½]
2L
Equation (3) represents fundamental frequency or lowest frequency of vibration.
7
Board Answer Paper : July 2017
ii. Second mode or first overtone:
In this mode of vibration, there are two nodes and three
antinodes as shown in figure (b). A
Let,
v = wave velocity in air [As the medium is same, wave N
velocity remains same] 1
L A
n1 = next higher frequency
1 = corresponding wavelength
N
L = length of tube
Velocity of wave is given by, A
v = n11 .…(4)
Figure (b)
Also, L = 1 .…(5)
From equation (4) and (5), [Diagram  ½]
v = n1 L
v
 n1 =
L
v
 n1 = 2  ….(6)
2L
From equation (3) and (6)
 n1 = 2n .…(7) [½]
Thus, frequency of first overtone (second harmonic) is twice the fundamental frequency.
iii. Third mode or second overtone:
In this mode of vibration, there are 3 nodes and 4 antinodes as shown in figure (C).
Let,
v = wave velocity in air (As the medium is same, wave velocity remains same)
n2 = next higher frequency
2 = corresponding wavelength
L = length of tube A
Velocity of wave is given by,
N
v = n22 .…(8)
3 A
Also L = 2
2 L N 3 2
2L A 2
2 = .…(9)
3 N
From equation (8) and (9),
A
2L
v = n2
3 Figure (c)
3v
 n2 = ….(10)
2L [Diagram  ½]
From equation (3) and (10),
n2 = 3n [½]
Thus, frequency of second overtone is thrice the fundamental frequency.
Causes:
End correction arises because air particles in the plane of the open end of tube are not free to
move in all directions, hence reflection takes place at the plane, small distance outside the tube. [1]
End correction in open pipe:
i. Let,
l1 and l2 = Vibrating lengths of pipe
n1 and n2 = Resonating frequency
v = Velocity of sound
e = End correction
8
Physics
ii. For the first resonance,
v
n1 =
2(l1 + 2e)
 v = 2n1 (l 1 + 2e) .…(1)
iii. For the second resonance,
v
n2 =
2(l2 + 2e)
 v = 2n2 (l 2 + 2e) .…(2)
iv. From (1) and (2),
n1 (l1 + 2e) = n2 (l 2 + 2e)
 n1 l 1 + 2e n1 = n2 l 2 + 2e n2
 2e n1  2e n2 = n2 l 2  n1 l 1
 2e (n1  n2) = n2l2  n1l1
n l  n1l1 n l  n 2 l2
 e= 22 = 11 [1]
2  n1  n 2  2(n 2  n1 )

B. Solution:
Given: l = 1m, M =10g = 10103kg, n= 50 Hz
To find: Tension (T)
1 T
Formula: n= [½]
2l m
Calculation: Linear density of wire,
M 10  103
m= = = 10  103 kg/m
l 1
From formula,
1 T
50 = [½]
2  1 10  103
T
 100 =
10  103
T
 (100)2 =
10  103
 T = 104  10  103
 T = 100 N
The tension to be applied to the wire is 100 N. [1]
Q.4. Select and write the most appropriate answer from the given alternatives for each sub-
question:
i. (D) [1]
ii. (C) [1]
iii. (C) [1]
iv. (A) [1]
v. (A) [1]
vi. (D) [1]
vii. (D) [1]

9
Board Answer Paper : July 2017
SECTION – II
Q.5. Attempt any SIX:
i. When an -particle is emitted by an atom, its atomic number decreases by 2 and mass
number decreases by 4. [½]
When 90Th232 disintegrates to 82Pb200, 8  particles are emitted. [½]
When -particle is radiated, the atomic number increases by 1 and mass number does not
change. [½]
When 90Th232 disintegrates to 82Pb200, 8  particles are emitted. [½]
ii. Solution:
Given: W0 = 3eV = 3  1.6  1019
= 4.8  1019 J,
h = 6.63  1034 J – s,
e = 1.6  1019 C,
c = 3  108 m/s,
To find: Threshold wavelength (0),
hc
Formula: 0 = [½]
W0
Calculation: From formula,
hc 6.63 1034  3108
(0) = = [½]
W0 4.8  1019
 0 = 4.14  10–7 m
Threshold wavelength of the metal is 4.14  10–7 m. [1]
iii. Solution:
Given: C1 = 8 F, C2 = 8 F, C3 = 4 F, V = 120 volt
To find: Charge on the capacitor C3
1 1 1 1
Formulae: i.    [½]
CS C1 C 2 C3
Q
ii. C=
V
Calculation: Using formula (i),
1 1 1 1
=  
CS C1 C 2 C3
1 1 1
= + +
8 8 4
 CS = 2 F = 2  10–6 F [½]
In series combination,
Q1 = Q2 = Q3 [½]
Using formula (ii),
Q = CS V
 Q = 2  10–6  120 = 240  10–6 C
The charge on the capacitor C3 is 240  10–6 C. [½]
iv. Solution:
Given: E = 6.63 J,  = 1014 Hz, h = 6.63  10–34 Js.
To find: Number of photons (n)
E
Formulae: n= [½]
h
10
Physics
Calculation: Using formula,
6.63
n= [½]
6.63  1034  1014
 n = 1020
The number of photons emitted in the radiation are 1020. [1]

v.

No. Diamagnetic substance Paramagnetic substance


a. In an external magnetic field, a substance In an external magnetic field, a
gets weakly magnetised in the direction paramagnetic substance gets weakly
opposite to that of the field. magnetised in the same direction as that of
the field.
b. When placed in a non-uniform magnetic When placed in a non-uniform magnetic
field, it tends to move from the stronger to field, it tends to move from the weaker to the
the weaker part of the field. stronger part of the field.
c. It is weakly repelled by a magnet. It is weakly attracted by a magnet.
d. Magnetic moment of every atom of a Every atom of a paramagnetic substance is a
diamagnetic substance is zero. magnetic dipole having a certain resultant
magnetic moment.
e. When a rod of diamagnetic substance is When a rod of paramagnetic substance is
suspended in a uniform magnetic field, it suspended in a uniform magnetic field, it
comes to rest with its length perpendicular to comes to rest with its length parallel to the
the direction of the field. direction of the field.
f. There is no effect of temperature on There is effect of temperature on
diamagnetic substance. paramagnetic substance.

[Any two points of difference] (1  2) [2]


vi.
Exosphere (> 500 km)

Appleton layer
Ionosphere

Kennelly heavyside layer

400 km
Thermosphere Edge of
140 km
atmosphere
Mesosphere
80 km
Ozone layer
Stratosphere 50 km

Troposphere 12 km

Earth’s surface

[Diagram + labelling] (1  2) [2]

11
Board Answer Paper : July 2017
vii. Huygens’ construction of plane wavefront:

ct
P P1 N1

ct
Q Q1 N2

ct
R R1 N3

PQR: Plane wavefront at any instant,


P1Q1R1 : Plane wavefront after time ‘t’,
PP1N1, QQ1N2, RR1N3 : wave normals at PQR
[1]
a. A plane wavefront is formed when point of observation is very far away from the
primary source.
b. Let PQR represent a plane wavefront at any instant. According to Huygens’ principle,
all the points on this wavefront will act as secondary sources of light sending out
secondary wavelets in the forward direction.
c. Draw hemispheres with P, Q, R…. as centres and ‘ct’ as radius. The surface tangential
to all such hemispheres is P1Q1R1…. at instant ‘t’. It is a new wavefront at time ‘t’.
d. The plane wavefronts is propagated as plane waves in homogeneous isotropic medium.
They are parallel to each other.
e. PP1N1, QQ1N2, RR1N3 are the wave normals at P, Q, R respectively. These wave
normals show the direction of propagation of plane wavefront.
f. The new wavefront P1Q1R1 is parallel to primary wavefront PQR. [1]
viii. Expression for electric intensity due to uniformly charged infinite plane sheet:
a. Consider an infinite thin plane sheet of positive charge having a uniform surface charge
density  on both sides of the sheet.
b. By symmetry, it follows that the electric field is perpendicular to the plane sheet of
charge and is directed in outward direction.
c. Electric field intensity has same magnitude at a given distance on either sides of the sheet.

Gaussian
surface

P
ds
E1 = E E1 = E

PS [½]

d. To find electric field intensity at a point P due to uniformly charged infinite thin plane
sheet, construct an imaginary cylinder around P with its axis perpendicular to plane
sheet carrying charge with ends having cross sectional area ds.
12
Physics
e. The plane sheet passes through the middle of cylinder’s length so that the ends of
cylinder are equidistant from the plane sheet carrying charge.

f. Electric field intensity, E is perpendicular to the ends of cylinder, hence the electric flux
through each end is Eds.

g. Since E is perpendicular to plane sheet, it is parallel to the curved surface of Gaussian
cylinder. Hence, electric flux does not pass through the curved surface of Gaussian
cylinder. [½]
h. Now, Total Normal Electric Induction over Gaussian surface =  E (2ds)
where, ds is surface area of end faces of the cylinder.
Algebraic sum of charges enclosed by Gaussian cylinder = ds
i. According to Gauss’ law,
 E (2ds) = ds [½]

 E= [½]
2
This is the expression for electric field intensity at a point outside uniformly charged
thin plane sheet.
j. Above equation shows that the magnitude of electric field intensity is independent of
the distance of point from plane sheet.
Q.6. Attempt any THREE:
i.
+
mA
IC 
C +
N-P-N
R I +
A B B V
+   VCE VCC

+ + T E
VBB  V
 VBE
IE

N-P-N transistor in CE configuration


[1]
Output characteristics of transistor:

IB = 50 A
IB = 40 A
IC (mA)
IB = 30 A
IB = 20 A
IB = 10 A
IB = 0 A
0 0.3 1 2 3 4 5
VCE (volt)

Transistor output characteristics


[½]

a. To study output characteristics, IB is kept constant, VCE is increased in small steps


and IC is recorded at every step.
b. For the plot IC versus VCE, output characteristics are as shown in figure.
13
Board Answer Paper : July 2017
c. From output characteristics it is clear that:
1. When IB = 0, then IC is nearly zero. Thus the transistor is in cut off region.
2. For VCE < 0.3 V,
IC increases with VCE and can be controlled by VCE.
3. When VCE > VBE (0.7), C-E junction gets reverse biased and IC gets saturated. i.e.,
IC doesn’t depend on VCE but can be controlled by IB. In this case the transistor is in
active region. It has large current gain in this region.
4. When VCE < 0.2 V, both the junctions gets forward biased and IC no longer can be
controlled by IB. In this case relation between IC and IB gets lost.
5. The dynamic output resistance of transistor is given by,
 V 
ro =  CE 
 IC  I B = constant
6. The output resistance in this mode varies from 1 k to 100 k. [1½]
ii. Expression for magnetic induction at a point along the axis of a toroid:
a. A toroid is a long solenoid bent in the shape of a ring.

O P
r

I I [½]
b. The magnetic field around the toroid consists of concentric circular lines of force
around it. Magnetic field is produced, when a steady current ‘I’ flows through toroid.
c. The direction of magnetic field at a point is along the tangent to the circular path at that
point.
d. Let r be the radius of the Amperian loop. This loop is concentric with the axis of toroid.
P is a point on the loop. We have to determine magnetic induction at P. [½]
e. Applying Ampere’s law,
 

 B . dl = 0NI .…(1) [½]


where, N = total number of turns in the toroid.
NI = total current flowing through toroid.
 
Now,  B . dl =  B dl cos….2
 
f. But, B and dl are in same direction
  = 0
 cos  = 1
 Equation (2) can be written as,
 

 B . dl =  Bdl = B  dl .…(3)
g. Also  dl = 2r
From equation (3),
 

 B . dl = B (2r) .…(4) [½]


h. From equation (1) and (4),
B (2r) = 0NI
14
Physics
 0 NI
 B= .…(5) [½]
2r
N
i. If ‘n’ is the number of turns per unit length of toroid then n = ….(6)
2r
j. From equation (5) and (6),
B = 0nI ….(7) [½]
Equation (5) and (7) both represent magnetic induction at a point along the axis of toroid.
iii. Solution:
Given: R1 = 12 , l2 = 120 cm,
R2 = 18 , l2 = 150 cm
To find: Balancing length (l1),
Internal resistance (r)
l l 
Formula: r=R  1 2 [½]
 l2 
Calculation: From first condition,
 l  120 
r = 12  1  .…(1)
 120 
From second condition,
 l  150 
r = 18  1  .…(2)
 150 
From equation (1) and (2),
 l  120   l1  150 
12  1  = 18   [½]
 120   150 
 5(l1  120) = 6(l1  150)
 l1 = 300 cm [1]
Using equation (i) we get
Internal resistance,
 l  120 
r = 12  1 
 120 
 300  120 
= 12  
 120 
 r = 18  [1]
i. The balancing length is 300 cm.
ii. The internal resistance of the cell is 18 .
iv. Solution:
Let, S = shortest wavelength
L = longest wavelength
1  1 1 
= R  2  2  [½]
 p n 
Longest wavelength in Paschen series is obtained when p = 3, n = 4
For longest wavelength,
1 1 1
 =R  2  2 [½]
L 3 4 
1 1 1 
 =R   
L  9 16 
15
Board Answer Paper : July 2017

1 16  9 
 =R  9  16 
L  
1 7R
 =
L 9  16
9  16 144
 L = = [½]
7R 7R
Shortest wavelength in Balmer series is obtained when p = 2, n = 
For shortest wavelength,
1 1 1 
= R  2  2 [½]
S 2  
4
 S = [½]
R
L 144 R 36
 =  =
S 7R 4 7
L
 = 5.131
S
The ratio of longest wavelength in Paschen serires to shortest wavelength in Balmer series is
5.131. [½]
Q.7. A. Principle:
It is based on the principle of mutual induction i.e., whenever the magnetic flux linked with a
coil changes, an e.m.f is induced in the neighbouring coil. [½]
Construction:
i. A transformer consists of two sets of coils P and S insulated from each other. The coil
P is called the primary coil and coil S is called the secondary coil.
ii. The two coils are wound separately on a laminated soft iron core.
iii. The a.c input voltage is applied across the primary and the induced output a.c voltage is
obtained across the secondary, which is used to drive current in the desired circuit.
iv. The two coils are electrically insulated from each other but they are magnetically
linked.
v. To minimise eddy currents, the soft iron core is laminated. [1]
Soft iron core

Input a.c.
voltage  P S
Load

P S

core [½]
Working:
i. When an alternating voltage is applied to the primary coil the current through the coil
goes on changing. Hence, the magnetic flux through the core also changes.
ii. As this changing magnetic flux is linked with both the coils, an e.m.f is induced in each
coil.
iii. The amount of the magnetic flux linked with the coil depends upon the number of turns
of the coil.
16
Physics
iv. Let, ‘’ be the magnetic flux linked per turn with both the coils at certain instant ‘t’.
v. Let ‘NP and ‘NS’ be the number of turns of primary and secondary coil,
NP = magnetic flux linked with the primary coil at certain instant ‘t’
NS = magnetic flux linked with the secondary coil at certain instant ‘t’ [1]
vi. Induced e.m.f produced in the primary and secondary coil is given by,
dP d
eP =  =  NP ….(1)
dt dt
d d
eS =  S =  NS ….(2)
dt dt
vii. Dividing equation (2) by (1),
eS N
 = S ….(3) [1]
eP NP
Equation (3) represents equation of transformer.
N
The ratio S is called turns ratio (transformer ratio) of the transformer.
NP
viii. For an ideal transformer,
Input power = Output power
 ePIP = eSIS
eS I
 = P ….(4)
eP IS
ix. From equation (3) and (4),
eS N I
= S = P [1]
eP NP IS

B. Solution:
Given: Mnet = 2 Am2, l = 10 cm = 10  10–2 m,
area of cross section = 4 cm2 = 4  10–4 m2
To find: magnetisation (MZ)
M
Formula: MZ = net [½]
V
Calculation: Since, V = l  cross sectional area
= (10  10–2  4  10–4)
= 4  10–5 m3
From formula,
2
MZ = [½]
4  105
 MZ = 50000 A/m
The intensity of magnetisation is 50000 A/m. [1]
OR
A. Theory of interference bands:
i. Let S1 and S2 be the two coherent monochromatic sources which are separated by short
distance d. They emit light waves of wavelength .
ii. Let D = horizontal distance between screen and source.
iii. Draw S1M and S2N  AB
OP = perpendicular bisector of slit.
Since S1P = S2P, the path difference between waves reaching P from S1 and S2 is zero,
therefore there is a bright point at P.
17
Board Answer Paper : July 2017
iv. Consider a point Q on the screen which is at a distance x from the central point P on the
screen. Light waves from S1 and S2 reach at Q simultaneously by covering path S1Q
and S2Q, where they superimpose. [½]

A
Q

x
S1 M
d/2
d O P
x d/2
K
S2 N
D
[½]
B

v. In  S1MQ,
(S1Q)2 = (S1M)2 + (MQ)2
2
 d
(S1Q)2 = D2 +  x   ….(1)
 2
vi. In  S2NQ,
(S2Q)2 = (S2N)2 + (NQ)2
2
2  2d
 (S2Q) = D +  x   ….(2)
 2
vii. Subtract equation (1) from (2),
 d  
2
d 
2

(S2Q)2  (S1Q)2 =  D2   x      D2   x   
  2    2 
   
2 2
 d  d
= D2   x    D2   x  
 2  2
2 2
 d  d
= x   x  
 2  2
 d2   d2 
=  x 2   xd    x 2   xd 
 4   4 
d2 d2
= x2   xd  x 2   xd
4 4
(S2Q)2  (S1Q)2 = 2xd
 (S2Q + S1Q) (S2Q  S1Q) = 2xd
2xd
 S2 Q  S1 Q = ….(3)
S2 Q  S1Q
viii. If x << D and d << D then,
S1 Q  S2 Q  D
S2Q + S1Q = 2D
 Equation (3) becomes,
2xd
S2 Q  S1 Q =
2D
18
Physics
xd
 S 2 Q  S1 Q =
D
xd
 x = ….(4) [1]
D
Equation (4) gives the path difference of two interfering light waves.
ix. Point Q will be bright if,

x = n = 2n
2
xd 
 = n = 2n [From equation (4)]
D 2
D
 x=n ….(5) [½]
d
Equation (5) represents distance of nth bright fringe from central bright fringe.
where n = 0, 1, 2,….
x. Point Q will be dark point if,

x = (2n 1)
2
where n = 1, 2, 3,……
xd 
 = (2n 1)
D 2
D
 x = (2n 1) .…(6) [½]
2d
Equation (6) represents distance of nth dark fringe from central maximum.
Expression for the fringe (band) width:
i. The distance between the centre of two consecutive bright or dark fringe is the fringe
(band) width.
ii. Let xn = distance of nth bright fringe and xn+1 = distance of (n + 1)th bright fringe
D
 xn = n
d
D
 xn+1 = (n + 1)
d
iii. If X is the fringe width between two bright fringes then,
X = xn+1  xn
D D
= (n + 1 ) n
d d
D
 X= ….(7) [1]
d
Equation (7) represent fringe (band) width of bright fringes.
iv. Similarly if xn and x(n + 1) represent distance of nth and (n + 1)th dark fringe then,
D
xn = (2n  1)
2d
D
x(n + 1) = [2(n + 1)  1]
2d
 width of dark fringe or dark band is given by,
X = x(n + 1)  xn
D D
= [2 (n + 1)  1]  (2n  1 )
2d 2d
19
Board Answer Paper : July 2017
D
 X = ….(8) [1]
d
Equation (8) represents fringe (band) width of dark fringes.
B. Solution:
3 4
Given: ag = , aw =
2 3
To find: wg

a g
Formula: wg = [½]
a w
Ca C
Calculation: ag = and aw = a
Cg Cw
Cw
 wg = [½]
Cg
From formula,
3/ 2
wg = [½]
4/3
 
w g = 1.12
The refractive index of glass w.r.t. water is 1.12. [½]
Q.8. Select and write the most appropriate answer from the given alternatives for each sub-
question:
i. (C) [1]
ii. (D) [1]
iii. (B) [1]
iv. (D) [1]
v. (B) [1]
vi. (A) [1]
vii. (D) [1]

20
Chemistry

BOARD ANSWER PAPER : MARCH 2016


CHEMISTRY
SECTION – I
Q.1. Answer any SIX of the following:
i. a. The substances which are strongly attracted by the magnetic field and show permanent
magnetism even when the magnetic field is removed are known as ferromagnetic
substances. The property thus exhibited is called ferromagnetism. [1]
b. Ferromagnetic substances contain a large number of unpaired electrons.
The electronic configuration of iron (atomic number 26) is [Ar] 3d6 4s2.
[Ar]      
6
3d 4s2
Since, iron (Fe) contains four unpaired electrons, it is strongly ferromagnetic. [1]
ii. The boiling point of a liquid is defined as the temperature at which the vapour pressure of
the liquid becomes equal to the atmospheric pressure. [1]
The formula to determine molar mass of a solute using freezing point depression method is
K W
M2 = f 2 [1]
Tf W1
Where M2 = Molar mass of solute in kg mol–1
Kf = Cryoscopic in K kg mol–1 constant
Tf = Depression in freezing point in K
W2 = Mass of solute in kg
W1 = Mass of solvent in kg
iii. a. Adiabatic process: The mathematical expression of first law of thermodynamics is
U = q + W.
In adiabatic process, q = 0.
Hence, the mathematical expression of first law of thermodynamics for adiabatic process is,
U = 0 + W
 U = +W
or  U = W (Explanation + Final equation) [1]
b. Isochoric process: The mathematical expression of first law of thermodynamics is
U = q + W (W =  pex V).
In isochoric process, V = 0, W = 0
Hence, the mathematical expression of first law of thermodynamics for isochoric process is,
U = qv + 0
 U = qv (Explanation + Final equation) [1]
iv. Activation energy (Ea) can be calculated graphically (from the collected data) as follows:
 Ea  1
log10 k =     + log10 A ....(Arrhenius equation) (Arrhenius equation) [½]
 2.303R  T
1
Thus, when the graph of log10k is plotted against , a straight line with negative slope is
T
obtained. (Description of graph) [½]
From the slope of the graph, activation energy can be calculated.
Ea
Slope =  (Equation of slope) [½]
2.303R
 Ea =  slope  2.303  R. (Final equation) [½]
1
Board Answer Paper : March 2016
v. a. Iron:
Name of the ore: Haematite [½]
Formula: Fe2O3 [½]
b. Zinc:
Name of the ore: Zinc blende [½]
Formula: ZnS [½]

vi. a. Arsenic forms sodium arsenide (Na3As2) with sodium.


2As + 3Na  Na3As2 (Explanation + Chemical reaction) [1]
Arsenic Sodium Sodium
arsenide

b. Bismuth forms magnesium bismuthide (Mg3Bi2) with magnesium.


2Bi + 3Mg  Mg3Bi2 (Explanation + Chemical reaction) [1]
Bismuth Magnesium Magnesium
bismuthide

vii. a. The enthalpy change that accompanies the conversion of one mole of solid directly into
its vapour at constant temperature and pressure is called its enthalpy of sublimation
(subH). [1]
b. Whether the conversion of solid to vapour takes place directly in one step or in two
steps (i.e., first melting of solid into liquid and then its vaporization), the enthalpy
change is the same because enthalpy is a state function.
Therefore,
ΔsubH = ΔfusH + ΔvapH [1]
Where fusH is enthalpy of fusion and vapH is enthalpy of vaporization.

viii. Ellingham diagram:


The graphical representation showing the variation of Gibbs energy with increase of
temperature for the formation of oxide (oxidation) is known as Ellingham diagram.
OR
The Ellingham diagram is the plot of free energy change G against temperature for the
reaction of a metal and other elements with one mole of gaseous oxygen at 1 atmosphere. [1]

Features of Ellingham diagram:


The Ellingham diagram for oxides shows the following important features:
a. The graph for the formation of a metal oxide is a straight line with an upward slope.
b. There is sudden change in the slopes for some metal oxides, like MgO, ZnO and HgO.
c. For a few metal oxides of mercury and silver, the graph is at the upper part in
Ellingham diagram (Ag2O and HgO).
d. The graph for the formation of CO is straight line with the negative slope. This
lineintersects the lines of many metal oxides.
e. The graph for the formation of CO2 is straight line almost parallel to the temperature
axis.
(Any two features) [½  2] [1]
2
Chemistry
Q.2. Answer any THREE of the following:

i. Given: Density (d) = 10.51 g cm3.


Molar mass of Ag (M) = 108 g mol1
To find: Volume of the unit cell
Atomic mass
Formulae: a. Mass of one atom =
Avogadro number
Mass of unit cell
b. Density =
Volume of unit cell
1 1
 8 +  6 = 4 atoms
Calculation: Unit cell of fcc type contain =
8 2
Atomic mass of Ag
Mass of one atom = [½]
Avogadro number
108 g mol1
= [½]
6.022 1023 mol1
 Mass of fcc unit cell of Ag = Number of atoms in the unit cell  Mass of one Ag atom [½]
108
=4 g [½]
6.022  1023
Massof unit cell
Density =
Volumeof unit cell
Massof unit cell
 Volume of unit cell =
Density
108
4 g
= 6.022  10 23 [½]
10.51 g cm 3
= 6.825  1023 cm3 [½]
z.M
Note: The above problem can alternatively be solved by using the formula: d = .
a 3 .N A

ii. Given: Vapour pressure of pure benzene  p1o  = 640 mm Hg


Mass of solute (W2) = 2.175  103 kg
Mass of solvent (W1) = 39 g = 39  103 kg
Vapour pressure of solution (p) = 600 mm Hg
To find: Molar mass of solute (M2)
p1o  p W M
Formula: o
= 2 1 [½]
p1 W1 M 2
Calculation: Molar mass of solvent (benzene) (M1) = 78  103 kg mol1 [½]
From formula,
640  600 2.175  103 78  103
=  [1]
640 39  103 M2
640  2.175  103  78  103
 M2 = 3
= 69.6  103 kg/mol = 69.6 g/mol [1]
39  10  40
3
Board Answer Paper : March 2016
iii. Given: H for the given reaction = 104 kJ
To find: H(C  Cl)
Formula: ΔH = ΔH (reactant bonds)  ΔH (product bonds)
Calculation: ΔH = ΔH (reactant bonds)  ΔH (product bonds) [½]
= [3ΔH (CH) + ΔH(CCl) + ΔH(ClCl)] [2ΔH(CH)
+ 2ΔH(CCl) + ΔH(HCl)] [1]
= ΔH(CH )  ΔH(CCl) + ΔH(ClCl)  ΔH(HCl)
 104 = 414  ΔH(CCl) + 243  431 [½]
 ΔH(C  Cl) = 414 + 104 + 243  431 = 330 kJ mol1 [1]

iv. Cell constant: The ratio of the distance between the electrodes divided by the area of cross-
section of the electrode is called as cell constant. [1]

Copper Pure and dry H2


wire gas at 1 atm
Glass jacket
Vessel
Mercury
Platinum
wire
Solution of Platinised
H+ ions (1M) platinum foil

Standard Hydrogen Electrode (Diagram-1 mark, Labelling-1 mark) [2]


Q.3. Answer any ONE of the following:
i. a. Nitrogen differs from the other elements of group 15 in the following ways:
1. Nitrogen is a gas at room temperature while the rest of the members of the family
are solids.
2. Nitrogen exists as diatomic molecule (N2), whereas the other elements exist as
tetratomic molecules (As4, Sb4, P4, etc).
3. Nitrogen can form hydrogen bonds in its hydride compounds due to small size
and high electronegativity and rest of the members do not form hydrogen bonds
due to small electronegativities and larger size.
4. Nitrogen forms p-p multiple bonds. Other elements of the group form multiple
bonds through d-p overlapping.
5. Nitrogen shows all oxidation states from –3 to +5. The other group 15 elements
show limited number of oxidation states.
6. The trihalides of nitrogen (except NF3) are unstable. Trihalides of other group 15
elements are stable.
7. The hydride of nitrogen (NH3) is more stable and more basic than the hydrides of
the other members of the family.
8. Nitrogen does not exhibit pentavalency in its compounds while other members
show pentavalency.
9. Nitrogen does not form coordination compounds due to absence of d-orbitals.
Other group 15 elements form coordination compounds.
(Any four points of difference) [½  4] [2]
4
Chemistry
b. Interhalogen compound, ClF3 possesses trigonal bipyramidal or T-shaped structure. In
ClF3, Cl is the central atom and it undergoes sp3d hybridization forming five hybrid
orbitals. (Type of hybridization) [½]
5 0
3s2
3p 3d
Ground state of Cl



3s2 3p4 3d1
First excited state



sp3d Hybridized state


0
sp3d hybrid orbitals 3d
Hybridization of Cl in ClF3
(Hybridized state of Cl) [½]
Three half filled hybrid orbitals overlap with half filled p-orbitals of three F atoms and
form three ClF sigma bonds.
Two lone pairs occupy the equatorial positions to minimise the lone pair-lone pair and
the lone pair-bond pair repulsions which are greater than bond pair-bond pair
repulsions.
The two axial F atoms bend towards equatorial F atom to minimise the lone pair-lone
pair repulsion, giving T-shaped structure.

o
Cl

8729
F F
F
Molecular Shape of ClF3
[Diagram (geometry + bond angle)] [1]

c. Given: Conductivity (k) = 6.23 × 10–5 –1 cm–1


Resistance (R) = 13710 
Distance between electrodes (l) = 0.7 cm
To find: Area of cross-section of the electrode (a)
l
Formula: k × R = [½]
a
Calculation: From formula,
l
a=
kR
0.7
= [½]
6.23  105  13710
= 0.82 cm2 [1]

d. Molality is the best way of expressing concentration of solutions as it is a temperature


independent quantity. This is because molality involves measurement of masses of
solute and solvent, which are independent of temperature. [1]
5
Board Answer Paper : March 2016
ii. a. 1. Neutral oxides: Oxides which are neither acidic nor basic are called as neutral
oxides.
eg. CO, NO and N2O [1]
2. ZnO is an amphoteric oxide (show both acidic and basic characterstics). This
oxide react with acids as well as bases to form salts.
Basic character (Reaction with acid)
ZnO(s) + 2HCl(aq)  ZnCl2(aq) + H2O(l)
(Basic) Zinc chloride [1]
Acidic character (Reaction with base)
ZnO(aq) + 2NaOH  Na2ZnO2(aq) + H2O(l) [1]
(Acidic) (excess)

b. 1. Molar conductivity is defined as the conductance of a volume of solution


containing 1 mole of dissolved electrolyte when placed between two parallel
electrodes 1 cm apart and large enough to contain between them all the solution. [1]
2. Zero order reaction is the reaction whose rate is independent of the reactant
concentration and remains constant throughout the course of the reaction. [1]
c. Given: For a first order reaction, amount of reactant that remains after
45 minutes = 40%
To find: Rate constant (k)
2.303  A 0
Formula: k= log10
t  A t
Calculation: [A]0 = 100, [A]t = 40, t = 45 min [½]
From formula,
2.303  A 0
k = log10 [½]
t  A t
2.303 100
 k = log10 [½]
45 40
 k = 0.02036 min–1 [½]
Q.4. Select and write the most appropriate answer from the given alternatives for each
sub-question:
i. (B) Thiosulphurous acid [1]
ii. (C) non-polar molecular solid [1]
iii. (A) measuring heat capacity of substance at various temperatures [1]
iv. (C) ebullioscopy [1]
v. (D) Hall’s process [1]
vi. (B) 2 amperes [1]
Q = It
Q 840
 I = =
t 7  60
= 2 amperes
vii. (B) 1.1 × 10–4 s–1 [1]
Rate = k[A]
Rate 6.6  105
 k = =
A 0.6
= 1.1 × 10–4 s–1
6
Chemistry
SECTION – II
Q.5. Answer any SIX of the following:
i. a. Electronic configuration of:
Sc3+ : [Ar] 3d0 [½]
Ti3+: [Ar] 3d1 [½]
b. Sc3+ ion has completely empty 3d-orbital, i.e., no unpaired electrons are present. As d-d
transitions are not possible, Sc3+ ion is colourless. [½]
Ti3+ ion has one unpaired electron in 3d-orbital. As d-d transitions are possible, Ti3+ ion
is coloured. [½]
ii. a. Double salts are the molecular or addition compounds that exist only in solid state and
dissociate into ions in aqueous solution or in any other solvent. They lose their identity
in aqueous solution. Fe(NH4)2(SO4)2(H2O)6 or Mohr’s salt is a double salt, which
dissolves in water and gives the characteristics of Fe2+, NH +4 and SO24 ions.
b. Coordination compounds are the molecular or addition compounds that exist in the
solid state as well as when dissolved in water or any other solvent. They do not
completely lose their identity in aqueous solution. K4[Fe(CN)6] is a coordination
compound and when dissolved in water its constituent complex ion [Fe(CN)6]4 does
not dissociate into Fe2+ and CN ions. [1]
iii. Preparation of chlorobenzene from aniline:
+ 
NH2 N2Cl Cl
NaNO 2  HCl

273K
 
Cu 2 Cl2
+ N2
Benzene diazonium Chlorobenzene
Aniline
chloride
(Reaction + names of reactants, products and reagents) [1]
Conversion of chlorobenzene to diphenyl:
Cl

2 + 2Na 
dry ether
 + 2NaCl

Chlorobenzene Diphenyl
(Reaction + names of reactants, products and reagents) [1]
iv. Metamerism (positional isomerism): Ethers having same molecular formula but different
alkyl groups attached on either side of the oxygen atom are called metamers of each other.
This phenomenon is called metamerism (positional isomerism). [1]
eg. Ethers with molecular formula C4H10O have three metamers having different alkyl
groups attached on either side of oxygen atom.
a. CH3  CH2  O  CH2  CH3 Diethyl ether
b. CH3  O  CH2  CH2  CH3 Methyl n-propyl ether
c. CH3  O  CH  CH3 Isopropyl methyl ether

CH3 (Example of any two ethers) [1]


v. Ketones are the first oxidation products of secondary alcohols. The functional group in
ketones is keto or oxo group ( C = O). The keto group is not located at the terminal carbon
of the chain. [1]
Classification of ketones: Depending upon the type of alkyl group (R) attached to the carbonyl
carbon, ketones are classified into simple ketones (symmetrical ketones) and mixed ketones
(unsymmetrical ketones).
7
Board Answer Paper : March 2016
a. Simple ketones (Symmetrical ketones): When both the alkyl groups present in a
ketone are same (identical), then the ketone is called a simple or symmetrical ketone.
O
General formula: R  C  R, where R may be alkyl or aryl group.
eg. CH3  CO  CH3 H5C6  CO  C6H5
Acetone Diphenyl ketone
(Dimethyl ketone) (Explanation + One example) [½]
b. Mixed Ketones (Unsymmetrical ketones): When both the alkyl groups present in a
ketone are different, then the ketone is called mixed or unsymmetrical ketone.
O
General formula: R  C  R, where R and R may be alkyl or aryl group (R  R).
eg. CH3  CO  C2H5 C2H5  CO  C6H5 (Explanation + One example) [½]
Ethyl methyl ketone Ethyl phenyl ketone

vi. a.
H H O O

C2H5  C = N  OH + [O] 


trifluoroperoxyacetic
acid
 C2H5  C = N+  OH 
 C2H5  CH2  N+ = O
Propionaldoxime 1-Nitropropane
(Reaction + names of reactant, product and reagent) [1]

CH3 CH3 O CH3 O
b. CH3  C = N  OH + [O] 
trifluoroperoxyacetic
 CH3  C = N+  OH 
 CH3  CH  N+ = O
acid
Acetoxime 2-Nitropropane

(Reaction + names of reactant, product and reagent) [1]

vii. Antioxidant is a substance which when added to food, retards or prevents oxidative
deterioration of food. [1]
Structure of Butylated hydroxy toluene (BHT):

OH
(CH3)3C C(CH3)3

CH3
Butylated hydroxy toluene (BHT) [1]

viii. Carbohydrates are optically active polyhydroxy aldehydes or polyhydroxy ketones or the
compounds that can be hydrolysed to polyhydroxy aldehydes or polyhydroxy ketones. [1]
Preparation of nylon-6:
O H
C N
O H O H
CH2 CH2
533K
n 
N
  C (CH2)5 N    C (CH2)5 Nn
2
[1]
CH2 CH2 Repeating unit Nylon-6
CH2
-Caprolactum

8
Chemistry
Q.6. Answer any THREE of the following:
i. a. The elements in which the last electron enters into (n2) f-orbital of the atoms are
called f-block elements. [1]
b.

Lanthanoids Actinoids
1. In lanthanoids, last differentiating electron
In actinoids, last differentiating electron
occupies 4f orbital. occupies 5f orbital.
2. They are the elements of first inner They are the elements of second inner
transition series. transition series.
3. They are present in period 6. They are present in period 7.
4. Most of lanthanoids (with exception of Most of actinoids (with exception of
promethium) occur in nature. uranium and thorium) are prepared in
laboratory.
5. Most of lanthanoids (with exception of All the actinoids are radioactive.
promethium) are non-radioactive.
6. Lanthanoids do not form oxo cation. Actinoids form oxo cation such as UO 22  ,
PuO2+, UO+.
7. 4f orbitals in lanthanoids have higher 5f orbitals in actinoids have lower binding
binding energy. energy.
8. Contraction in atomic and ionic radii is Contraction in atomic and ionic radii is
relatively less in lanthanoids. relatively greater in actinoids due to poor
shielding of 5f electrons.
9. Lanthanoids show +2, +3 and +4 oxidation Actinoids show +3, +4, +5, +6, +7
states. oxidation states.
10. Lanthanoids have less tendency to form Actinoids have greater tendency to form
complexes. complexes.
11. Some ions of lanthanoids are coloured. Most of the ions of actinoids are deeply
coloured.
12. Lanthanoid hydroxides are less basic in Actinoid hydroxides are more basic in
nature. nature.
(Any four distinguishing points) [½  4] [2]

ii. a. Optical activity (Chirality): The property of a substance to rotate the plane of a plane
polarised light towards the right (clockwise) or towards the left (anticlockwise) is
called optical activity.
eg. Lactic acid, glucose, cane sugar, 2-chlorobutane, etc., shows optical activity. [1]

b. The molecules or ions which are coordinated to the central atom or ion in the
coordination compound are called ligands or donor groups.
eg. 1. In [Ni(NH3)6]2+ , central metal ion is Ni2+ and ligands are NH3 molecules.
2. In [Cr(H2O)6]3+, the central metal ion is Cr3+ and the ligands are H2O
molecules. [1]
c. 1. Interstitial compounds are those which are formed when small atoms like H, C,
N, B, etc., are trapped inside the crystal lattice of metals.
2. Interstitial compounds have variable composition and are non-stoichiometric in
nature.
eg. TiC, TiH1.73, Mn4N, Fe3H, VH0.56, ZrH1.92, etc. [1]
9
Board Answer Paper : March 2016
iii. Formula: [Pt(NH3)4Cl2]Cl2 [1]
Propene on reaction with hydrogen bromide in the presence of peroxide follows anti-
Markownikoff’s rule and forms n-propyl bromide as a major product.
peroxide
CH3  CH = CH2 + HBr  CH3  CH2  CH2Br
Propene n-Propyl bromide
(1-Bromopropane)
(Explanation + Reaction)
(Major product) [1]
Addition of hydrogen bromide to propene gives isopropyl bromide as a major product
according to Markownikoff’s rule.
Markownikoff's rule
CH3  CH = CH2 + H  Br  CH3  CH  CH3
Propene
Br
Isopropyl bromide
(2-Bromopropane)
(Major product)

(Explanation + Reaction) [1]


iv. Broad spectrum antibiotics:
Antibiotics which are effective against a wide range of gram positive and gram negative
bacteria are known as broad spectrum antibiotics.
eg. Tetracycline, chloramphenicol, ampicillin, amoxicillin, etc. [1]
Preparation of polythene:
CH2
n 
Polymerization
 , pressure,catalyst
  CH2  CH2   CH2  CH2
CH2 n
Ethene
(Monomer) Repeating unit Polythene
(Reaction + names of reactants, products and reagents + Reaction condition) [1]
Preparation of neoprene:
nCH2 = C  CH = CH2  
Polymerization
catalyst  CH2 C = CH  CH2  CH2 C = CH  CH2 

Cl Cl Cl
n
Chloroprene Repeating unit Neoprene
(Reaction + names of reactants, products and reagents + Reaction condition) [1]
Q.7. Answer any ONE of the following:
i. a. Mechanism of esterification reaction:
The mechanism involves two steps:
1. The mineral acid protonates the carbonyl oxygen of the carboxylic acid. This
increases the positive character of carbonyl carbon. The nucleophile (alcohol)
attacks the carbonyl carbon. This is followed by the loss of proton to obtain ester
hydrate.
+ OH OH
O OH 
 +
R 'O  H
H   R  C  OH
 R R  C  OH + ROH2

R  C  OH 
 R  C  OH O
+OH H O  R
R
Tetrahedral intermediate Ester hydrate

(Explanation + chemical reaction for acid catalysed addition) [2]


10
Chemistry
2. In the ester hydrate, one of the OH group is protonated and lost as water. Then
another OH group loses a proton to obtain ester.

OH OH H +O  H O
H 
+  R  C + H3O+

R  C  O  H  R C  O H
 RC + H2O 
OR OR OR OR
(Explanation + chemical reaction for acid catalysed dehydration) [2]
b. Dehydration of alcohols:
1. Primary alcohol is dehydrated by heating with 95% H2SO4 at 443 K.
eg. H H
H H
HCCH 
95% H 2SO4
443K
 C=C + H2O

H OH H H
Ethanol Ethene
(Reaction + names of reactants, products and reagents) [1]
2. Secondary alcohol is dehydrated by heating with 60% H2SO4 at 373 K.
eg. H H H
 

H  C  C  C  H 
60% H 2SO 4
373K
 CH3  CH = CH2 + H2O
Propene
H OH H
Propan-2-ol
(Reaction + names of reactants, products and reagents) [1]
3. Tertiary alcohol is easily dehydrated by heating with 20% H2SO4 at 363 K.
eg.

H CH3 H CH3
  
H  C  C  C  H 
20% H 2SO 4
363K
 CH3  C = CH2 + H2O
2-Methylpropene
H OH H
2-Methylpropan-2-ol
(Reaction + names of reactants, products and reagents) [1]
ii. a. Vitamins: Vitamins are the organic substances that must be supplied to permit
proportionate growth in living beings (humans) or for the maintenance of the structure. [1]
b. Diseases caused due to deficiency of vitamin A:
1. Night blindness
2. Dryness of skin and hair
3. Retardation of growth
(Any two diseases) [½ × 2] [1]
c. Structure of nucleoside and nucleotide:
O
O  O
HO H2C Base O P O H2C Base
Sugar Sugar
H H O H H
H H H H
OH OH OH OH
(I) (II)
Nucleoside Nucleotide [1  2] [2]
11
Board Answer Paper : March 2016
d. Reaction with nitrous acid:
The reaction is used to distinguish between primary, secondary or tertiary nitroalkanes.
With nitrous acid, -hydrogen is replaced by nitroso group.
1. 1-Nitropropane (Primary nitroalkane) reacts with nitrous acid to form blue
coloured nitroso-nitroalkane (aci form) which on dissolution in sodium
hydroxide gives red solution.
H O H
O
+
+
C2H5  C – N + HO  N = O  NaNO2 / HCl
 C2H5  C N + H2O
O O
H NO
1-Nitropropane (nitroform)

OH ONa
C2H5  C = N + NaOH  C2H5  C = N+ + H2O
+

O O
NO NO
(aci form) Red solution
Blue solution
[1]
2. 2-Nitropropane (Secondary nitroalkane) reacts with nitrous acid to form
corresponding blue coloured nitroso-nitroalkane, which is insoluble in sodium
hydroxide as it does not contain replaceable -hydrogen atom.
CH3 O CH3
O
+
H3C  C – N + HO  N = O  NaNO2 / HCl
 H3C  C N+ + H2O
O O [1]
H NO
2-Nitropropane (blue solution)

3. 2-Methyl-2-nitropropane (Tertiary nitroalkane) does not react with nitrous acid


because there is no replaceable -hydrogen atom on the carbon atom carrying the
 NO2 group. [1]
Q.8. Answer any ONE of the following:
i. (C) Swarts reaction [1]
ii. (D) p-Aminophenol [1]
Phenols with electron donating substituents like alkyl (R) and amine ( NH2) groups
are less acidic because these substituents do not delocalize the negative charge.
p-aminophenol (pKa  10.5) is weaker acid than p-cresol (pKa  10.2)
iii. (D) propanoic acid [1]
CH3  CH2  C  N + 2H2O + HCl 

 CH3  CH2  COOH + NH4Cl
Propanenitrile Propanoic acid
iv. (A) Ethylamine [1]
Primary amines undergo this reaction.
v. (D) Uracil [1]
vi. (C) PHBV [1]
Others are homopolymers.
vii. (D) antidepressant [1]

12
Chemistry

BOARD ANSWER PAPER : JULY 2016


CHEMISTRY
SECTION – I
Q.1. Select and write the most appropriate answer from the given alternatives for each
sub-question:
i. (B) Cesium chloride [1]
ii. (B) Second law of thermodynamics [1]

iii. (D) 2NH3(g)  Pt


 N2(g) + 3H2(g) [1]
Rate of reaction is independent of the total concentration of NH3.
iv. (A) 0.242 V [1]
v. (D) Copper ferrocyanide [1]
vi. (B) Siderite [1]
Siderite is FeCO3.
vii. (C) Arsenic (Z = 33) [1]
[Ar]3d104s24p3
Q.2. Answer any SIX of the following:
i. Van Arkel method for refining titanium: The volatile iodide of titanium (formed after
treatment of impure titanium with iodine) is electrically heated using tungsten filament above
1700 K. Pure titanium gets deposited on the filament and as the deposition goes on, the
current is steadily raised to maintain the temperature.
Ti(s) + 2I2(g) 523K
 TiI4(g)
Impure Volatile
iodide of
titanium [1]
TiI4(g) 
1700 K
 Ti(s) + 2I2(g)
Volatile Pure
iodide of
titanium [1]

ii.  cC + dD
For the reaction, aA + bB  [½]
before approaching equilibrium, the reaction quotient is given by,
C  D or Q =
c d
pcC  pdD
Qc = p [½]
 A   B paA  p Bb
a b

Here, the values of concentrations or partial pressures are other than equilibrium values.
At equilibrium, Q = K called equilibrium constant and G = 0, because the reaction mixture
has no tendency to change in either direction. [½]
Therefore, equation G = G + RT ln Q becomes:
0 = G + RT ln K
 G =  RT ln K
 G =  2.303 RT log10K [½]
The above equation gives the relationship between standard Gibbs energy change of the
reaction and its equilibrium constant.
1
Board Answer Paper : July 2016
iii. Face-centred cubic lattice (fcc):
a. In face-centred cubic unit cell, eight constituent particles (spheres) are present at eight
corners of unit cell. Six constituent particles (spheres) are present at centres of six
faces.
1
atom
2
1
atom (Diagram) [1]
8

Actual portion of atoms


belonging to one fcc unit cell
b. A constituent particle present at a corner is shared by eight neighbouring unit cells. Its
contribution to a unit cell is only 1/8.
Thus, the number of atoms present at corners per unit cell
1
= 8 corner atoms  atom per unit cell = 1 [½]
8
c. A constituent particle present at the centre of a face is shared by two neighbouring unit
cells. Its contribution to a unit cell is only 1/2.
The number of atoms present at faces per unit cell
1
= 6 atoms at the faces  atom per unit cell = 3
2
d. The total number of atoms per unit cell = 1 + 3 = 4
Thus, a face-centred cubic unit cell has 4 atoms per unit cell. [½]
iv. Isolation method:
a. In the isolation method, the concentrations of all the reactants except one are made to
remain constant throughout the course of the reaction by taking them in large excess.
The dependence of rate on the concentration of the isolated species is experimentally
determined.
b. The experiment is repeated by isolating one of the reactants each time and determining
the rate law with respect to the isolated species.
c. Consider the reaction,
aA + bB  cC + dD [½]
The rate law has the form,
Rate = k[A]x[B]y [½]
d. In one of the experiment, A is isolated by making the initial concentration of B (i.e.,
[B]0) to remain constant throughout the course of the reaction (by taking it in large
excess).
Hence, the rate law is written as
rate = k[A]x  B0 = k[A]x (where, k = k  B0 )
y y

The dependence of rate on [A] that is ‘x’ is found out. [½]


e. In another experiment, B is isolated by keeping initial concentration of A (i.e., [A]0)
constant throughout the course of the reaction (i.e., by taking it in large excess).
The rate law then becomes, Rate = k [A]0x [B]y = k [B]y (where, k = k [A]0x ).
The dependence of rate on B that is y is found out.
f. Thus, by knowing the values of x and y, the order of the reaction (x + y) and the rate
law can be determined. [½]
2
Chemistry
v. a. Galvanization:
1. The rusting of iron can be prevented by coating the metal surface with another metal
such as zinc, that is oxidized in preference to iron. This process is called galvanization.
2. If the zinc layer is scratched, even then it protects iron from corrosion as long as
Fe and Zn are in contact. If Fe is oxidized to Fe2+, then Zn immediately reduces
Fe2+ back to Fe. [1]
b. Passivation: The surface of metal is treated with strong oxidizing acid such as
concentrated nitric acid and a thin preventive layer of oxide is formed on the surface of
metal. Thus, the surface is made inactive. This is process is called as passivation. [1]
vi. Nernst equation can be given as,
2.303RT [Products]
E = E0  log10 [1]
nF [Reactants]
where,
E0 = Standard potential of electrode or cell,
n = Number of moles of electrons used in reaction,
F = Faraday = 96500 C/mol e, [Products] = Concentration of products,
[Reactants] = Concentration of reactants, T = temperature in K and
R = gas constant = 8.314 J K1 mol1 [1]
vii. a. Dilute H2SO4 reacts with Fe which is below hydrogen in the electrochemical series,
with the evolution of H2.
Fe + H2SO4  FeSO4 + H2(g)
Iron Sulphuric Ferrous Hydrogen [1]
acid sulphate

b. When H2SO4 is added to calcium fluoride, hydrogen fluoride is obtained.


CaF2 + H2SO4  CaSO4 + 2HF [1]
Calcium Sulphuric Calcium Hydrogen
fluoride acid sulphate fluoride

viii. a. Osmosis: The spontaneous and unidirectional flow of solvent molecules through a
semipermeable membrane, into the solution or flow of solvent from a solution of lower
concentration to the solution of higher concentration through a semipermeable
membrane is called osmosis. [1]
b. Freezing point: The freezing point of a substance may be defined as the temperature
at which the vapour pressure of solid is equal to the vapour pressure of liquid. [1]
Q.3. Answer any THREE of the following:
i. Given: Rate constant k1 = 0.58 s1; Rate constant k2 = 0.045 s1
T1 = 313 K; T2 = 293 K
R = 8.314 J K1mol1
To find: Activation energy (Ea)
k2 Ea  T2  T1 
Formula: log10 =   [½]
k1 2.303R  T1T2 
Calculation: From formula,
 0.045 s 1  Ea  293K  313 K 
log10  1 
= 1 1   [½]
 0.58 s  2.303  8.314 J K mol  293 K×313 K 
Ea  20 
 log 0.0776 = 1   [½]
19.147 J mol  293  313 
E a  20 
 1.110 =  
19.147  293  313 

3
Board Answer Paper : July 2016
1.110  19.147  293  313
 Ea = [½]
20
= 97455.34 J mol1
= 97.45 kJ mol1 [1]
ii. Given: Given equations are,
7
C2H6(g) + O2(g)  2CO2(g) + 3H2O(l), ΔH = 1560 kJ…. (1)
2
1
H2(g) + O2(g)  H2O(l), ΔH = 285.8 kJ…. (2)
2
C(graphite) + O2(g)  CO2(g), ΔH = 393.5 kJ…. (3)
To find: Standard enthalpy of the given reaction
Calculation: Reversing equation (1),
7
2CO2(g) + 3H2O(l)  C2H6(g) + O2(g), ΔH = 1560 kJ…. (4) [½]
2
Multiplying equation (2) by 3 and (3) by 2, then adding to equation (4) [½]
7
2CO2(g) + 3H2O(l)  C2H6(g) + O2(g), ΔH = +1560 kJ
2
3
3H2(g) + O2(g)  3H2O(l), ΔH = 857.4 kJ
2
2C(graphite) + 2O2(g)  2CO2(g), ΔH = 787 kJ
2C(graphite) + 3H2(g)  C2H6(g) , [1]
ΔH = 1560 + (857.4) + (787.0) = 84.4 kJ [1]
iii. Given: Mass of sulphur = W2 = 3.795 g = 3.795  103 kg
Mass of solvent = W1 = 100 g = 100  103 kg
Boiling point of solution = Tb = 319.81 K
Boiling point of pure solvent = Tb = 319.45 K
Molal elevation constant = Kb = 2.42 K kg mol1
Atomic mass of sulphur = 32
To find: Molecular formula of sulphur in solution
K ×W2
Formula: Molar mass of solute = M2 = b
Tb  W1
Calculation: For solution of sulphur in CS2 (solvent),
Tb = Tb – Tb = 319.81 – 319.45 = 0.36 K
From formula,
K ×W2
M2 = b [½]
Tb ×W1
2.42  3.795  103
= [½]
0.36  100  103
= 0.2551 kg mol1
= 255.1 g mol1 [1]
Molar mass of sulphur
n = [½]
Atomic mass of sulphur
255.1
 n =
32
= 7.97  8
 Molecular formula of sulphur in CS2 = S8 [½]
4
Chemistry
iv. Ostwald’s process (Large scale preparation): Ostwald’s process is used to prepare nitric
acid on a large scale.
a. This method is based upon catalytic oxidation of NH3 by atmospheric oxygen.
Pt /Rh gaugecatalyst
4NH3(g) + 5O2(g) 
500K,9bar
 4NO(g) + 6H2O(g)
Ammonia (From air) Nitric oxide
(Reaction + names of reactants, products and reagents + Reaction conditions) [1]

b. Nitric oxide reacts with oxygen to form nitrogen dioxide.


2NO(g) + O2(g) 
 2NO2(g)
Nitric oxide Oxygen Nitrogen dioxide
(Reaction + names of reactants and products) [1]

c. Nitrogen dioxide on dissolution in water gives nitric acid.


3NO2(g) + H2O(l)  2HNO3(l) + NO(g)
Nitrogen dioxide Water Nitric acid

The nitric oxide (NO) thus formed is recycled and the aqueous HNO3 is concentrated
by distillation to give 68 % HNO3 by mass. Further, concentration to 98 % can be
achieved by dehydration with concentrated H2SO4.
(Reaction + names of reactants and products) [1]

Q.4. Answer any ONE of the following:


i. a. Third law of thermodynamics: The third law of thermodynamics states that, “The
entropy of a perfectly ordered crystalline substance is zero at absolute zero of
temperature”. [1]

Applications of standard molar entropy:


1. S for a chemical reaction can be calculated by knowing the values of standard
molar entropy of all reactants and products in the reaction. It is obtained by
subtracting total standard molar entropy of all the reactants from those of all the
products. The entropy change for the reaction is thus given by,
ΔS = m S (products)  n S (reactants)
where, m and n are the coefficients of products and reactants respectively in the
balanced equation.
eg. aA + bB  cC + dD
ΔS is given by,
ΔS = ( cSC + dSD )  ( aSA + bSB )
2. The standard molar entropy is useful in comparing the entropies of different
substances under the same conditions of temperature and pressure.
The S values increase with increasing complexity of molecules.
eg. CH3COOH is more complex than CH3OH. Hence S of CH3COOH
(160 J K1 mol1) is larger than that of CH3OH (127 J K1 mol1)
The S values of the heavier substance are larger than those of the lighter
substance.
eg. For I(g), S = 180.7 J K1 mol1 whereas for F(g), S = 158.6 J K1 mol1.
(Two applications) [½  2] [1]
5
Board Answer Paper : July 2016
b. Battery
+ 

+ 
Impure copper
Pure copper
(blister copper)
Cu2+ as cathode
as anode Cu2+
Cu2+
Cu2+
15% CuSO4 + 5% H2SO4
Anode mud solution (electrolyte)

Electrolytic refining of blister copper

(Diagram-1 mark, Labelling-1 mark) [2]


c. Given: Edge length (a) = 412.1 pm = 4.12  108 cm
Molar mass = 133 + 35.5 = 168.5 g mol1
To find: Density (d)
Molar mass
Formulae: 1. Mass of one molecule =
Avogadro number
3
2. Volume of unit cell = a
Mass of unit cell
3. Density =
Volume of unit cell
Calculation: In the bcc type unit cell of CsCl, there is one Cs+ ion at the body centre
position and 8 Cl ions are at the 8 corners.
 Number of Cs+ in unit cell = 1
1
Number of Cl in unit cell =  8 = 1
8
Hence, the unit cell contains one CsCl molecule.
Molar mass 168.5 g mol 1
Mass of one CsCl molecule = =
Avogadro number 6.0231023 mol1
= 2.798  1022 g
 Mass of unit cell = 1  2.798  1022 g = 2.798  1022 g [½]
Volume of unit cell = a3 = (4.12  10–8 cm)3 = 6.993  1023 cm3 [½]
Mass of unit cell 2.798  1022 g
 Density = = = 4.0 g cm–3 [1]
Volume of unit cell 6.993  1023 cm3

d. The radius ratio (r+/r–) defines the coordination number of the cation.
rCs+ = 1.69 Å, rCl = 1.81 Å
rCs 1.69
 = = 0.9337 [½]
rCl 1.81
Since, radius ratio is greater than 0.732, the coordination number of cation (Cs+) is 8. [½]
ii. a. When thin copper leaves are thrown in a jar of chlorine they catch fire and form cupric
chloride. [1]
Cu + Cl2  CuCl2
Copper Chlorine Cupric chloride [1]
6
Chemistry
b. 1. Due to greater electronegativity of O than S, H2O undergoes extensive
intermolecular H-bonding. As a result, H2O exists as an associated molecule in
which each O is tetrahedrally surrounded by four water molecules. Large amount
of energy is required to break these H-bonds. Therefore, H2O is a liquid at room
temperature. [1]
2. In contrast, H2S does not undergo H-bonding. It exists as discrete molecules
which are held together by weak van der Waals forces of attraction. To break
these forces of attraction, only a small amount of energy is required. Therefore,
H2S is a gas at room temperature. [1]

c. Given: C = 0.02 M , k = 2.428  103  1 cm1


To find: Molar conductivity
1000 k
Formula: Molar conductivity = [½]
C
1000 k 1000  2.428  10 3
Calculation: Molar conductivity = = [½]
C 0.02
= 121.4  cm mol1
1 2
[1]
d. Henry’s law relates solubility of a gas with external pressure. The law states that, “the
solubility of a gas in liquid at constant temperature is proportional to the pressure of
the gas above the solution”. [1]

SECTION – II
Q.5. Select and write the most appropriate answer from the given alternatives for each
sub-question:
i. (C) acetic anhydride [1]
CH3COONa + CH3COCl  (CH3–CO)2O + NaCl
Sodium acetate Ethanoyl Acetic anhydride
chloride

ii. (B) diethyl ether and ethanol [1]


Natalite is used as fuel.
iii. (D) 36 [1]
In ferrocyanide ion, [Fe(CN)6]4–
2e 
Fe   Fe2+ + 6CN–  [Fe(CN)6]4–
Formula : for EAN = Z – X + Y
Here, Z = atomic number of iron = 26.
X = number of electrons lost due to oxidation of Fe to Fe2+ = 2.
Y = number of electrons donated by 6CN– = 6  2 = 12.
 EAN = 26 – 2 + 12 = 36
iv. (B) disaccharide [1]
v. (C) +5 [1]
vi. (A) 3-Bromo-3,4-dimethylheptane [1]

vii. (D) (CH3)3C – NH2 [1]


Tertiary alkyl halides on treatment with alcoholic base undergo elimination in
preference to substitution.
7
Board Answer Paper : July 2016
Q.6. Answer any SIX of the following:
i. a. 3-Methylcyclohexanone [1]
b. Butane-2,3-dione [1]
ii. a. Citrus fruits, onion, cabbage, amla, green leafy vegetables, tomatoes, etc.
(Any two sources) [½  2] [1]
b. Green leafy vegetables, fish, meat, etc.
(Any two sources) [½  2] [1]
iii.
Lanthanoids Actinoids
a. In lanthanoids, last differentiating electron In actinoids, last differentiating electron occupies 5f
occupies 4f orbital. orbital.
b. They are the elements of first inner transition series. They are the elements of second inner transition series.
c. They are present in period 6. They are present in period 7.
d. Most of lanthanoids (with exception of Most of actinoids (with exception of uranium and
promethium) occur in nature. thorium) are prepared in laboratory.
e. Most of lanthanoids (with exception of All the actinoids are radioactive.
promethium) are non-radioactive.
f. Lanthanoids do not form oxo cation. Actinoids form oxo cation such as UO 22  , PuO2+, UO+.
g. 4f orbitals in lanthanoids have higher binding energy. 5f orbitals in actinoids have lower binding energy.
h. Contraction in atomic and ionic radii is relatively Contraction in atomic and ionic radii is relatively greater
less in lanthanoids. in actinoids due to poor shielding of 5f electrons.
i. Lanthanoids show +2, +3 and +4 oxidation states. Actinoids show +3, +4, +5, +6, +7 oxidation states.
j. Lanthanoids have less tendency to form complexes. Actinoids have greater tendency to form complexes.
k. Some ions of lanthanoids are coloured. Most of the ions of actinoids are deeply coloured.
l. Lanthanoid hydroxides are less basic in nature. Actinoid hydroxides are more basic in nature.
(Any four distinguishing points) [½  4] [2]

iv. C6H5  C  N + C6H5 Mg Br  


dry ether
 C6H5  C = NMgBr [1]
Benzonitrile Phenyl
magnesium
C6H5
Adduct
bromide

H3O+
C6H5
C = O + Mg(OH)Br + NH3
C6H5
Benzophenone
(Reaction + names of reactants, products and reagents) [1]
v. a. Raw materials used for the preparation of Buna-S:
CH2 = CH – CH = CH2 [½]
CH = CH2

[½]

b. Raw materials used for the preparation of Dextron:


OH
CH3 – CH – COOH [½]
HO – CH2 – COOH [½]
8
Chemistry
vi. Sandmeyer’s reaction:
When a primary aromatic amine, dissolved or suspended in cold aqueous mineral acid, is
treated with sodium nitrite, a diazonium salt is formed. Mixing the solution of freshly
prepared diazonium salt with cuprous chloride or cuprous bromide results in the replacement
of the diazonium group by –Cl or –Br. This reaction is known as Sandmeyer’s reaction.
(Description of reaction) [1]
+ 
NH2 N2X
NaNO 2  HX

273K

Aniline Benzenediazonium halide [½]
+ 
N2X X

 Cu 2 X 2 + N2
Aryl halide [½]
Benzenediazonium halide
[X = Cl, Br]

vii. a. The reaction of benzenediazonium chloride with phenol in mild alkaline medium yields
p-hydroxyazobenzene (orange dye).

 
OH  
OH
N  NCl + N=N OH + HCl
Benzenediazonium chloride Phenol p-Hydroxyazobenzene
(orange dye)

(Reaction + names of reactants, products and reagents) [1]


b. The reaction of benzenediazonium chloride with aniline in mild alkaline medium yields
p-aminoazobenzene (yellow dye).

 
N  NCl NH2  
OH
+ N=N NH2 + HCl
Benzenediazonium chloride p-Aminoazobenzene
Aniline
(yellow dye)
(Reaction + names of reactants, products and reagents) [1]
viii. Chemical methods used to preserve food:
a. Addition of sugar: In this method, sugar is added and then food is heated. This method
is used for the preparation of jams, jellies and marmalades. Fruits like apples, mangoes,
strawberries, carrots, etc are preserved by this method.
b. Addition of salt: Common salt has antimicrobial activity. Salt is added to food to
control the growth of fungus and micro-organisms. When salt is added, bacterial
cell loses water due to osmosis. Due to this, the cell either remains dormant or dies.
This helps in storage of food. This method of preservation of food by addition of
salt is called salting. It is used for preservation of fish products, meat and for the
preparation of pickles of lemon, chillies, raw mangoes, etc. It is also used in the
preservation of amla, beans, tamarind, etc.
c. Addition of vinegar: Vinegar is added to preserve food like pickles, salad dressings,
mustard, fish, etc.
d. Addition of other chemicals: Chemicals like sodium benzoate, salts of sorbic acid and
propionic acid etc., are used as preservatives.
(Any two methods) [1  2] [2]
9
Board Answer Paper : July 2016
Q.7. Answer any THREE of the following:
i. a. Action of bromine water on glucose: The oxidation of glucose with bromine water
(which is a mild oxidizing agent) forms gluconic acid. This indicates presence of
aldehyde group.

CHO COOH
Bromine water
(CHOH)4 + [O]   (CHOH)4
CH2OH CH2OH
Glucose Gluconic acid

(Reaction + names of reactants, products and reagents) [1]


b. Action of dil. HNO3 on glucose: The oxidation of glucose with dilute nitric acid gives
saccharic acid (a dicarboxylic acid). Hence, a primary alcoholic group (CH2OH) is
present in glucose.
CHO COOH
dil HNO
(CHOH)4 
3
 (CHOH)4
CH2OH COOH
Saccharic acid
Glucose
(Reaction + names of reactants, products and reagents) [1]
c. Action of hydroxylamine (NH2OH) on glucose: The reaction of glucose with
hydroxylamine gives an oxime. This indicates the presence of carbonyl group.
CHO CH = N  OH
NH OH
(CHOH)4 
2  (CHOH)4 + H2O
CH2OH CH2OH
Glucose Glucoxime
(Reaction + names of reactants, products and reagents) [1]
ii. Ligands:
The molecules or ions which are coordinated to the central atom or ion in the coordination
compound are called ligands or donor groups. [1]
Postulates of Werner’s theory:
a. Two types of valencies are shown by most metallic elements:
1. Primary valence or principal valence
2. Secondary valence.
b. The tendency of every metal is to satisfy both primary and secondary valences.
c. The number of secondary valence shown by each metal is fixed.
d. The secondary valence is always directed towards fixed positions in space.
(Four postulates) [½  2] [2]
iii. Reactions involved in the preparation of potassium dichromate from chrome iron ore
(chromite ore):
a. Conversion of chrome iron ore into sodium chromate (Roasting):
4FeO.Cr2O3 + 8Na2CO3 + 7O2  2Fe2O3 + 8Na2CrO4 + 8CO2
Chrome iron Sodium Oxygen Ferric Sodium Carbon
ore carbonate oxide chromate dioxide
(Reaction + names of reactants and products) [1]
10
Chemistry
b. Conversion of sodium chromate into sodium dichromate:
2Na2CrO4 + H2SO4  Na2Cr2O7 + Na2SO4 + H2O
Sodium (conc.) Sodium Sodium
chromate dichromate sulphate
(Reaction + names of reactants and products) [1]
c. Conversion of sodium dichromate into potassium dichromate:
Na2Cr2O7 + 2KCl  K2Cr2O7 + 2NaCl
Sodium Potassium Potassium Sodium
dichromate chloride dichromate chloride
(Reaction + names of reactants and products) [1]
iv. a. Metamerism (positional isomerism): Ethers having same molecular formula but
different alkyl groups attached on either side of the oxygen atom are called metamers
of each other. This phenomenon is called metamerism (positional isomerism). [1]
b. Methyl n-propyl ether:
Structure:
CH3 – O – CH2 – CH2 – CH3 [½]
IUPAC Name: 1-Methoxypropane [½]
c. Action of hot HI on methyl n-propyl ether:
Hot /373 K
CH3 – O – CH2 – CH2 – CH3 + 2HI   CH3I + CH3 – CH2 – CH2 – I + H2O
Methyl n-propyl ether (Conc.) Methyl Propyl iodide
iodide
(Reaction + names of reactants and products + Reaction condition) [1]
Q.8. Answer any ONE of the following:
i. a. Preparation of orlon:
It is prepared by the addition polymerization of acrylonitrile. A peroxide catalyst is
used during the process.
n CH2 = CH 
Polymerization
  CH2  CH  
  CH2  CH 
n
CN CN CN
Acrylonitrile Repeating unit Polyacrylonitrile [1]
(Orlon)
Preparation of teflon:
Teflon is the addition polymer made from the monomer tetrafluoroethylene.
Tetrafluoroethylene is heated under high pressure in presence of oxygen (which acts as a
catalyst) to form Teflon.
n CF2 = CF2 
Polymerization
  CF2  CF2  
 CF2  CF2 n
[1]
Tetrafluoroethylene repeating unit Teflon

b. 1. Ofloxacin – Antibiotic [½]


2. Morphine – Analgesic [½]
3. Ampicillin – Antibiotic [½]
4. Chloramphenicol – Antibiotic [½]
c. 1. CH3 CH3 CH3
NH2 +– Br
N2Br
NaNO 2 /HBr

273 K
 Cu 2 Br2
 + N2 

Diazonium 2–Bromotoluene
2-Methylaniline
bromide (B)
(A)

11
Board Answer Paper : July 2016
CH3
+–
N2Br
A is [½]

CH3
Br
B is [½]


2. 2CH3 – CH2 – CH – CH3 + 2KOH  H3C – CH = CH – CH3 + H3C – CH2 – CH = CH2
(Alc.) But-2-ene But-1-ene
Br (A) (B)
2-Bromobutane
+ 2KBr + 2H2O
A is H3C – CH = CH – CH3 [½]
B is H3C – CH2 – CH = CH2 [½]
3. CH3  CH2  N+(CH3)3I 
Ag 2 O/H 2 O

CH3  CH2  N+(CH3)3OH + AgI
Ethyltrimethyl Ethyltrimethyl
ammonium iodide ammonium hydroxide
(A)

CH3  CH2  N+(CH3)3OH 


  CH2 = CH2 + (CH3)3N + H2O
Ethyltrimethyl Ethylene Trimethyl
ammonium hydroxide (B) amine
(A)
+ –
A is C2H5N(CH3)3OH [½]
B is CH2 = CH2 [½]
ii. a. 1. Conversion of 2-methylbutan-1-ol into 2-methylbutanoic acid:
CH3
K 2 Cr2 O7  dil.H 2SO4
CH3 – CH2 – CH – CH2 – OH + 2[O]  
2-Methylbutan-1-ol CH3
CH3 – CH2 – CH – COOH + H2O
2–Methylbutanoic
acid
(Reaction + names of reactant, product and reagent) [1]
2. Conversion of phenylethene into benzoic acid:
CH = CH2 COOH

KMnO 4 /dil.H 2SO 4

+ CO2

Phenylethene Benzoic acid


(Reaction + names of reactant, product and reagent) [1]
3. Conversion of benzoic acid into meta-nitrobenzoic acid:
COOH COOH

Conc.HNO3

Conc.H SO
 + H2O
2 4
NO2
Benzoic acid m-Nitrobenzoic acid
(Reaction + names of reactant, product and reagent) [1]
12
Chemistry
b. 1. Reaction of primary amine with benzene sulphonyl chloride gives corresponding
N-alkyl benzene sulphonylamide.
Owing to the presence of strong electron withdrawing sulphonyl group, hydrogen
attached to nitrogen atom in N-alkyl benzene sulphonylamide is strongly acidic
and dissolves in aqueous KOH. On acidification of this solution, insoluble amide
gets regenerated.
O O

R  N  H + Cl  S  R  N  S + HCl

H O H O
Primary Benzene N-Alkyl benzene
amine sulphonyl sulphonyl amide
chloride

O O O
  H
RNS 
KOH
 K RN  S  R  N  S
[1]
H O O H O
N-Alkyl benzene N-Alkyl benzene
sulphonyl amide sulphonyl amide

2. Reaction of secondary amine with benzene sulphonyl chloride gives N,N-dialkyl


benzene sulphonylamide.
N,N-dialkyl benzene sulphonylamide does not contain acidic hydrogen. Hence, it
is not acidic and is insoluble in aqueous KOH. It is also insoluble in acid.
O O

R  N  H + Cl  S  R  N  S + HCl
[1]
R O R O
Secondary Benzene N,N-Dialkyl benzene
amine sulphonyl sulphonylamide
chloride

3. Due to absence of hydrogen atom directly attached to the nitrogen atom, tertiary
amines do not react with benzene sulphonyl chloride. The unreacted tertiary
amine is insoluble in aqueous KOH but soluble in acid.
H
+  
C6H5SO2Cl + R3N  C6H5SO2N R3Cl  KOH
 Insoluble Clear solution
Benzene Tertiary compound
sulphonyl amine [1]
chloride

c. Uses of formaldehyde:
1. Formalin (40% solution of formaldehyde) is used as preservative for biological
specimens.
2. Formaldehyde is used for silvering mirror.
3. Formaldehyde is used for the production of several plastic and resins, bakelite
and binders in plywood.
(Any two uses) [½  2] [1]
13
Chemistry

BOARD ANSWER PAPER : MARCH 2017


CHEMISTRY
SECTION – I
Q.1. Select and write the most appropriate answer from the given alternatives for each sub-question:
i. (B) Babbitt metal [1]
ii. (A) Density, viscosity [1]
2+
iii. (D) [Fe(H2O)5NO] [1]
iv. (C) Fusion [1]
v. (B) A3 B [1]
1
As ‘B’ is present at the 8 corners of the cube, number of atoms of ‘B’ in the unit cell =  8 = 1.
8
As ‘A’ atoms are present at the face centres of the 6 faces of the cube, the number of atoms of
1
‘A’ in the unit cell =  6 = 3
2
Ratio of atoms A : B = 3 : 1
The formula of the compound is A3B
vi. (C) 0.50 [1]
The half reaction for the reduction of Al3+ ion is
Al3+ + 3e–  Al
The reaction indicate that 1 mole of Al is produced by the passage of 3 mole electrons. The
charge of 3 mole electrons is 3 faraday.
 3 F of charge produces 1 mole of Al.
 1.5 F of charge produces 0.50 mole of Al.
vii. (B) 0.5 [1]
Rate1 = k[A]x …(i)
Rate2 = k4x[A]x …(ii)
dividing (ii) by (i)
Rate 2
= 4x
Rate1
but Rate2 = 2Rate1 (given)
2Rate1
= 4x
Rate1
2 = 4x
x = 1 or 0.5
2
Q.2. Answer any SIX of the following:
i. a. The galvanic cells in which the energy of combustion of fuels is directly converted into
electrical energy are called fuel cells. (Definition) [1]
b. Cell reactions in a fuel cell
Oxidation at anode (): Oxidation of hydrogen gas to water occurs at anode.

2H2(g) + 4OH (aq)  4H2O(l) + 4e [½]

Reduction at cathode (+): Reduction of oxygen gas to OH ions occurs at cathode.
O2(g) + 2H2O(l) + 4e  4OH (aq)

[½]

1
Board Answer Paper : March 2017
ii. Relationship between half life and rate constant for a first order reaction:
a. For the first order reaction A  products, the integrated rate law is given as follows:
2.303 [A]0
k= log10 ....(1) [½]
t [A]t
where, [A]0 = initial concentration of the reactant at t = 0
[A]t = concentration of the reactant at time t.
[A]0
b. At t = t1/2, [A]t =
2
Equation (1) becomes,
2.303 [A]0
k= log10 ....(2) [½]
t1/ 2 [A]0 / 2
2.303
c.  t1/2 = log10 2 ....(3) [½]
k
2.303
=  0.301
k
0.693
t1/2 = ....(4) [½]
k
d. From the above equation (4) for a first order reaction, half-life period is constant, i.e., it
is independent of initial concentration of the reacting species.
iii. Magnetic separation process:
a. The magnetic separation process is based on the differences in magnetic properties of
the ore components.
b. If either ore or the gangue is attracted by a magnet, then the ore can be separated from
the impurities with the help of magnetic separation method.
c. It requires an electromagnetic separator which consists of a brass or leather belt moving
over two rollers, one of which is magnetic in nature as shown in the figure.
d. Powdered ore is dropped over the moving belt at one end.
e. At the other end, the magnetic portion of the ore is attracted by the magnetic roller and
falls nearer to the roller, while the non-magnetic impurities fall separately farther off.
(Explanation) [1]

Pulverized ore

Magnetic
roller Leather

Non-magnetic
particles Magnetic particles

Magnetic separation using electromagnetic separator


(Diagram) [1]

2
Chemistry
iv. a. Let W2 g of non-volatile solute of molar mass M2 be dissolved in W1 g of solvent of
molar mass M1.
b. The number of moles of solvent, n1 and number of moles of solute n2, in solution are
given as,
W W2  mass of the substance 
n1 = 1 and n2 =  Number of moles (n) =  [½]
M1 M2  molar mass of the substance 

c. The mole fraction of solute, x2 is given by,


n2 W2 / M 2
x2 = = ….(1) [½]
n1  n 2 W1 / M1 + W2 / M 2

d. For a solution of two components A1 and A2 with mole fraction x1 and x2 respectively,
if the vapour pressure of pure component A1 is p1 and that of component A2 is p 2 = 0,
The relative lowering of vapour pressure is given by,
 p p1  p p1 x2
= = = x2 .….(2)
p1 p1 p1
e. Combining equations (1) and (2)
 p p1  p W2 / M 2

= 
= x2 = …..(3) [½]
p1 p1 W1 / M1 + W2 / M 2

f. For dilute solutions n1 >> n2. Hence, n2 may be neglected in comparison with n1 in
equation (1) and thus equation (3) becomes,
p n W / M2 WM

= 2 = 2 = 2 1 [½]
p1 n1 W1 / M1 W1M 2

v. a. Enthalpy: Enthalpy of a system may be defined as the sum of the internal energy of the
system and the energy that arises due its pressure and volume. [1]
b. First law of thermodynamics can be given as:
U = q + W
If work is done by the system on the surroundings (work of expansion), then W is
negative (W). The negative value of ‘W’ signifies that energy has left the system as
work. Therefore, internal energy of the system decreases. [1]
vi. a. The electronic configuration of 7N is 1s2 2s2 2p3. It has 3 unpaired electrons which can
form 3 covalent bonds, thus forming NX3 molecule. [1]
b. Valence shell of Nitrogen (n = 2) contains only ‘s’ and ‘p’ orbitals. [½]
c. Therefore, due to absence of d-orbitals in valence shell, nitrogen cannot expand its
octet and its maximum covalence is restricted to 4. It does not form compounds in +5
oxidation state like NCl5 and NF5. [½]
vii. Simple cubic unit cell:
a. In a simple cubic lattice, the atoms are located only on the corners of the cube. The
particles touch each other along the edge.
b. Thus, the edge length or side of the cube ‘a’, and the radius of each particle, ‘r’ are
related as a = 2r
c. The volume of the cubic unit cell = a3 = (2r)3 = 8r3 [½]
d. Since a simple cubic unit cell contains only 1 atom, [½]
4
The volume of one atom (occupied space) = r3
3
3
Board Answer Paper : March 2017
Volumeof oneatom
e. Packing efficiency =  100 % [½]
Volumeof cubic unit cell
4 / 3r 3 
= 3
 100 =  100
8r 6
= 52.36 %  52.4 %
f. The packing efficiency of simple cubic metal crystal is 52.4 %. [½]
viii. a. Electronic configuration of sulphur (Z = 16) is, 16S  1s22s22p63s23p4 or [Ne]3s23p4 [1]
b. Electronic configuration of krypton (Z = 36) is, 36Kr  1s22s22p63s23p63d104s24p6
or [Ar]3d104s24p6 [1]
Q.3. Answer any THREE of the following:
i. Preparation of Phosphine using
a. HCl: The reaction of calcium phosphide with dilute HCl gives phosphine.
Ca3P2 + 6HCl  3CaCl2 + 2PH3
Calcium Calcium Phosphine
phosphide chloride
(Reaction + names of reactants and products) [1]
b. H2SO4: The reaction of aluminium phosphide with dilute H2SO4 gives phosphine.
2AlP + 3H2SO4  Al2(SO4)3 + 2PH3
Aluminium Aluminium Phosphine
phosphide sulphate
(Reaction + names of reactants and products) [1]
c. Caustic soda (NaOH)
White phosphorus is heated with concentrated NaOH solution in an inert atmosphere of
CO2. It is the laboratory method.
P4 + 3NaOH + 3H2O  PH3 + 3NaH2PO2
White Sodium hydroxide Phosphine Sodium
phosphorus (hot) hypophosphite

(Reaction + names of reactants and products) [1]


–1
ii. Given: Cell constant (b) = 0.367 cm
Molar concentration (C) = 0.05 M
Resistance (R) = 31.6 
To find: Molar conductivity ()
Formulae: a. Cell constant, (b) = k  R
1000 k
b. Molar conductivity () =
C
Calculation: From formula (a),
Cell constant, b = k  R [½]
0.367 = k  31.6
0.367
k= = 11.61  10–3 –1 cm–1 [½]
31.6
From formula (b),
1000 k
Molar conductivity (m) = [½]
C
1000  11.61  103
= [½]
0.05
= 232.2  cm2 mol–1
–1
[½ + ½]
4
Chemistry
iii. Given: The standard enthalpy of combustion of C2H5OH(l) i.e. c H C2H5OH(l) = –1368 kJ
The standard enthalpy of combustion of ethene i.e. c H C2H4(g) = –1410 kJ
To find: H for the enthalpy of formation of liquid C2H5OH
Calculation: Given equations are,
C2H5OH(l) + 3O2(g)  2CO2(g) + 3H2O(l) ; cH = –1368 kJ ...(1)
C2H4(g) + 3O2(g)  2CO2(g) + 2H2O(l) ; cH = –1410 kJ ...(2)
The required equation is,
C2H4(g) + H2O(l)  C2H5OH(l), [½]
To get required equation, reverse equation (1) and add to equation (2). [½]
2CO 2(g) + 3H2O(l)  C2H5OH(l) + 3O 2(g) cH = +1368 kJ
C2H4(g) + 3O 2(g)  2CO 2(g) + 2H2O(l) cH = –1410 kJ

C2H4(g) + H2O(l)  C2H5OH(l) H = +1368 kJ – 1410 kJ [½]
= – 42 kJ. [½]
 The calculated H = – 42 kJ is not the enthalpy of formation of liquid ethanol because
the reaction does not involve the formation of liquid ethanol from its constituent
elements. [1]
iv. Given: Frequency factor (A) = 4  1013 sec–1
Energy of activation (Ea) = 98.6 kJ mol–1
Half life period (t1/2) = 10 minutes = 10  60 = 600 sec.
To find: Temperature(T)
0.693
Formulae: a. k =
t1/2
Ea
b. log10k = log10A –
2.303RT
Calculation: From formula (a)
0.693
k= [½]
t1/2
0.693
k= = 11.55  10–4 sec–1 [½]
600
From formula (b),
Ea
log10 k = log10A – [½]
2.303 RT
98.6 103
log10 11.55  10–4 = log10 4  1013 –
2.303 8.314  T
98.6 103
–2.937 = 13.602 – [½]
19.147  T
5149.632
–2.937 = 13.602 –
T
5149.632
= 13.602 + 2.937
T
5149.632
= 16.539 [½]
T
5149.632
T= = 311.36 K
16.539
T = 311.36 K or 38.36C [½]

5
Board Answer Paper : March 2017
Q.4. Answer any ONE of the following:
i. a. Faraday’s First Law of Electrolysis: It states that “the amount of substance that
undergoes oxidation or reduction at each electrode during electrolysis is directly
proportional to the amount of electricity that passes through the cell.” [1]
b. 1. Uses of sulphuric acid:
i. Sulphuric acid is used in the manufacture of fertilizers.
eg. Ammonium sulphate, superphosphate.
ii. It is used in petroleum refining.
iii. It is used in manufacture of pigments, paints and dyestuff intermediates.
iv. It is used in detergent industry.
v. It is used in storage batteries.
vi. It is used in the manufacture of explosives. eg. T.N.T., nitroglycerine, green
cotton, etc.
vii. It is a laboratory reagent.
viii. Sulphuric acid is used in the preparation of important chemicals like HNO3, HCl,
H3PO4, Na2CO3, sulphates, alums, ether, etc.
ix. It is a dehydrating agent and used for drying wet gases which do not react with acid.
x. It is used as an oxidizing agent.
xi. Sulphuric acid is also used as a pickling agent. Pickling is an industrial process
for removing layers of basic oxides from metals like Fe and Cu before
electroplating, enamelling, galvanizing and soldering.
(Any two uses) [½  2] [1]
2. Uses of chlorine:
i. Large quantities of chlorine are used industrially for bleaching wood pulp
(required for the manufacture of paper and rayon), bleaching cotton and textiles.
ii. It is used in the extraction of gold and platinum.
iii. It is used for the manufacture of dyes, drugs and organic compounds such as CCl4,
CHCl3, refrigerants, etc.
iv. It is used in sterilising drinking water.
v. It is used in preparation of poisonous gases such as phosgene (COCl2), tear gas
(CCl3NO2), mustard gas (Cl.C2H4  S  C2H4Cl).
vi. It is used in the manufacture of DDT and BHC which are important insecticide.
vii. It is used in the manufacture of vinyl chloride which is starting material for
polyvinyl chloride plastics.
viii. It is used in the manufacture of bleaching powder, aluminium chloride, hydrogen
chloride, hypochlorites, chlorates, perchlorates, etc., which are important
industrial compounds.
(Any two uses) [½  2] [1]
c.
Property Crystalline solids Amorphous solids
1. Shape They have definite characteristic They have irregular shape and lack
geometrical shape due to the orderly
characteristic geometrical shape due
regular long range arrangement of to the short range orderly
constituent particles. arrangement of constituent particles.
2. Melting point They have sharp and characteristic They do not have sharp melting
melting point. point. They gradually soften over a
range of temperature.
3. Cleavage When cut with a sharp edged tool, When cut with a sharp edged tool,
property they split into two pieces and the they cut into two pieces with
newly generated surfaces are plain irregular surfaces.
and smooth.

6
Chemistry

4. Heat of fusion They have a definite and They do not have definite heat of
characteristic heat of fusion. fusion.
5. Anisotropy They are anisotropic, i.e., have They are isotropic, i.e., have same
different physical properties in physical properties in all
different direction. directions.
6. Nature They are true solids. They are pseudo solids or super
cooled liquids.
7. Order in They have long range order. They have only short range order.
arrangement of
constituent
particles
eg. Copper, silver, iron, zinc sulphide, Glass, rubber, plastics, etc.
common salt, potassium nitrate,
etc.

(Any four distinguishing points) [½  4] [2]


d. Given: Volume (V) = 1dm3 = 1L
Mass of substance (W2) = 1.8  10–3 kg = 1.8g
Osimotic pressure () = 0.52 atm
R = 8.314 J K–1 mol–1 or R = 0.082L atm K–1 mol–1
Temperature (T) = 280K
To find: Molar mass (M2)
W RT
Formula: = 2
M2V
Calculation: From formula,
W RT
M2 = 2 [½]
V
1.8  0.082  280
M2 = [½]
0.52 1
M2 = 79.47 g mol–1 [1]
ii. a. 1. Leaching: Leaching is the process of extracting a soluble material from an
insoluble solid by dissolving out in a suitable solvent. [1]
2. Metallurgy: The process of extraction of a metal in a pure state from its ore is
called metallurgy. [1]
3. Anisotropy: The ability of crystalline solids to change values of physical
properties when measured in different directions is called anisotropy. [1]
b. 1. Consider ‘n’ moles of an ideal gas placed in a cylinder fitted with weightless,
frictionless piston at a constant temperature ‘T’. If the gas expands isothermally
and reversibly from initial volume V1 to final volume V2 and if the expansion is
by a small volume ‘dV’, then opposing pressure ‘P’ decreases by a small amount
‘dP’ producing small work done ‘dW’. Now, the pressure (pex) will be ‘P – dP’.
2. This work done is given as:
Work done (dW) = pressure  change in volume
= (P  dP)  dV
= (PdV  dPdV)
 Both dP and dV are small, dP.dV is also very small and is neglected.
 The above equation becomes dW =  PdV ….(1) [½]
7
Board Answer Paper : March 2017
3. When the expansion of a gas is carried out reversibly, there will be series of such
‘PdV’ terms. As the pressure decreases continuously, volume gradually increases
from initial volume ‘V1’ to final volume ‘V2’.
Thus, in order to obtain maximum work (Wmax), integrate equation (1) between
Wmax V2

the limits V1 to V2 as 
0
dW    PdV
V1
….(2) [½]

4. The general ideal gas equation for n mole is, PV = nRT


nRT
or P  ....(3)
V
Substituting (3) in equation (2),
Wmax V2
nRT

0
dW   
V1
V
dV

5. Since the number of moles is constant, R is a gas constant and the process is
isothermal,
therefore, T is constant. Hence, nRT = constant.
Wmax V2
1
 
0
dW  nRT 
V1
V
dV [½]

 [W]0max  nRT log e [V]VV12


 dx  x 
 
 [Wmax  0]  nRT[log e (V2 )  log e (V1 )] ……  1 
  x dx  log e x 
 
V2
 Wmax  nRT log e
V1
V2
6. Wmax =  2.303 nRT log10  log e x  2.303 log10 x  [½]
V1
P1
OR Wmax = –2.303 nRT log10 [ According to Boyle’s law, P1V1 = P2V2]
P2
c. Given: Kb for benzene = 2.53 K kg mol–1
Mass of solute = W2 = 1.8 g = 1.8  10–3 kg
Mass of solvent = W1 = 90 g = 90  10–3 kg
Boiling point of solution = Tb = 354.11 K
Boiling point of pure solvent = Tb = 353.23 K
To Find: Molar mass of solute (M2)
K  W2
Formula: Tb = b
M 2  W1
Calculation: Elevation in boiling point (Tb) = Tb  Tb [½]
Tb = 354.11 – 353.23
Tb = 0.88 K [½]
From formula,
K  W2
M2 = b
Tb  W1
2.531.8 103 4.554
M2 = = [½]
0.88 90 103 79.2
–1
= 0.0575 kg mol
= 57.5 g mol–1 [½]
8
Chemistry
SECTION – II
Q.5. Select and write the most appropriate answer from the given alternatives for each
sub-question:
i. (D) an isocyanide [1]
ii. (C) n-butane [1]
CH3 – CH2 – Br 
alc.KOH

 CH2 = CH2 
HBr
 CH3 – CH2 – Br 
Na /ether
Wurtz
 CH3 – CH2 – CH2 – CH3
synthesis
Ethyl Ethene Ethyl n-butane
bromide (B) bromide (D)
(C)
iii. (B) cancer [1]
iv. (C) SO2 [1]
Sulphur dioxide gas is oxidized to sulphuric acid when passed through acidified
potassium dichromate solution. The colour of the solution changes from orange to
green because potassium dichromate is reduced to chromic sulphate.
v. (B) ethanol [1]
Ethanol is used in thermometers as it has low freezing point.
vi. (D) Vitamin A [1]
vii. (C) Ketone [1]
Q.6. Answer any SIX of the following:
i. When an excess of ethyl alcohol is distilled with concentrated H2SO4 at 413 K (140 C),
diethyl ether is formed.
This reaction takes place in two steps:
Step-I: Formation of ethyl hydrogen sulphate: When excess of ethyl alcohol is heated with
concentrated H2SO4, ethyl hydrogen sulphate is obtained.
C2H5  OH + H  O  SO3H  C2H5  O  SO3H + H2O

Ethyl alcohol Conc.sulphuric Ethyl hydrogen


acid sulphate

Step-II: Formation of diethyl ether: Ethyl hydrogen sulphate then reacts with excess of
ethyl alcohol at 413 K and forms diethyl ether.
C2H5  O  SO3H + OH  C2H5 
413K
 C2H5  O  C2H5 + H2SO4
Ethyl hydrogen Ethyl Diethyl ether
sulphate alcohol
(Explanation + Reaction) [2]
ii. Hoffmann bromamide degradation:
a. Primary amine can be prepared by reaction of amide with bromine and aqueous or
alcoholic sodium hydroxide. This reaction is known as Hoffmann bromamide
degradation.
b. It involves molecular rearrangement.
c. An alkyl or aryl group migrates from the carbonyl carbon to the adjacent nitrogen
atom.
d. This reaction is useful for decreasing the length of carbon chain by one carbon atom.
(Explanation) [1]
O
R  C NH2 + Br2 + 4NaOH  R  NH2 + Na2CO3 + 2NaBr + 2H2O
Amide (aqueous or 1 Amine [1]
alcoholic)
9
Board Answer Paper : March 2017
iii. Preparation of ethanoic acid from dry ice:
When solution of methyl magnesium iodide (Grignard reagent) in dry ether is added to solid
carbon dioxide (dry ice), it gives an adduct (magnesium salt of carboxylic acid), which on
acid hydrolysis gives ethanoic acid or acetic acid.
eg.
H3O
O = C = O + CH3  Mg  I  
dry ether
 CH3  C  OMgI   CH3  C  OH + Mg(OH)I
Dry ice Methyl
(Solid carbon O O
magnesium Adduct Acetic acid
dioxide) iodide (Ethanoic acid)

(Explanation + Reaction) [2]


iv. a. Butylated hydroxy anisole (BHA)
Molecular formula: C11H16O2
OH
C(CH3)3

OCH3
Butylated hydroxy anisole (BHA)
(Molecular formula + Structural formula) [½  2] [1]
b. Butylated hydroxy toluene (BHT)
Molecular formula: C15H24O
OH
(CH3)3C C(CH3)3

CH3
Butylated hydroxy toluene (BHT)
(Molecular formula + Structural formula) [½  2] [1]
v. Preparation of glucose from cane sugar:
Glucose can be prepared in the laboratory by boiling sucrose (cane sugar) with dilute hydrochloric
acid or sulphuric acid for about two hours. This hydrolyzes sucrose to glucose and fructose. In
order to separate glucose from fructose, alcohol is added during cooling. Glucose is almost
insoluble in alcohol. It crystallizes out first, while fructose is more soluble. It remains in the
solution. The solution is filtered to obtain the crystals of glucose. (Explanation) [1]

C12H22O11 + H2O 


dil.HClor H 2SO 4

 C6H12O6 + C6H12O6
Sucrose Water Glucose Fructose
(Reaction + names of reactants, products and reagents) [1]
vi. Factors which are related to the colour of transition metal ions are as follows:
a. The presence of unpaired electrons in d-orbitals.
b. d-d transitions of electrons due to absorption of radiation in the visible region.
c. Nature of groups (anions or ligands) linked to the metal ion in the compound or a
complex.
d. Type of hybridisation in metal ion in the complex.
e. Geometry of the complex containing transition metal ion.
(Any four points) [½  4] [2]
10
Chemistry
vii. a. Homopolymers:
Polymers whose repeating structural units are derived from only one type of monomer
units are called homopolymers.
eg. Polythene (prepared by using only one type of monomer, i.e., ethene), PVC (monomer:
vinyl chloride), Polystyrene (monomer: styrene), etc.
(Definition + Any one example) [1]
b. Elastomers:
The polymers that have elastic character like that of rubber are called elastomers.
eg. Neoprene, vulcanized rubber, etc.
(Definition + Any one example) [1]
viii. a. Racemic mixture or Racemates: A mixture of equimolar amounts of dextro and laevo
rotatory forms of same optically active substance is called racemic mixture or
racemates.
eg. dl-lactic acid or ()-lactic acid. [1]
CH3
b.
CH3  CH2  CH  CHO
IUPAC name is 2-Methylbutanal [1]
Q.7. Answer any THREE of the following:
i. a. Effective atomic number (EAN): Effective atomic number (EAN) is the total number
of electrons around the central metal ion present in a complex. It is the sum of the
electrons of metal ion and the electrons donated by the ligands.
EAN = Z  X + Y

where, Z = Atomic number of the metal


X = Number of electrons lost during oxidation of metal to metal ion
Y = Number of electrons donated by the ligands. (Definition + Formula) [1]
b.
Atomic Number of Number of
number electrons electrons EAN
Complex
of metal lost by from ligands (Z – X + Y)
atom (Z) metal (X) (Y)
1. K4Fe(CN)6 26 2 6  2 = 12 26 – 2 + 12 = 36 [1]
2. Cr(CO)6 24 0 6  2 = 12 24 – 0 + 12 = 36 [1]
ii. a. Oxidation states of Fe:
+2, +3, +4, +5, +6 [1]
b. Electronic configuration of Mn2+:1s2 2s2 2p6 3s2 3p6 3d5 4s0
Due to the presence of half filled ‘d’ orbital, the +2 oxidation state of manganese is
more stable.
(Electronic configuration + Explanation) [2]
iii. Aldol condensation (Self/Auto condensation):
a. Two molecules of an aldehyde or ketone (containing -H-atom) in the presence of
dilute basic solution of NaOH, KOH, Ba(OH)2, K2CO3 or Na2CO3 or dil.HCl undergo
addition reaction to give corresponding -hydroxyl aldehyde (aldol) or -hydroxyl
ketone (ketol). The reaction is called aldol condensation/self oxidation.
b. This is a nucleophilic addition of aldehydes (or ketones) in which -carbon of the first
molecule gets attached to carbonyl carbon of second molecule forming a new C  C
bond. The reaction is reversible but the equilibrium is established between reactants
and products.
11
Board Answer Paper : March 2017
c. Due to steric factors, yield of ketol is very poor as compared to that of aldol.
d. On heating, the aldol or ketol rapidly undergoes dehydration (in the presence of acid or
base) to yield corresponding -unsaturated aldehyde or ketone. This product is obtained
due to stabilization of newly formed C  C double bond via conjugation with carbonyl
group. (Explanation) [2]
eg. Aldehydes: When acetaldehyde is warmed with dil NaOH or Na2CO3 or K2CO3 or
dil. HCl, it gives acetaldol, (3-Hydroxybutanal), which on heating with acid or base, loses a
water molecule and gives crotonaldehyde.

H H H H H H
| | | | | |
293 303K, dil Na 2 CO3
CH3  C + 
H  C  C = O  
 CH3  C  C  C = O
|| | | |
O H B OH H
Acetaldehyde Acetaldehyde -Hydroxybutyraldehyde (acetaldol)
(First molecule) (Second molecule) (3-Hydroxybutanal) [½]

H H H H H H
H / 
CH3  C  C  C = O 
dehydration
 CH3  C = C  C = O + H2O [½]
Crotonaldehyde
HO H (But-2-enal)
Acetaldol
iv. a. Nucleic acids: Nucleic acids are biologically important polymers present in all living
cells, which direct the synthesis of proteins and are responsible for the transfer of
genetic information i.e., the hereditary characteristics. [1]
b. Complex lipids: The esters of long chain fatty acids which can be easily hydrolyzed
are called complex lipids. [1]
c. Functions of lipids:
1. Oils and fats can supply food energy in plants and animals.
2. Glycolipids are present in bacterial cell wall.
3. In plants, the principal lipid constituents of chloroplasts are glycolipids.
4. Cerebrosides are present in plasma membranes of neural tissues. They are animal
glycolipids. They are abundant in myelin sheath of neurons.
5. In water, phospholipids form membrane like structure.
6. The major components of cell membranes are phospholipids and sterols such as
cholesterol.
7. Waxes are water repelling solids. They provide waterproofing for body surfaces. They
form protective coatings on leaves, fruits, berries, animal fur and feather of birds.
8. Adrenal hormones, sex hormones and bile acids are steroids. Lipoproteins are the
combination of lipids with proteins. They are found in cell membranes.
9. Bile acids (such as cholic acid) aid digestion of fat in intestine.
10. Prostaglandins have a wide range of biological effects.
11. Vitamin A, E, K and phytol are terpenes. They are present in essential oils such as
menthol and camphor. The glands of certain aromatic plants secrete essential oils
which contains terpenes as the main constituents.
(Any two functions) [½  2] [1]
12
Chemistry
Q.8. Answer any ONE of the following:
i. a. Action of nitrous acid i.e., mixture of NaNO2 and dil. HCl on:
1. Ethylamine:
Ethylamine reacts with nitrous acid in cold condition to give ethyl alcohol and nitrogen gas.
NaNO 2  dil.HCl
C2H5  NH2 + HO  N = O  273 278K
C2H5OH + H2O + N2
Ethylamine Ethyl alcohol [1]
2. Aniline:
Aniline reacts with nitrous acid in cold condition to give benzene diazonium salt.
NH2 N 2 Cl

NaNO 2  dil.HCl
+ HO – N = O 
273 278 K
+ NaCl + 2H2O
Aniline Benzenediazonium chloride [1]
3. Diethylamine:
Diethylamine reacts with nitrous acid in cold condition to give N-nitrosodiethylamine.
NaNO 2  dil.HCl
(C2H5)2NH + HO – N = O  273 278 K
(C2H5)2N – N = O + H2O
Diethylamine N–Nitrosodiethylamine [1]
b. Preparation of Nylon-6,6:
1. Nylon-6,6 is prepared by the condensation polymerization of hexamethylenediamine
with adipic acid under high pressure and at high temperature.
2. Equimolar aqueous solutions of both monomers are mixed. Nylon salt is formed by
neutralization.
+ +  
H2N ( CH2 ) NH2 + HOOC ( CH2 )4 COOH   H3N ( CH2 )6NH3OOC ( CH2 )4 COO
6
Hexamethylenediamine Adipic acid Hexamethylene diammonium adipate
(Hexane-1,6-diamine) (Hexane-1,6-dioic acid) (Nylon salt)

(Formation of Nylon salt) [1]


3. The step growth condensation of the nylon salt with elimination of water molecule gives
nylon-6,6 polymer. O O
+ +   
nH3N ( CH2) NH3OOC( CH2) COO   H O HN ( CH2 )6 NH  C ( CH2 )4 C 
6 4 2
Hexamethylene diammonium Repeating unit
adipate
(Nylon salt) O O

[ HN ( CH2 ) NH  C ( CH2 )4 C ]n
6
Nylon-6,6
(Formation of polymer) [1]
c. The chemical substances which neutralize excess acid in the gastric juices and give
relief from acid indigestion, acidity, heart burns and gastric ulcers are called antacids.
eg. Magnesium hydroxide, magnesium carbonate, magnesium trisilicate,
aluminium hydroxide, sodium bicarbonate, etc.
(Definition + Any one example) [1]
d. Side effects of tranquilizers are:
1. It produces drowsiness.
2. It is addictive.
3. It causes headache, fatigue, dizziness and muscle weakness.
4. It also causes visual disturbances (blurring of vision), discomfort and weight gain.
(Any two side effects) [½  2] [1]
13
Board Answer Paper : March 2017
ii. a.
1. Alkaline hydrolysis of tert-butyl bromide can be explained by unimolecular
nucleophilic substitution S   mechanism.
N1

Reaction:
CH3 CH3
CH3  C  Br + OH  CH3  C  OH + Br
(Nucleophile) (Bromide ion)
[½]
CH3 CH3
(tertButyl bromide) (tertButyl alcohol)
2. Kinetics:
Rate  [(CH3)3C  Br]
Rate = k[(CH3)3C  Br]
Hence, it is a first order reaction. [½]
3. Unimolecular nucleophilic substitution (SN ) is a two step reaction process. The first step is
1

a slow step, while the second one is a fast step. The mechanism can be written as follows:
i. Formation of carbonium ion:
a. The C – Br bond in tertiary butyl bromide slowly dissociates to form bromide ion (Br)
and tertiary butyl carbonium ion [+C(CH3)3].
b. This is a slow process and hence, it is a rate determining step (R.D.S.).
CH3
H3C CH3
* + 
C Slow step

 
 H3C  C Br  +
+ Br
R.D.S. C
H3C Br
H3C 120
CH3 H3C CH3
tert-Butyl bromide tert-Butyl carbonium ion [½]
Transition state I /carbocation

ii. Formation of the product:


a. The second step is the attack of OH leading to the C  OH bond formation.
b. It is a fast process. CH3

Back side
attack C*
[½]
HO CH3
CH3 CH3
H3C
+  tert-Butyl alcohol
C+ Fast H3C  C OH [Inversion of

 configuration (50%)]
120
H3C CH3 H3C

HO CH3
tert-Butyl carbonium ion
/carbocation Transition state II
Front side
attack C* [½]
H3C OH
CH3
tert-Butyl alcohol
[Retention of
configuration (50%)]

14
Chemistry

iii. Stereochemistry of S N1:

The intermediate formed after the bond has broken is known as carbocation (or
carbonium ion). In the carbocation, the carbon atom is sp2 hybridized and therefore, it
has planar geometry. The nucleophile can attack it either from front side or from back
side with nearly same probability. The products obtained in this case due to attack from
either side are the same, as the starting compound itself is optically inactive.

CH3 
HO CH3 OH CH3

+
C*     C*
Back side Front side
attack C attack

CH3 H3C CH3


HO H3C OH
CH3 CH3
tert-Butyl alcohol tert-Butyl alcohol
[Inversion of [Retention of
configuration (50%)] configuration (50%)] [½]

iv. Energy profile diagram:

First step where,


T.S.I Second step
T.S.II Reactants = C(CH3)3Br + OH
E1 = Activation energy for T.S.I.
Potential energy

E1 E2
E2 = Activation energy for T.S. II
Carbocation
H = Heat of reaction and
product = C(CH3)3OH
Reactants
H CH3
Carbocation =
+
Products C
H3C CH3
Reaction co-ordinate 
Energy profile diagram for S N1 mechanism (Diagram) [1]

b. Carbolic acid: Carbolic acid or Phenol is an organic aromatic, hydroxyl compound,


in which the hydroxyl (OH) group is directly attached to the aromatic nucleus (i.e.,
benzene ring). [1]
c. Preparation of carbolic acid from benzene sulphonic acid:
1. When benzene sulphonic acid is neutralised by aqueous sodium hydroxide (NaOH),
sodium benzene sulphonate is obtained.
 +
SO3H SO3Na

+ NaOH(aq.) 
 Neutralisation
+ H2O

Benzene Sodium benzene [½]


sulphonic acid sulphonate

15
Board Answer Paper : March 2017
2. Dry sodium benzene sulphonate when fused with excess of sodium hydroxide at 573 K,
sodium phenoxide is obtained along with sodium sulphite (Na2SO3).
 +  +
SO3Na ONa

+ 2NaOH   fused


573K + Na2SO3 + H2O
Sodium
Sodium benzene (Excess) Sodium
sulphite
sulphonate phenoxide
(Reaction) [½]
(Mentioning temperature) [½]

3. Sodium phenoxide when hydrolysed by heating with dilute sulphuric acid, phenol is
obtained.
 +
ONa OH

2 + H2SO4 
Hydrolysis
2 + Na2SO4
(dil.)
Sodium Phenol or
phenoxide carbolic acid

OR
When a current of carbon dioxide is passed through aqueous sodium phenoxide, phenol
is obtained as product.
 +
ONa OH

+ H2O + CO2  + NaHCO3

Sodium Phenol or
phenoxide carbolic acid
(Any one reaction) [½]

16
Chemistry

BOARD ANSWER PAPER : JULY 2017


CHEMISTRY
SECTION – I
Q.1. Select and write the most appropriate answer from the given alternatives for each
sub-question:
i. (C) MgO [1]
The oxide which combines with water to form a base is termed as basic oxide.
MgO + H2O  Mg(OH)2(aq)
Magnesium Magnesium
oxide hydroxide

ii. (B) comma [1]


e.g. Fe3+, Fe2+ | Pt
iii. (A) 6 [1]
iv. (B) mol dm–3 t–1 [1]
v. (D) flux [1]
vi. (C) 1.0 M [1]
10
Molarity of NaOH solution = = 1.0 M
40  0.25

vii. (B) 5.5 atm [1]


W = – Pex.V
55 =  Pex(18.5 – 28.5)
55
 Pex = = 5.5 atm
10

Q.2. Answer any SIX of the following:


i. a. Henry’s law states that “the solubility of a gas in liquid at constant temperature is
proportional to the pressure of the gas above the solution.” [1]
b. The solubility of a gas in a liquid decreases with increase in temperature.
Reason: According to Charles’ law, volume of a given mass of a gas increases with
increase in temperature. Therefore, volume of a given mass of dissolved gas in solution
also increases with increase in temperature, so that it becomes impossible for the
solvent in solution to accommodate gaseous solute in it and gas bubbles out. Hence,
solubility of gas in liquid decreases with increase in temperature. [1]
ii. a. This method is based upon catalytic oxidation of NH3 by atmospheric oxygen.
Pt /Rh gaugecatalyst
4NH3(g) + 5O2(g)  500K,9bar
 4NO(g) + 6H2O(g)
Ammonia (From air) Nitric oxide [1]
b. Nitric oxide reacts with oxygen to form nitrogen dioxide.
2NO(g) + O2(g)   2NO2(g)
Nitric oxide Oxygen Nitrogen dioxide [½]
c. Nitrogen dioxide on dissolution in water gives nitric acid.
3NO2(g) + H2O(l)  2HNO3(l) + NO(g)
[½]
Nitrogen dioxide Water Nitric acid
1
Board Answer Paper : July 2017
iii. a. Ammonium phosphate  Ionic solid [½]
b. Brass  Metallic solid [½]
c. S8 molecule  Molecular solid [½]
d. Diamond  Covalent or network solid [½]
iv. In the given representation of the cell, a standard Zn2+(1M)|Zn electrode is combined with
standard hydrogen electrode to form the cell:
Zn | Zn2+(1M) || H+(1M) | H2(g, 1 atm) | Pt
e e

() salt bridge


Zinc anode (+)
H2 gas (1 atm)
SHE
1M
ZnSO4
solution 1 M H+ ion solution

Standard zinc electrode combined with SHE


(Diagram + labelling) [2]
v. a. ZnO reacts with acid to form salt. This shows that ZnO acts as a basic oxide. [½]
eg. ZnO + 2HCl  ZnCl + HO [½]
(s) (aq) 2(aq) 2 (l)
(Basic)

b. ZnO reacts with base to form salt. This shows that ZnO acts as an acidic oxide. [½]
eg. ZnO + 2NaOH  Na ZnO + HO [½]
(s) (aq) 2 2(aq) 2 (l)
(Acidic) (excess)
ZnO acts as both acidic as well as basic oxide. Hence, zinc oxide is an amphoteric oxide.
vi. Minerals of aluminium:
Name of mineral Formula
[1]
a. Bauxite Al2O3.2H2O
b. Cryolite Na3AlF6 [1]

vii. a. Role of N2O: N2O acts as an intermediate since it is produced in first step and
consumed in second step. [1]
b. The first step is trimolecular/termolecular. [½]
The second step is bimolecular. [½]
viii. The mathematical expression of first law of thermodynamics is U = q + W.
Name of process Condition Mathematical expression of the first law
a. Isothermal T = 0 hence W = q [½]
U = 0
b. Adiabatic q=0 U = +W [½]
c. Isochoric V = 0, W = 0 U = qV [V is constant, hence q = qv] [½]
d. Isobaric P = Constant, U = qP + W
hence, q = qP U = qp  pex.V [ W =  pex.V]
[½]
or [qP = U + pex.V]
2
Chemistry
Q.3. Answer any THREE of the following:
i. Calculation:
We require to calculate the rate constant at different time intervals.
2.303 [A]
k= log10 0 ....[Integrated rate law for first order reaction]
t [A]t
a. [A]0 = 0.624
[A]t1 = 0.446
t1 = 600 s
2.303 [A]0
k1 = log [½]
t1 [A]t1
2.303 0.624
k1 = log
600 0.446
 k1 = 3.838  103 log [1.399]
 k1 = 3.838  103  0.1458
 k1 = 5.596  104 s1 [½]
2.303 [A]0
b. k2 = log10
t2 [A]t 2
[A]0 = 0.624
[A]t 2 = 0.318
t2 = 1200 s
2.303 0.624
k2 = log
1200 0.318
 k2 = 1.919  103 log (1.962)
 k2 = 1.919  103  0.2927
 k2 = 5.617  104 s1 [½]
2.303 [A]0
c. k3 = log10
t3 [A]t3
[A]0 = 0.624
[A]t3 = 0.226
t3 = 1800 s
2.303 0.624
k3 = log
1800 0.226
3
 k3 = 1.279  10 log [2.7610]
 k3 = 1.279  103  0.4411
 k3 = 5.641  104 s1 [½]
4 4 4
k1  k 2  k 3 (5.596  10 )  (5.617  10 )  (5.641  10 )
k= =
3 3
4 1
k = 5.618  10 s [½]
All the k values calculated at different time intervals are almost the same. This implies that
the reaction obeys the integrated rate equation of first order reaction. Hence, the reaction is a
first order reaction. [½]
3
Board Answer Paper : July 2017
ii. Given: C(s) + O2(g)  CO2(g) ; fH = –393.3 kJ mol–1 ...(1) [½]
1
H2(g) + O2(g)  H2O(l) ; fH = –285.8 kJ mol–1 ...(2) [½]
2
2C(s) + 2H2(g) + O2(g)  CH3COOH(l) ; fH = 483.2 kJ mol–1 ...(3) [½]
To find: The standard enthalpy of combustion of CH3COOH(l) (cH)
Calculation: The required equation is,
CH3COOH(l) + 2O2(g)  2CO2(g) + 2H2O(l) [½]
Multiplying equation (1) and equation (2) by 2, then adding to reverse of
equation (3). [½]
2C(s) + 2O2(g)  2CO2(g) ; fH = –786.6 kJ mol–1 ...(4)
–1
2H2(g) + O2(g)  2H2O(l) ; fH = –571.6 kJ mol ...(5)
–1
CH3COOH(l)  2C(s) + 2H2(g) + O2(g) ; fH= 483.2 kJ mol ...(6)
[½]
CH3COOH(l) + 2O2(g)  2CO2(g) + 2H2O(l) ; cH = –875 kJ mol–1 [½]

iii. Given: The Cell reaction at 25 C,



Ni(s) + 2Ag (aq) (1 M)  Ni (aq)
2
(1 M) + 2Ag(s)
E Ni = –0.25 V
E Ag = 0.799 V
To find: a. Cell representation
b. Equilibrium constant (K)
Calculation: Cell representation:
2 
Ni(s) | Ni (aq) (1 M) || Ag (aq) (1 M) | Ag(s) [1]
Calculation of equilibrium constant
0.0592
E cell = log10 K [½]
n
E cell = E Ag – ENi [½]
= 0.799  (–0.25) = 1.049 V
0.0592
Hence, 1.049 = log10 K
2
1.049  2
 log10 K = = 35.44 [½]
0.0592
 K = antilog (35.44)
= 2.754  1035 [½]

iv. a. Concentrated sulphuric acid reacts with PCl5 to form chlorosulphuric acid.
HOSO2OH + PCl5  ClSO2OH + POCl3 + HCl [½]
Sulphuric Phosphorus Chlorosulphuric
acid pentachloride acid

Chlorosulphuric acid further reacts with PCl5 to form sulphuryl chloride.


ClSO2OH + PCl5  ClSO2Cl + POCl3 + HCl [½]
Chlorosulphuric Phosphorus Sulphuryl
acid pentachloride chloride

4
Chemistry
b. Sulphuric acid oxidizes metal Cu.
Cu + 2H2SO4  CuSO4 + SO2 + 2H2O [1]
Copper Sulphuric Copper Sulphur Water
acid sulphate dioxide

c. Sulphuric acid reacts with KClO3 to form KHSO4, HClO4 and ClO2.
3KClO3 + 3H2SO4  3KHSO4 + HClO4 + 2ClO2 + H2O [1]
Potassium Sulphuric Potassium Perchloric
chlorate acid hydrogen acid
sulphate
Q.4. Answer any ONE of the following:
i. a. Molality (m): Molality is defined as the number of moles of the solute dissolved in one
kilogram (kg) of the solvent. [1]
b. Osmotic pressure: The excess of pressure on the side of the solution, that stops the net
flow of solvent into the solution through a semipermeable membrane is called osmotic
pressure. [1]
c. Advantages of calomel electrode:
1. It is convenient to handle, easy to construct and transport.
2. No separate salt bridge is required for its combination with other electrode.
3. The potential of the electrode is reproducible and remains constant.
(Any two advantages) [½  2] [1]
d. Given: Edge length of FCC unit cell (a1) = 3.5 Å
Edge length of BCC unit cell (a2) = 3.0 Å
d1 (FCC)
To find: Ratio of density of FCC and BCC =
d 2 (BCC)
ZM
Formula: Density of unit cell = d = g cm–3
NA  a 3

4M
Calculation: Density of FCC unit cell = d1 = [½]
N A  (3.5 Å)3
2M
Density of BCC unit cell = d2 = [½]
N A  (3.0 Å)3
 Ratio of densities of FCC and BCC unit cell is,
d1 M N A  (3.0 Å)3
=  [½]
d2 N A  (3.5 Å)3 2M
d1 54
= = 1.259  1.26 [½]
d2 42.875

e. 1. Increasing order of thermal stability:


HClO < HOClO < HOClO2 < HOClO3
OR
HClO < HClO2 < HClO3 < HClO4 [1]
2. Increasing order of oxidizing power:
HOClO3 < HOClO2 < HOClO < HClO
OR
HClO4 < HClO3 < HClO2 < HClO. [1]
5
Board Answer Paper : July 2017
ii. a. Given: T = 273.15 + 12 = 285.15 K
V = 2000 cm3 = 2 L [½]
M2 = 169 g mol–1
R = 0.0821 L atm K–1 mol–1
 = 0.54 atm
To find: Mass of solute (W2)
W2 RT
Formula: = [½]
M2V
Calculation: From formula,
M 2 V
W2 =
RT
0.54  169  2
W2 = [½]
0.0821  285.15
182.52
W2 = = 7.796 g [½]
23.4108
b. 1. In face-centred cubic unit cell, eight constituent particles (spheres) are present at
eight corners of unit cell. Six constituent particles (spheres) are present at centres
of six faces.
2. The total number of atoms in a FCC unit cell
1 1
= 8 corner atoms  atom per unit cell + 6 atoms at the faces  atom per unit cell
8 2
= 1 +3 = 4 [1]
c.
No. Isothermal process Adiabatic process
1. The temperature of system remains The temperature of system changes i.e.
constant (T = 0). increases or decreases (T  0).
2. System can exchange heat with the System cannot exchange heat with the
surroundings (q  0). surroundings (q = 0).
3. The internal energy of the system The internal energy of the system changes
remains constant (U = 0). (U  0).
4. System is not thermally isolated from System is thermally isolated from its
its surroundings. Hence, open or surroundings. Hence, isolated system is
closed system is required. required.
5. In this U = 0, hence q = W. In this q = 0, hence U = W.
6. Enthalpy remains constant (H = 0). Enthalpy changes in this process (H  0).
7. Expansion takes place by absorbing Expansion takes place by utilizing internal
heat from surroundings. energy of the system.
8. Heat supplied is used only for doing Work done is only at the cost of internal
work. energy.
9. Compression takes place by losing Compression takes place by adding heat
heat to the surroundings. into internal energy of the system.
eg. Fusion of ice. Expansion of gas in vacuum.
(Any four points) [½  4] [2]
d. The various steps involved in the extraction of pure metals from their ores are as follows:
1. Concentration of ores [½]
2. Conversion of ores into oxides or other desired compounds. [½]
3. Reduction of ores to form crude metals. [½]
4. Refining of metals [½]
6
Chemistry
SECTION – II
Q.5. Select and write the most appropriate answer from the given alternatives for each
sub-question:
i. (A) Propan-1-amine [1]
H H

C2H5  C = O + NH2OH  C2H5  C = NOH 


Na /C2 H5OH
Δ
 C2H5  CH2  NH2 + H2O
Propanal (oxime) Propan-1-amine
[A] [B]

ii. (B) 3 > 2 > 1 [1]


iii. (D) number of possible ligands around metal ion in complex [1]
EAN rule can be used to determine the number of possible ligands around a metal ion
as the ligands are added to a metal ion until the number of electrons around the metal
ion reaches to its EAN.
iv. (D) V2+ [1]
V (Z = 23): [Ar] 3d3 4s2
V2+ : [Ar]3d3
Due to the presence of 3 unpaired electrons, V2+ is coloured.
v. (C) 2,4,6-trinitrophenol [1]
OH OH
O2N NO2
+ 3HNO3 
conc.H 2SO 4
+ 3H2O

Phenol (conc.)
NO2
Picric acid
(2,4,6-Trinitrophenol)
vi. (C) hydrogen bond [1]
vii. (B) acetaldehyde [1]
H H H

CH3  C  Cl + 2NaOH(aq) 
2 NaCl
CH3  C  OH  CH3  C = O + H2O
Cl OH Acetaldehyde
Ethylidene
dichloride Unstable

Q.6. Answer any SIX of the following:


i. Preparation of phenol from cumene:
a. When an alkaline solution of cumene (isopropyl benzene or 2-Phenylpropane) in
sodium carbonate is oxidised by passing air or oxygen in the presence of cobalt
naphthenate as a catalyst at 423 K, cumene hydroperoxide is obtained. [½]
CH3 CH3

H3C  C  H H3C  C  O  O H
[½]
+ O2  
Cobalt naphthenate
423K /(alkaline medium)
(Air)
Cumene or Cumene hydroperoxide
(Isopropyl benzene
or 2-Phenylpropane)

7
Board Answer Paper : July 2017
b. Auto oxidation: Cumene hydroperoxide on heating with dilute H2SO4 decomposes
forming phenol and acetone. In this method, acetone is obtained as an important
byproduct which is separated by distillation.
CH3
H3C  C  O  O  H OH
O
[1]


dil.H 2SO 4
 + CH3  C  CH3
Phenol Acetone
Cumene
hydroperoxide

ii. a. Self oxidation and reduction (disproportionation) of aldehydes (which do not possess
an hydrogen atom) in the presence of concentrated alkali (aqueous or alcoholic) is
known as Cannizzaro reaction. The reaction products are alcohol and salt of
carboxylic acid. [½]
b. In this reaction, alcohol is obtained due to reduction of one molecule of aldehyde and
carboxylic acid is obtained due to oxidation of another molecule of an aldehyde. The
carboxylic acid forms a salt with alkali. [½]
eg. H H H O
 
H  C = O + H  C = O + NaOH  H  C OH + H  C  ONa [1]
Formaldehyde (50%) Sodium formate
H
Methyl alcohol
iii. Nitrogen atom of amines contains a lone pair of electrons which can be donated. Thus, amines act
as Lewis bases. Amines are Lowry-Bronsted bases as they accept a proton. Thus, amines act as
bases and nucleophiles.

R  NH2 + HX  R NH 3 X  [1]



Amine Alkyl ammonium halide
Halogen acid
eg. a. The reaction of ethylamine with dilute hydrochloric acid results in the formation
of ethyl ammonium chloride.
 CH3  CH2  NH 3 Cl
CH3  CH2  NH2 + HCl 
Ethylamine Ethyl ammonium
(Ethanamine) chloride
b. The reaction of aniline with dilute hydrochloric acid results in the formation of
anilinium chloride.
NH2 NH 3 Cl

+ HCl 


Aniline Anilinium chloride (Any one example) [1]


iv. Antiseptics: Drugs which are applied to the living tissues to kill the bacteria and to stop
their growth in wound, thus preventing its infection are called antiseptics. [1]
eg. Dettol, Phenol (dilute solution), Bithionol, Boric acid, Hydrogen peroxide, Iodoform,
Soframycin, etc.
(Any two examples) [½  2] [1]
8
Chemistry
v. a. Action of hydroxylamine (NH2OH) on glucose: The reaction of glucose with
hydroxylamine gives an oxime. This indicates the presence of carbonyl group.
CHO CH = N  OH
NH OH
(CHOH)4 
2  (CHOH)4 + H2O
CH2OH CH2OH [1]
Glucose Glucoxime

b. Action of hydrogen cyanide (HCN) on glucose: The reaction of glucose with


hydrogen cyanide gives cyanohydrin. This indicates the presence of carbonyl group.
CN
CHO CHOH
(CHOH)4 HCN
 (CHOH)4
CH2OH CH2OH [1]
Glucose Glucose cyanohydrin

vi. Structure of chromate and dichromate ions:


O 2

Cr
O
O [1]
O
Chromate ion
2
O O O
O Cr 126 Cr O
O O [1]

Dichromate ion
vii. a. Dacron (Terylene) is obtained by the condensation polymerization of ethylene glycol
(1,2-Ethanediol) and dimethylterephthalate.
b. Monomers are heated at 503 K. The catalyst used is a mixture of zinc acetate and
antimony trioxide.
c. Transesterification followed by polymerization gives terylene.
O O
Catalyst,Δ
n HO  CH2  CH2  OH + n H3C O  C C  O  CH3 
CH OH

3

Ethylene glycol DMT (Dimethylterephthalate)

O O O O
Polymerisation
CH2  CH2  O C CO  
 CH2  CH2  O  C CO
n
Repeating unit Polyester

(Explanation + Two steps) [1  2] [2]


9
Board Answer Paper : July 2017
viii. CH3  Br + Mg 
dry ether
 CH3  MgBr
Methyl bromide Methyl magnesium
bromide (A)

CH3  MgBr + CO2 
dry ether
 CH3  C  OMgBr 
H / H2O
 CH3COOH + Mg(Br)OH
(A) (dry ice) Acetic acid (B)
O
Adduct

A  CH3MgBr (Methyl magnesium bromide) [1]


B  CH3COOH (Acetic acid) [1]

Q.7. Answer any THREE of the following:


i. Ligands are classified as mono or unidentate, poly or multidentate and ambidentate ligands:
a. Mono or unidentate ligands: The ligand molecule or ion which has only one donor
atom or one point of attachment and can coordinate with the metal ion at only one site
in a complex is called unidentate or monodentate ligand.
eg. Cl, OH–, NH3, H2O, etc. (Explanation + Example) [1]

b. Poly or multidentate ligands: The ligand molecule or ion which has two or more
donor atoms or points of attachments and can be linked to the same metal in a complex
using two or more donating sites is called poly or multidentate ligands. Thus,
multiple sites of ligands are used in the coordination with metal. Based on the number
of donor atoms, polydentate ligands are further classified as bidentate, tridentate,
tetradentate, etc.
1. Bidentate ligands have two donor atoms.
eg. Ethylenediammine (en)
2. Tridentate ligands have three donor atoms.
eg. Diethylenetriammine (dien)
3. Tetradentate ligands have four donor atoms.
eg. Triethylenetetraammine (trien)
4. Hexadentate ligands have six donor atoms.
eg. Ethylenediamminetetraacetate (EDTA)
(Explanation + Examples) [1]

c. Ambidentate ligands: Ambidentate ligands are the ligands which have two or more
donor atoms capable of forming coordinate bonds; however only one donor atom is
utilized during complex formation.
eg. NO 2 group can form complexes by utilizing either N or O as donor atom, but not
both. This results in formation of either MNO2 or MONO complex
respectively.
(Explanation + Example) [1]

ii. a. Lanthanoid contraction:


The atomic and ionic radii of lanthanoids show gradual decrease with increase in
atomic number. It is known as Lanthanoid contraction. [1]
0 1 2
b. 57La  Electronic configuration is [Xe]4f 5d 6s [½]
After losing 3 electrons La forms La3+ ion which is stable due to empty 4f-orbitals. [½]
2 0 2
58Ce  Electronic configuration is [Xe]4f 5d 6s [½]
4+
After losing 4 electrons Ce forms Ce ion which is stable due to empty 4f-orbitals. [½]
Thus, lanthanum (Z = 57) forms La3+ ion, while cerium (Z = 58) forms Ce4+ ion.
10
Chemistry
iii. a. Propanone on reaction with phenyl hydrazine produces acetonephenylhydrazone.
CH3 CH3

CH3  C = O + H2N  NH  C6H5  CH3  C = N  NH  C6H5 + H2O [1]


Propanone Phenyl hydrazine Acetonephenylhydrazone
b. Propanone on reduction with zinc-amalgam and concentrated hydrochloric acid gives
propane.
Zn  Hg  conc.HCl
CH3  CO  CH3 + 4[H]  
 CH3  CH2  CH3 + H2O
Propanone Propane [1]

c. Propanone, when shaken with a saturated aqueous solution of sodium bisulphite gives
acetone sodium bisulphite.
CH3 CH3
CH3  C = O +  CH3  C  OH
NaHSO3(aq) 
Propanone Sodium bisulphite [1]
SO3Na
Acetone sodium bisulphite

iv. a. Enzymes are defined as biocatalysts, produced by the living cells which catalyse many
biochemical reactions in animals and plant bodies. [1]
b. 1. -Amino acids are bifunctional compounds containing a carboxylic acid group
and an amino group on -carbon.
H2N  CHCOOH
R
-Amino acid
(R = H or alkyl or aryl group)

2. The reaction of the COOH group of one amino acid molecule and NH2 group of
the neighbouring amino acid molecule forms peptide having CONH linkage
by elimination of water.
R1 O H R2 O R1 R2
 H2 O
H2N  CH C OH + H N  CH  C  OH 
 H2N  CH CO  NH  CH  COOH
-Amino acid -Amino acid Peptide linkage
(Dipeptide)
3. Above reaction repeats itself to give tri, tetra, penta and finally polypeptides (i.e.,
protein).
(Explanation + Reaction) [2]
Q.8. Answer any ONE of the following:
i. a. 1. Nitroethane reduced to ethylamine by the action of tin and conc. HCl.
CH3CH2NO2 + 6[H]  Sn /conc.HCl
 CH3CH2NH2 + 2H2O [1]
Nitroethane Ethylamine

2. When nitroethane is reduced in a neutral medium with metal, it forms


N-ethylhydroxylamine.
CH3CH2NO2 + 4[H]  Zn/NH 4Cl
 CH3CH2NHOH + H2O [1]
Nitroethane N-Ethylhydroxylamine
3. Nitroethane undergoes hydrolysis on boiling with hydrochloric acid to give acetic
alongwith hydroxylamine.
CH3CH2NO2 + H2O  HCl
 CH3COOH + NH2OH [1]

Nitroethane Acetic acid Hydroxylamine

11
Board Answer Paper : July 2017
b. Preparation of Buna-N:
Name of monomers Formula
1. Buta-1,3-diene CH2 = CH  CH = CH2
2. Acrylonitrile CH2 = CH  CN
(Names) [½  2] [1]
(Formulae) [½  2] [1]
c. Soaps are sodium or potassium salts of higher fatty acids which contain more than 12
carbon atoms. [1]
Preparation of soaps:
1. By hydrolysis of fat using sodium or potassium hydroxide solution
(saponification):
When oils and fats are heated with solution of sodium hydroxide or potassium
hydroxide, they are hydrolysed to glycerol and sodium (or potassium) salt of fatty acid
(known as soap). This process is called saponification.
O
CH2  O  C  R
O CH2OH O
 

CH  O  C  R + 3NaOH   CHOH + 3RC ONa
Soap
O CH2OH
CH2  O  C  R Glycerol
Fat/oil
After saponification, common salt (NaCl) is added to the aqueous solution. This
decreases the solubility of soap due to which it precipitates. Soap being lighter,
floats on the surface of water and can be separated.
2. Direct neutralization of fatty acids: Soaps are prepared by direct neutralization of
fatty acids by using sodium hydroxide or sodium carbonate.
O O
 
RCOH + NaOH  RC ONa + H2O
Higher fatty acid Soap (Any one method) [1]
ii. a. 1. Methanal reacts with methyl magnesium iodide in presence of dry ether to give
an addition compound, which on acid hydrolysis gives ethanol.

H H
H /H  OH  I
H  C = O + CH3  Mg  I 
dry ether
 CH3  C  O  Mg  I   CH3  CH2  OH + Mg
Methanal Methyl Ethanol OH
magnesium H
iodide Addition compound [1]

2. Ethanal reacts with methyl magnesium iodide in presence of dry ether to give an
addition compound, complex which on acid hydrolysis gives propan-2-ol.
CH3 CH3
H /H  OH  I
CH3  C = O + CH3  Mg  I 
dry ether
 CH3  C  O  Mg  I   CH3  CH  OH + Mg
Methyl Propan-2-ol OH
H magnesium H
Ethanal iodide Addition compound [1]
12
Chemistry
3. Propanone reacts with methyl magnesium iodide in presence of dry ether to give
an addition compound, which on acid hydrolysis gives 2-methylpropan-2-ol.

CH3 CH3 CH3


H  /H  OH I
CH3  C = O + CH3  Mg  I 
dry ether
 CH3  C  O  Mg  I 
 CH3  C  OH + Mg
Propanone Methyl OH
magnesium CH3 CH3
iodide Addition compound 2-Methylpropan-2-ol [1]

b. Optical activity (Chirality): The property of a substance to rotate the plane of a plane
polarised light towards the right (clockwise) or towards the left (anticlockwise) is
called optical activity. [1]
c. The optical activity of lactic acid can be discussed as:
1. Presence of asymmetrical carbon atom:
i. Lactic acid contains one asymmetrical carbon atom.
(Number of asymmetric carbon atom) [½]
ii. According to van’t Hoff’s rule: a = 2n, where, a is the number of isomers and n is
the number of asymmetric carbon atom.
iii. Thus, two isomers of lactic acid are possible. (Number of isomers) [½]
2. Non-superimposable mirror image structures:

COOH COOH

H  C  OH HO  C*  H

CH3 CH3
d or (+) lactic acid
Mirror
 or () lactic acid (Two enantiomer structures) [1]

Hence, lactic acid can exist as d-form and l-form which are non-superimposable
mirror images of each other.
(Mentioning ‘d’ and ‘l’ forms) [½]
3. (dl) Racemic mixture and its optical inactivity:
A mixture containing equal moles of the d and l forms of lactic acid is a racemic
mixture which is optically inactive i.e., dl or () form. This inactivity arises due
to external compensation.
(Explanation of racemic mixture) [½]

13
Mathematics

BOARD ANSWER PAPER : MARCH 2016


MATHEMATICS AND STATISTICS
SECTION – I
Q.1. (A) Select and write the most appropriate answer from the given alternatives in each of the
following sub-questions:
i. (D)
~[p  (q  r)]
≡ ~p  ~(q  r) ….(Negation of conjunction)
≡ ~p  (q  ~r) .…(Negation of implication) [2]
ii. (C)

sin1 (1  x)  2 sin1 x =
2
Let x = sin 

 sin1 (1  sin )  2 sin1 (sin ) =
2

 sin1 (1  sin )  2 =
2

 sin1 (1  sin ) = + 2
2
 
 1  sin  = sin   2  
2 
  
 1  sin  = cos 2 …  sin  
    cos  
 2  
2
 1  sin  = 1  2sin 
 2sin2   sin  = 0
 sin  (2 sin   1) = 0
1
 sin  = 0 or sin  =
2
1
 x = 0 or x =
2
1  1 1
For x = , sin1 (1  x)  2sin1 x = sin1  1    2 sin1  
2  2 2
1 1 1  
= sin1    2 sin1   =  sin1   =  
2 2 2 6 2
1
 x
2
 x=0 [2]
iii. (A)
Equations of the co-ordinate axes are x = 0 and y = 0.
Equations of the lines passing through the point (2, 3) and parallel to the co-ordinate axes are
x = 2 and y = 3
i.e., x  2 = 0 and y  3 = 0.
 the joint equation of these lines is
(x  2) (y  3) = 0
 xy – 3x – 2y + 6 = 0 [2]
1
Board Answer Paper : March 2016
(B) Attempt any THREE of the following:
1 1
1 2 3   6 3
i. AB =   1 2  =   [1]
1  2  3  4 1 
1 2 
6 3
Consider, AB = = (6  1) – (–4  –3) = 6 – 12 =  6 ≠ 0
4 1
 AB is a non-singular matrix
1 1 1 3
 (AB)1 = (Adj AB) = [1]
AB 6  4 6 

ii. Let a = 3iˆ  2jˆ + kˆ , n = 4iˆ + 3jˆ + 2kˆ


Vector equation of the plane passing through the point A  a  and perpendicular to the vector
n is r . n = a . n [1]
     
r . 4iˆ  3jˆ  2kˆ = 3iˆ  2ˆj  kˆ . 4iˆ  3jˆ  2kˆ 
= 3(4)  2(3) + 1(2)
= 12  6 + 2
 r .  4i  3j  2k  = 8
ˆ ˆ ˆ [1]
Which is the vector equation of the plane
iii. Given, p = ˆi  2ˆj  kˆ and q = ˆi  4ˆj  2kˆ
Since R  r  , divides the line segment PQ internally in the ratio 2 : 1,

 r =
 
2 q 1 p
2 1

=
  
2 ˆi  4ˆj  2kˆ  ˆi  2ˆj  kˆ  [1]
3
2i  8 j  4k  ˆi  2ˆj  kˆ
ˆ ˆ ˆ
=
3
=
3

1 ˆ ˆ ˆ
3i  6 j  3k 
 r = ˆi  2ˆj  kˆ [1]

iv. Given equation is 6x2 + kxy + y2 = 0.


Its auxiliary equation is m2 + km + 6 = 0
Since, 2x + y = 0 is one of the lines given by 6x2 + kxy + y2 = 0
Slope of the line 2x + y = 0 is 2.
 2 is one of the roots of the auxiliary equation m2 + km + 6 = 0 [1]
 (2)2 + k(2) + 6 = 0
 4  2k + 6 = 0
 2k = 10
 k=5 [1]
v. Direction ratios of the given lines
are 3, 2k, 2 and 3k, 1, 5
Since, the lines are at right angles
 a1a2 + b1b2 + c1c2 = 0
 (3)(3k) + (2k)(1) + (2) (5) = 0 [1]
 9k + 2k  10 = 0
10
 k= [1]
7

2
Mathematics
Q.2. (A) Attempt any TWO of the following:
i. [(p  q)  q]  p

1 2 3 4 5
P q pq (p  q)  q [(p  q)  q]  p
T T T T T
T F F F T
F T T T F
F F T F T

In the above truth table, the entries in the last column are a combination of T and F.
 [(p  q)  q]  p is contingency [1]
[one mark each for column 4 and column 5]

ii. Consider  ABC.


Let P, Q, R be the midpoints of the sides BC, CA, AB respectively.
Let the medians BQ and CR intersect at G.
To prove that the third median AP also passes through G.
Let a , b , c , p , q , r , g be the position vectors of the points A, B, C, P, Q, R, G
respectively.
Since P, Q, R are the mid-points of the sides BC, CA, AB respectively
A

R Q
G

B C
P
 By midpoint formula, we get
bc
p= ….(i)
2
ca
q= ….(ii) [1]
2
ab
r= ….(iii)
2
From (i), (ii) and (iii), we get
2p = b + c  2p + a = a + b + c
2q = c + a  2q + b = a + b + c
2r = a + b  2r + c = a + b + c
2p  a 2q  b 2r  c a bc
 = = =
3 3 3 3
2p  a 2q  b 2r  c a bc
 = = = = g (say) [1]
2 1 2 1 2 1 3

This shows that the point G whose position vector is g lies on the three medians AP, BQ, CR
dividing them internally in the ratio 2:1.
Hence, the three medians are concurrent. [1]
3
Board Answer Paper : March 2016
iii. The shortest distance between the lines
r  a1   b1 and r  a 2   b2 is

d
a 2 
 a1  b1  b2 
b1  b2

Given lines are r   4 ˆi  ˆj   (iˆ  2ˆj  3k)


ˆ

and r  (iˆ  ˆj  2k)


ˆ  (iˆ  4 ˆj  5k)
ˆ

Here, a1  4iˆ  ˆj , b1  ˆi  2ˆj  3k,


ˆ

a 2  ˆi  ˆj  2kˆ and b2  ˆi  4ˆj  5kˆ


Now, a 2  a1  (iˆ  ˆj  2 k)
ˆ  (4 ˆi  ˆj)

  3iˆ  2 kˆ
ˆi ˆj kˆ
and b1  b2  1 2 3
1 4 5

 ˆi ( 10  12)  ˆj(  5  3)  kˆ (4  2)


 2 ˆi  2 ˆj  2 kˆ

 b1  b 2 = 22  22  22

= 444
= 12
= 2 3 [1]
Consider,  a 2  a1    b1  b2    3iˆ  2kˆ    2iˆ  2ˆj  2kˆ 
=32+02+22
=6+0+4
=2 [1]
 shortest distance between the given lines
2 1
  units [1]
2 3 3

(B) Attempt any TWO of the following:


C C
i. Consider L.H.S. = (a  b)2 cos2 + (a + b)2 sin2
2 2
C C
= (a2 + b2  2ab) cos2 + (a2 + b2 + 2ab) sin2 [1]
2 2
2 2 2 C 2 C 2 2 2 C C
= (a + b ) cos  2ab cos + (a + b ) sin + 2ab sin2
2 2 2 2

= (a2 + b2)  cos2  sin 2   2ab  cos2  sin 2 


C C C C
[1]
 2 2  2 2
= (a2 + b2)(1)  2ab cos C ….[ cos2   sin2  = cos 2] [1]
2 2
= a + b  2ab cos C
= c2 ….[By cosine rule]
= R.H.S. [1]
C C
 (a  b)2 cos2 + (a + b)2 sin2 = c2
2 2

4
Mathematics
ii. To draw the feasible region, construct table as follows:

Inequality 2x + y  7 2x + 3y  15 y3
Corresponding equation (of line) 2x + y = 7 2x + 3y = 15 y = 3
Intersection of line with X-axis 7   15 
 ,0   ,0  
2  2 
Intersection of line with Y-axis (0, 7) (0, 5) (0, 3)
Region Non-origin side Origin side Origin side
Y

5
x=3
4 C(3/2,4)
[1]
3 B(3, 3)

1 A(3, 1)

X
O 1 2 3 4 5 6 7 8 9

2x + y = 7 2x + 3y = 15
Y

Shaded portion ABC is the feasible region whose vertices are A, B and C.
A is the point of intersection of the lines x = 3 and 2x + y = 7.
Putting x = 3 in 2x + y = 7, we get
2(3) + y = 7
 y=1
 A  (3, 1)
B is the point of intersection of the lines x = 3 and 2x + 3y = 15.
Putting x = 3 in 2x + 3y = 15, we get
2(3) + 3y = 15
 x=3
 B  (3, 3)
C is the point of intersection of the lines 2x + y = 7 and 2x + 3y = 15.
Solving both equations ,we get
C(3/2 ,4)
 A  (3, 1), B  (3, 3) and C (3/2 ,4) [1]
Here, the objective function is
Z = 4x + 5y,
5
Board Answer Paper : March 2016
 Z at A(3, 1) = 4(3) + 5(1) = 17
Z at B (3, 3) = = 4(3) + 5(3) = 27 [1]
Z at C(3/2 ,4) = 4(3/2) + 5(4) = 26
 Z has minimum value 17 at A(3 ,1)
 Z is minimum, when x = 3 and y = 1. [1]
iii. Let the cost of 1 dozen pencils, 1 dozen pens and 1 dozen erasers be ` x, ` y and ` z
respectively.
According to the given conditions,
4x + 3y + 2z = 60
2x + 4y + 6z = 90 i.e. x + 2y + 3z = 45
6x + 2y + 3z = 70 [1]
Matrix form of the given system of equations is,
é 4 3 2ù é xù é60ù
ê ú ê ú ê ú
ê 1 2 3ú ê y ú = ê 45ú [1]
ê ú ê ú ê ú
ê 6 2 3ú êzú ê70ú
ë û ë û ë û
Applying R1  R2
é 1 2 3ù é x ù é 45ù
ê ú ê ú ê ú
ê 4 3 2ú ê y ú = ê60ú
ê ú ê ú ê ú
ê 6 2 3ú ê z ú ê70ú
ë û ë û ë û
Applying R2  R2  4R1, R3  R3  6R1
é1 2 3ù é xù é 45ù
ê ú ê ú ê ú
ê0 - 5 - 10ú ê y ú = ê-120ú
ê ú ê ú ê ú
ê0 - 10 -15 ú êzú ê-200ú
ë û ë û ë û
Applying R3  R3  2R2,
é1 2 3ù é xù é 45ù
ê ú ê ú ê ú
ê0 - 5 - 10ú ê y ú = ê-120ú
ê ú ê ú ê ú
ê0 0 5 úû êzú ê 40ú
ë ë û ë û
Hence, the original matrix is reduced to an upper triangular matrix.
 By equality of matrices, we get

x + 2y + 3z = 45 ....(i)
5y  10z = 120
i.e. y + 2z = 24 ....(ii) [1]
5z = 40 ....(iii)
i.e. z=8
Substituting z = 8 in equation (ii), we get
y + 2(8) = 24
 y=8
Substituting z = 8 and y = 8 in equation (i), we get
x + 2(8) + 3(8) = 45
 x + 16 + 24 = 45
 x=5
 x = 5, y = 8, z = 8 [1]
Thus, the cost of pencils is ` 5 per dozen, that of pens is ` 8 per dozen and that of erasers is ` 8
per dozen.
6
Mathematics
Q.3. (A) Attempt any TWO of the following:
i. Let a = 7iˆ  kˆ , b  2iˆ  5jˆ  3kˆ , c  4iˆ  3jˆ  kˆ be the coterminus edges of a tetrahedron.
1
Volume of the tetrahedron =  a b c  [1]
6
7 0 1
1
= 2 5 3 [1]
6
4 3 1
1
= 7(5  9)  0 1(6  20)
6
1
= (98  14)
6
1
= (84) = 14
6
 Volume of the tetrahedron is 14 cubic units. [1]

ii. Consider, LHS = (p  q)  (p  q)


 (p  q)  (p  q) …. (Negation of disjunction) [1]
 p  (q  q) ….(Distributive law) [1]
 p  T ....(Complement law)
p ....(Identity law) [1]
= RHS.
 (p  q)  (p  q)  p

iii. Let ax2 + 2hxy + by2 = 0 ….(i)


be a homogeneous equation of degree 2 in x and y.
Case I:
If b = 0 (i.e., a  0, h  0), then the equation (i) reduces to ax2 + 2hxy = 0
i.e., x(ax + 2hy) = 0
This represents two lines, x = 0 and ax + 2hy = 0, both passing through the origin.
Case II:
If a = 0 and b = 0 (i.e., h  0), then the equation (i) reduces to 2hxy = 0, i.e., xy = 0 which
represents the coordinate axes and they pass through the origin. [1]
Case III:
If b  0,
Multiplying both sides of equation (i) by b, we get
abx2 + 2hbxy + b2y2 = 0
 b2y2 + 2hbxy = abx2
To make L.H.S. a complete square, we add h2x2 on both the sides.
 b2y2 + 2hbxy + h2x2 = abx2 + h2x2
 (by + hx)2 = (h2  ab)x2

 
2
 (by + hx)2 =  h 2  ab x 

(by + hx)   h  ab  x  = 0


2
2
 2
 

   2
  
 by  hx   h  ab x   (by  hx ) 
 
h 2  ab x  = 0

[1]

7
Board Answer Paper : March 2016
This is the joint equation of two lines
(by + hx) +  
h 2  ab x = 0

and (by + hx)   h  ab  x = 0


2


i.e., h  h 2  ab  x + by = 0

and  h  h2  ab  x + by = 0

These lines pass through the origin.


From the above three cases, we conclude that the equation ax2 + 2hxy + by2 = 0 represents a
pair of lines passing through the origin, if h2  ab  0. [1]
(B) Attempt any TWO of the following:
i. Let AB be a line drawn from the point A(1, 2, 1), which is perpendicular to the line joining
P(1, 4, 6) and Q(5, 4, 4).
Let B divides PQ internally in the ratio  : 1.
5  1 4  4 4  6 
 B   , ,  ….(i) [1]
  1  1  1 
Direction ratios of AB are
5  1 4  4 4  6
 1,  2, 1
 1  1  1
4 2  2 3  5
i.e., , ,
 1  1  1
Now, direction ratios of PQ are 5  1, 4  4, 4  6 i.e., 4, 0, –2.
Since AB is perpendicular to PQ
 4   2  2   3  5 
 4   0   2  0 [1]
  1   1    1 
16  6  10
 0
 1
 10  = 10
 =1 [1]
Putting  = 1 in (i), we get
B   , , 
6 8 10
2 2 2 
 B  (3, 4, 5)
 Coordinates of the required foot of the perpendicular are (3, 4, 5). [1]

ii. Let a, b, c be the position vectors of the points A, B, C respectively.


 a = î + ĵ  2 k̂ , b = î + 2 ĵ + k̂ , c = 2 î  ĵ + k̂

B C

8
Mathematics

Now, AB = b  a = ĵ + 3 k̂
AC = c  a = î  2 ĵ + 3 k̂
n is perpendicular to AB and AC both.
 n = AB  AC
ˆi ˆj kˆ
= 0 1 3
1 -2 3

= î (3 + 6)  ĵ (0  3) + k̂ (0  1)
= 9 î + 3 ĵ  k̂ [1]
Vector equation of the plane is r.n  a.n
 r   9iˆ + 3jˆ  kˆ  =  ˆi + ˆj  kˆ    9iˆ + 3jˆ  kˆ  [1]
= 1(9) + 1(3)  2(1)
=9+3+2
 r   9iˆ + 3jˆ  kˆ  = 14 [1]
for cartesian form,
putting r = xˆi + yˆj+ zkˆ in (i), we get
 xˆi + yˆj+ zkˆ  9iˆ  3jˆ  kˆ  = 14
9x + 3y  z = 14 [1]
Which is the cartesian form of equation
iii. sin x + sin 3x + sin 5x = 0
 (sin 5x + sin x) + sin 3x = 0
5x  x   5x  x 
 2sin   cos   + sin 3x = 0 [1]
 2   2 
 2 sin 3x cos 2x + sin 3x = 0
 sin 3x (2cos 2x + 1) = 0
 sin 3x = 0 or 2cos 2x + 1 = 0
1
 sin 3x = 0 or 2cos 2x =  [1]
2
  
 sin 3x = 0 or cos 2x =  cos = cos    
3  3
2
 sin 3x = 0 or cos 2x = cos [1]
3
Since, sin  = 0 implies  = n and cos  = cos  implies  = 2n   , n  Z.
2
 3x = n or 2x = 2m 
3
n 
 the required general solution is x = or x = m  , where n, m  Z. [1]
3 3
SECTION – II
Q.4. (A) Select and write the most appropriate answer from the given alternatives in each of the
following sub-questions:
i. (C)
f(x) is continuous at x = 1 ….(given)
 lim f  x   lim f(x)
x 1 x 1

9
Board Answer Paper : March 2016

 lim  k  x   lim  4 x + 3 
x 1 x 1

 k +1 = 4(1)+3
 k=6 [2]

ii. (A)
Equation of the curve is y = x2 + 4x + 1
Differentiating w.r.t. x, we get
dy
= 2x + 4
dx
 Slope of tangent at (1, 2) is
 dy 
  = 2(1) + 4 = 2 + 4 = 2
 dx ( 1,  2)
 dy 
Equation of tangent is y  y1 =   (x  x1)
 dx ( x1 , y1 )
Here, (x1, y1)  (1, 2)
 [ y  (2)] = 2[x  (1)]
 y + 2 = 2(x + 1) = 2x + 2
 2x  y = 0 [2]

iii. (C)
n = 10, E(X) = 8 ....(given)
But, E(X) = np
 8 = 10 (p)
8 4
 p=   0.8 [2]
10 5

(B) Attempt any THREE of the following:


i. y = xx
Taking logarithm on both sides, we get
log y = x log x
Differentiating w.r.t. x, we get
1 dy 1
 = x + log x1 [1]
y dx x
dy
 = y (1 + log x) = xx (1 + log x) [1]
dx
ii. Let s = 5 + 20t  2t2
Differentiating w.r.t. t, we get
ds
= 20  4t
dt
ds
v= = 20  4t ....(i) [1]
dt
Given, v = 0
 20  4t = 0
t=5
10
Mathematics
Differentiate equation (i) by t, we get
d 2s dv
a= 2  =04
dt dt
=4
 d 2s  2
  2  =  4 unit  sec [1]
 dt  t 5
 The acceleration is  4 unit  sec2

iii. Given equation of parabola is y2 = 4ax


 y=2 a x

Y
y2 = 4ax
A

X X
O S(a, 0)

Y

 required area = area of the region OBSAO


= 2(area of the region OSAO)
a


= 2 y dx
0
a


= 2 2 a x dx
0
[1]

=4 a  0
x dx

é2 3 a
ù
= 4 a êê x 2 úú
3 ë û0
8 é ù 8 2 3
= a êa - 0ú = a sq. units
2
[1]
3 ë ê ú 3
û
n
iv. Given,  Pi = 1
i 1
 k + 2k + 3k + 4k + 5k = 1
 15k = 1
1
 k= [1]
15
X=x 1 2 3 4 5
P(X = x) 1 2 3 4 5
15 15 15 15 15

11
Board Answer Paper : March 2016
 P(x ≤ 4) = P(x = 1) + P(x = 2) + P(x = 3) + P(x = 4)
1 2 3 4
=   
15 15 15 15
10
=
15
2
= [1]
3

sin x
v. Let I =  36  cos2 x
dx

Put cos x = t
Differentiating w.r.t.x, we get
sin x dx =  dt [1]
dt
 I=  62  t 2
 t 
=  sin 1    + c
 6 

I =  sin1 
cos x 
  +c [1]
 6 

Q.5. (A) Attempt any TWO of the following:


i. Let x be a small increment in the value of x.
Since u is a function of x, there should be a corresponding increment u in the value of u.
Also y is a function of u.
 there should be a corresponding increment y in the value of y.
y u
Consider, y = 
x u x
Taking lim on both sides, we get
x  0

y y u
lim = lim  lim ….[ x  0, u  0] [1]
x 0 x x 0 u x  0 x

 y = lim y  lim u ….(i)


lim
x  0 x u  0 u x  0 x

du
But, lim u = exists and is finite.
x 0 x dx
dy
Also, lim y = exists and is finite.
u  0 u du
 limits on R.H.S. of (i) exist and are finite. [1]
Hence, limits on L.H.S. should also exist and be finite.
y
dy
 lim =
exists and is finite.
x  0 x
dx
dy dy du
 =  [1]
dx du dx

ii. Since, there are six patients


n=6
p = P(success) = 0.5 and q = 1  p = 0.5
X  B(n = 6, P = 0.5)
The p.m.f. of x is given as P(X = x) = nCx(P)x (q)nx [1]
P(X = x) = P(x) = 6Cx (0.5)x (0.5)6x,
x = 0, 1, 2, 3, 4, 5, 6

12
Mathematics
a. P(none will recover)
P(X = 0) = 6C0 (0.5)0 (0.5)60
= (1) (1) (0.5)6 [1]
= 0.015625
b. P(half of them will recover)
P(X = 3) = 6C3 (0.5)3 (0.5)63
6!
= (0.5)3 (0.5)3
3!.3!
6  5  4  3!
= (0.125) (0.125)
3 2 1 3!
= 20  0.015625
= 0.3125 [1]

x
iii. Let I = a
0
2
cos2 x  b2 sin 2 x
dx ....(i)


 x  a a

 I=  dx ....   f  x  dx   f  a  x  dx  [1]
0
a 2 cos2    x   b 2 sin 2    x   0 0 

 x
 I= a0
2
cos2 x  b2 sin 2 x
dx ....(ii)

Adding (i) and (ii), we get



x x
2I = a
0
2
cos2 x  b 2 sin 2 x
dx


1
= a
0
2
cos2 x  b2 sin 2 x
dx

Dividing Nr and Dr by cos2x, we get



sec 2 x
2I =   dx
0
a  b2 tan 2 x
2


2
 sec2 x sec 2 (   x )   2a a

=  2 2 2  2 2 2 dx ….   f ( x )dx   f ( x )  f (2a  x ) dx 
0
a  b tan x a  b tan (   x )   0 0 

2
 sec2 x sec 2 x 
=    2 2 2  2 2 2  dx
0
a  b tan x a  b tan x 

2
sec 2 x
= 2 dx
0
a  b 2 tan 2 x
2


2
sec 2 x
 I =  dx [1]
0
a 2  b2 tan 2 x

Put tan x = t
 sec2 x dx = dt

When x = 0, t = 0 and when x = ,t=
2

13
Board Answer Paper : March 2016
 
dt  dt
 I=  a 2
 b 2 2
t
= 2
b a 2

  t
0 0 2

b

  
 1  1  t  
= 2   tan  a  
b a   
  
b   b   0

  1  bt  
=  tan  a  
ab  0

= (tan1   tan 1 0)
ab
  
=   0
ab  2 
2
 I= [1]
2ab

(B) Attempt any TWO of the following:


f(0) = log  
2
i. ….(given)
 3
(4 x  1)  (e x  1)
x
4 e 4 1 e 1
x x
x
x
lim f(x) = lim = lim = lim
x 0 x 0 6x  1 x 0 6x  1 x 0 6x  1
x
4x  1 ex  1

= lim x x x [1]
x 0 6 1
x
4x  1 ex  1
lim  lim
=
x 0 x x 0 x
6x  1
lim
x 0 x
log 4  log e
= [1]
log 6

4
log  
= e
log 6

 lim f(x)  f(0)


x 0

 f is discontinuous at x = 0. [1]
The discontinuity of f is removable and it can be made continuous by redefining the function
as
x x
4 e
f(x) = x
, for x  0
6 1
[1]
4 at x = 0
log  
= e , for x = 0
log 6

14
Mathematics

ii. Let I =  a 2  x 2  1dx

= a 2  x 2  1dx   
d
 dx
  
a 2  x 2   1dx  dx
 [1]
2 x
= x  a2  x2    x dx
2 a2  x2
(a 2  x 2 )  a 2
= x  a2  x2   dx
a2  x2
 a 2  x2  a2
= x  a2  x2    
 dx
 a x a x 
2 2 2 2

1
= x  a 2  x 2   a 2  x 2 dx + a 2  2 2 dx
a x [1]
I = x  a 2  x 2  I + a 2 sin 1   + c1
x

a [1]
2I = x  a 2  x 2 + a 2 sin 1   + c1
x

a
2
x 2 a x c
 I= a  x 2 + sin 1   + 1
2 2 a 2
x 2 a2 x
  a 2  x 2 dx =
2
a  x 2  sin 1    c ,
2 a
[1]

c1
where c =
2

iii. Let  be the temperature of the body at time t. Temperature of air is 10C.
According to the Newton’s law of cooling we have.
d
 (  0)
dt
d
= K (  0), k > 0 [1]
dt
 = 0 + c ekt
Using initial condition
 = 10 + 100 ekt [1]
But  = 60c, t = 1 hour
1
ek = [1]
2
we have to find t when  = 35c
t

35 = 10 + 100  
1
2
t

=  
1 1
4 2
2 t
1 1
  =  
2 2
t = 2 hours
Hence additional time required to cool the body form 60C to 35C is equal to
= Time required to cool the body to 35 c Time required to cool body to 60 c
= 2  1 = 1 hour [1]
15
Board Answer Paper : March 2016
Q.6. (A) Attempt any TWO of the following:
a a
i. Let I =  f ( x )dx  f (2a  x )dx
0 0

= I1 + I2
For I2, put 2a – x = t
 dx = dt
 dx = dt
When x = 0, t = 2a and when x = a, t = a [1]
a a
 I2 =  f (2a  x )dx =   f  t  dt
0 2a

 I = I1 + I2
a a
=  f ( x )dx   f (t)dt
0 2a
a 2a
 b a

=  f ( x )dx  f (t)dt ....   f ( x )dx   f ( x )dx 
0 a  a b 
[1]
a 2a
 b b

=  f ( x )dx   f ( x )dx ….   f ( x )dx f (t)dt 
0 a  a a 
2a
 b c b

=  f ( x )dx ….   f  x  dx   f  x  dx   f  x  dx;a  c  b  [1]
0  a a c 
2a a a
  f ( x )dx f ( x )dx   f (2a  x )dx
0 0 0

1  log x
ii. Let I =  x  2  log x 3  log x  dx
1  log x
=  x 1  1  log x  2  1  log x  dx

Put 1 + log x = t
1
 dx = dt
x
t dt
 I= 
1  t  2  t 
[1]

t A B
Let = +
1  t  2  t  1 t 2t
 t = A(2 + t) + B(1 + t) ….(i)
Putting t = 1 in (i), we get
 1 = A(2  1)
 A = 1
Putting t = 2 in (i), we get
 2 = B(1  2)
 B=2 [1]
t 1 2
 = +
1  t  2  t  1 t 2t
1 2
 I= 1 t dt + 2t dt
= log |1 + t| + 2log |2 + t| + c
= log |1 + 1 + log x| + 2log |2 + 1 + log x| + c
16
Mathematics
= log |2 + log x| + 2log |3 + log x| + c
 I = log |(3 + log x)2 |  log | (2 + log x) | + c [1]

iii. Let y = cos1 2 x 1  x 2  


Put x = sin 
  = sin1 x
y = cos1  2sinθ 1 sin 2θ  [1]
 
= cos1 [2 sin  cos ]
= cos1 [sin 2 ]
 
= cos1 cos   2θ  
π
 2  

y =  2
2

y =  2 sin1 x [1]
2
differentiating w.r.t.x, we get
dy 1 2
= 0 2 = [1]
dx 1 x 2
1 x 2

(B) Attempt any TWO of the following:


dy
i. cos (x + y) =1 ....(i)
dx
put x + y = v
differentiating, we get
dy dv
1+ = [1]
dx dx
 equation (i) becomes
cos v 
dv 
1 = 1
 dx 
dv
 cos v  cos v = 1
dx
dv
 cos v = 1 + cos v
dx
cos v
 dv = dx [1]
1  cos v
Intergrating both sides, we get
cos v
 1 cos v dv =  dx
1 cos v 1
  dv   dx
1 cos v
 1 
  1 1 cos v  dv =  dx
1
 1.dv   1 cos v dv   dx
1
1.dv   v
dv   dx
2cos 2
2

17
Board Answer Paper : March 2016

v  tan   = x + c
v
[1]
2  
 x+ y 
x + y  tan   =x+c
 2 

y = tan 
x+ y 
 +c
 2 
This is general solution of different equation.
Now x = 0, y = 0 then c = 0
y = tan 
x+ y 
  is particular solution. [1]
 2 

ii. Let r be the radius of the circle and x be the length of the side of the square.
Then,
circumference of the circle + perimeter of the square = total length of the wire
 2r + 4x = l
 2r = l  4x
l  4x
 r= ….(i)
2
Let A = area of the circle + area of the square
= r2 + x2
2
l  4x 
=    +x
2
….[From (i)] [1]
 2 
l 2  8lx  16 x 2
=  x2
4
4x 2  16 x 2  8lx  l 2
=
4
4(  4) x 2  8lx  l 2
=
4
dA 8(   4) x  8l
 =
dx 4
d2A 8(   4) 2(   4)
and = = [1]
dx 2
4 
dA
Now, A is minimum, if =0
dx
8(   4) x  8l
i.e., if =0
4
i.e., if 8( + 4) x  8l = 0
i.e., if 8( + 4) x = 8l
8l l
i.e., if x = = [1]
8(   4) 4
 d2A  2(   4) l
Also,  2  = > 0 and hence A is minimum when x = .
 dx  x  l  4
 4

From (i),
 l 
l  4 
r =    4  = l  4 l  4l
2 2    4 
l
=
2    4 

18
Mathematics
l
=
2    4
x
=
2
Thus, the sum of the areas of the circle and the square is the least, when radius of the circle is
half side of the square. [1]
iii. c.d.f. of a continuous random variable x is given by
x
F (x) =  f  y  dy

[1]

a. In the given example, range of x


x
F(x) =  f  y  dy
0
x
y
=  32
0
dy
x
 y2  x2
F(x) =   = xR [1]
 64  0 64
b. At x = 0.5 we can write
 0.5
2
1
F(x) = F(0.5) = = [1]
64 256
At x = 9
For any value of x ≥ 8
F(x) = 1 [1]

19
Mathematics

BOARD ANSWER PAPER : JULY 2016


MATHEMATICS AND STATISTICS
SECTION – I
Q.1. (A) Select and write the most appropriate answer from the given alternatives in each of the
following sub-questions:
i. (D)
(p  q)  ( p  q)
Its inverse is
~ (p  q )  ~ ( p  q)
≡ (~p  ~q) (~p  ~q) [2]
ii. (C)
Let a = 2 î  q ĵ + 3 k̂ and b = 4 î  5 ĵ + 6 k̂ .
Since, a and b are collinear.
 there exists a scalar t such that b = t a .
 4 î  5 ĵ + 6 k̂ = t(2 î  q ĵ + 3 k̂ ) = 2t î  qt ĵ + 3t k̂
 By equality of vectors, we get
4 = 2t,  5 =  qt, 6 = 3t
 4 = 2t and 6 = 3t  t=2
  5 =  q(2)
 –5 = – 2q  5 = 2q
5
 q= [2]
2
iii. (B)
a  b  c 18  24  30
s=   36
2 2

sin
A
=
 s  b  s  c  = (36  24)(36  30)
=
12  6
=
1
[2]
2 bc 24  30 24  30 10
(B) Attempt any THREE of the following:
i. Let b1 and b 2 be the vectors in the direction of the lines
r = (3iˆ  2ˆj  4k)
ˆ +  (iˆ  2ˆj  2k)
ˆ and r = (5iˆ  2k)
ˆ +  (3iˆ  2ˆj  6k)
ˆ respectively.

 b1 = ˆi  2ˆj  2kˆ and b 2 = 3iˆ  2ˆj  6kˆ


 b1  b 2 = (iˆ  2ˆj  2k)
ˆ (3iˆ  2ˆj  6k)
ˆ
=13+22+26
= 3 + 4 + 12
 b1  b 2 = 19 [1]
Consider,
b1 = (1) 2  (2) 2  (2) 2 = 1  4  4 = 9 = 3

and b 2 = (3) 2  (2)2  (6)2 = 9  4  36 = 49 = 7


Let  be the acute angle between the two given lines.
b1  b 2 19
 cos  = =
b1 b 2 3 7

1
Board Answer Paper : July 2016
19
 cos  =
21

 = cos1  
19
 [1]
 21 
ii. (r  q)  p
 (T  F)  T
 (T  F)  F [1]
 FF
T
Hence, the truth value is ‘T’ [1]
 2 3
iii. Given A = 
3 5 
2 3
 A = = 10 + 9 = 19  0
3 5
 A1 exists
A11 = (1)1 + 1. M11 = 5 A12 = (1)1 + 2. M12 = 3
A21 = (1)2 + 1. M21 = –(–3) = 3 A22 = (1)2 + 2. M22 = 2
Hence, matrix of the co-factors is
A A  5 3
[Aij]2×2 =  11 12
 = 3
A
 21 A 22   2 
T 5 3
Now, adj A =  A ij  =   [1]
2 2
 3 2 
1 1  5 3
A1 =  adj A  =  [1]
A 19  3 2 

iv. Let a, b,c and d be the position vectors of vertices A, B, C, D respectively of □ABCD
 a  ˆi  2ˆj  k,
ˆ b  8iˆ  3jˆ  4kˆ , c  5iˆ  4ˆj  kˆ and d   2iˆ  ˆj  4kˆ

a  c ˆi  2ˆj  kˆ  5iˆ  4ˆj  kˆ


Let e    3iˆ  ˆj ….(i)
2 2
b  d 8iˆ  3jˆ  4kˆ  2iˆ  ˆj  4kˆ
and f    3iˆ  ˆj ….(ii) [1]
2 2
 From (i) and (ii), we get
ef
The mid point of the diagonals AC and BD is same
 The diagonals AC and BD bisect each other.
 The □ABCD is a Parallelogram. [1]
v. sin x = tan x
sin x
 sin x =
cos x
 sin x cos x  sin x = 0
 sin x (cos x  1) = 0 [1]
 sin x = 0 or cos x = 1
 sin x = sin 0 or cos x = cos 0
Since, sin  = 0 implies  = n and cos  = cos  implies  = 2n   , n  Z.
 x = n or x = 2m  0
 the required general solution is x = n or x = 2m, where n, m  Z. [1]
2
Mathematics
Q.2. (A) Attempt any TWO of the following:
i. Given equation is ax2 + 2hxy + by2 = 0,
Let m1 and m2 be the slopes of the given lines
2h a
 m1 + m2 = and m1m2 = [1]
b b
Since, the required lines are perpendicular to these lines
1 1
 slopes of the required lines are  and 
m1 m2
Required lines also pass through the origin, therefore their equations are
1 1
y=  x aqnd y =  x
m1 m2
 x + m1y = 0 and x + m2y = 0
 the joint equation of the lines is (x + m1y)(x + m2y) = 0 [1]
 x2 + (m1 + m2)xy + m1m2y2 = 0
2h 
 x2 +  a 2
 xy +   y = 0
 b  b  
2 2
 bx – 2hxy + ay = 0 [1]
 1 
ii. Let sin–1   = x
 2
1
 sin x =
2

 sin x = –sin [1]
4
    
The principal value branch of sin–1 x is   ,  and     [1]
 2 2 2 4 2

Hence, the required principal value of x is  [1]
4

iii. The equation r  a  sb  tc represents a plane passing through a point having position vector
a and parallel to the vectors b and c .
Here, a = ˆi  ˆj , b  ˆi  ˆj  2kˆ and c  ˆi  2ˆj  kˆ
The given plane is perpendicular to the vector n
ˆi ˆj kˆ
= b  c = 1 -1 2
1 2 1

= î (1  4)  ĵ (1  2) + k̂ (2 + 1)
= 5 î + ĵ + 3 k̂ [1]
Vector equation of the plane in scalar product form is r  n = a  n
 r   5iˆ + ˆj + 3kˆ  =  ˆi + ˆj   5iˆ + ˆj + 3kˆ  [1]
 r   5iˆ + ˆj + 3kˆ  = 1(5) + 1(1) + 0(3)

 r   5iˆ + ˆj + 3kˆ  = 4 ....(i)


For cartesian form,
Putting r = x î + y ĵ + z k̂ in (i), we get
 xˆi + yˆj + zkˆ    5iˆ + ˆj + 3kˆ  =4
3
Board Answer Paper : July 2016
 x(5) + y(1) + z(3) =  4
 5x + y + 3z =  4
 5x – y – 3z = 4
which is the cartesian form of the equation of the plane. [1]
(B) Attempt any TWO of the following:
i. Let p: The switch S1 is closed.
q: The switch S2 is closed.
~p: The switch S1 is closed or the switch S1 is open.
~q: The switch S2 is closed or the switch S2 is open.
 symbolic form of the given circuit is
(p  ~q)  (~p  q)  (~p  ~q) [1]
 (pq)  [p(qq)] …(Associative and Distributive law)
 (pq)  (pT) ….(Complement law)
 (pq) p ....(Identity law)
 (p  ~p)  (~q  p) ....(Distributive law) [1]
 (p  ~p)  (~p  q) ….(Commutative law)
 T(~p  ~q) ….( Complement law)
 pq ….(Identity law) [1]
The simplified circuit of the given circuit are as follows:

SS1' 1

SS'22 [1]
+  L
ii. Let , ,  be the direction angles of the line
  = 45,  = 60
1 1
 m = cos  = cos 45 = and n = cos  = cos 60 =
2 2
Since, cos2  + cos2  + cos2  = 1 [1]
2 2
 1 
cos2  +      = 1
1

 2  2
1 1 1
 cos2  = 1   
2 4 4
1
 cos  =  [1]
2
1 1 1
i.e., l =  , m = ,n=
2 2 2
The unit vectors along the direction of line are
û = lˆi  mjˆ  nkˆ
1 1 ˆ 1ˆ
=  ˆi  j k [1]
2 2 2
 The vectors of magnitude 5 are
1 1 ˆ 1 ˆ  1 1 ˆ 1 ˆ
5  ˆi  j  k  and 5   ˆi  j  k [1]
 2 2 2   2 2 2 

4
Mathematics
iii. To draw the feasible region, construct table as follows:
Inequality x + 4y  24 3x + y  21 x+y9
Corresponding equation (of line) x + 4y = 24 3x + y = 21 x+y=9
Intersection of line with X-axis (24, 0) (7, 0) (9, 0)
Intersection of line with Y-axis (0, 6) (0, 21) (0, 9)
Region Origin side Origin side Origin side

24

21
3x + y = 21
18

15

12
[1]

6 D(0, 6) C(4, 5)

3 B(6, 3)

A(7, 0)
X X
O 3 6 9 12 15 18 21 24
x + 4y = 24

x+y=9
Y

Shaded portion OABCD is the feasible region,


whose vertices are O(0, 0), A(7, 0), B, C and (0, 6)
B is the point of intersection of the lines 3x + y = 21 and x+y = 9.
Solving the above equations, we get x = 6, y = 3
 B  (6, 3)
C is the point of intersection of the lines
x + 4y = 24
and x + y = 9.
Solving the above equations, we get
x = 4, y = 5
 C  (4, 5) [1]
Here, the objective function is Z = 3x + 5y,
 Z at O(0, 0) = 3(0) + 5(0) = 0
Z at A(7, 0) = 3(7) + 5(0) = 21
Z at B(6, 3) = 3(6) + 5(3)
= 18 + 15 = 33 [1]
Z at C(4, 5) = 3(4) + 5(5)
= 12 + 25 = 37
Z at D(0, 6) = 3(0) + 5(6) = 30
 Z has maximum value 37 at C(4, 5).
 Z is maximum, when x = 4, y = 5 [1]
5
Board Answer Paper : July 2016
Q.3. (A) Attempt any TWO of the following:
i. Shortest distance between the lines
x  x1 y  y1 z  z1 x  x2 y  y2 z  z 2
  and   is
a1 b1 c1 a2 b2 c2
x2  x1 y2  y1 z 2  z1
a1 b1 c1
a2 b2 c2
d= . [1]
 b1c2  b 2c1    c1a 2  c2a1    a1b 2  a 2 b1 
2 2 2

Equations of the given lines are


x +1 y +1 z  1 x 3 y 5 z 7
  and  
7 6 1 1 2 1
Here,
x1 =  1, y1 =  1, z1 =  1, x2 = 3, y2 = 5, z2 = 7,
a1 = 7, b1 =  6, c1 = 1, a2 = 1, b2 =  2, c2 = 1
x2  x1 y2  y1 z 2  z1 4 6 8
Now, a1 b1 c1  7 6 1
a2 b2 c2 1 2 1
= 4(6 + 2)  6(7  1) + 8(14 + 6)
=  16  36  64
=  116
and  b1c 2  b 2c1    c1a 2  c2a1    a1b2  a 2 b1 
2 2 2

= (6 + 2)2 + (1  7)2 + (14 + 6)2


= 16 + 36 + 64
= 116 [1]
116
 shortest distance between the given lines =
116
= 116
= 2 29 units [1]

ii. Let p1 and p2 be the distances of points î  ĵ + 3 k̂ and 3 î + 4 ĵ + 3 k̂ from the plane
r   5iˆ  2ˆj  7kˆ  + 8 = 0
The distance of the point A with position vector a from the plane r  n = p is given by
a n  p
d=
n

(ˆi -ˆj+3kˆ )⋅ (5iˆ + 2jˆ-7kˆ )-(-8)


 p1 =
52 + 22 + (-7)2
1(5)  1(2)  3(7)  8
=
25  4  49
5  2  21 8 10 10
= = = [1]
78 78 78
( ˆ ˆ ˆ ˆ )(
3i + 4j+ 3k ⋅ 5i + 2j-7kˆ -(-8)
ˆ )
and p2 =
52 + 22 + (-7) 2
3(5)  4(2)  3( 7)  8
=
25  4  49

6
Mathematics
15  8  21 8
=
78
10
= [1]
78
 p1 = p2
Hence, points are equidistant from the plane. [1]

iii. By cosine rule, we have b2 = c2 + a2  2ca cos B


c2  a 2  b2
 cos B =
2ac
b2  c2  a 2
Similarly cos A = [1]
2bc
R.H.S. = a cos B + b cos A
c2  a 2  b2 b2  c2  a 2
= a.  b. [1]
2ac 2bc
c2  a 2  b2 b2  c2  a 2
= 
2c 2c
c2  a 2  b 2  b2  c2  a 2
=
2c
2c2
=
2c
=c
= L.H.S.
 c = a cos B + b cso A [1]

(B) Attempt any TWO of the following:


i. Given equation is px2  8xy + 3y2 + 14x + 2y + q = 0 Comparing with
ax2 + 2hxy + by2 + 2gx + 2fy + c = 0, we get
a = p, h =  4, b = 3, g = 7, f = 1, c = q. [1]
The given equation represents a pair of lines perpendicular to each other
 a+b=0
 p+3=0
 p = 3 [1]
Also, the given equation represents a pair of lines
a h g
 h b f =0 [1]
g f c
-3 -4 7
 -4 3 1 = 0
7 1 q
 3(3q  1) + 4(  4q  7) + 7( 4  21) = 0
 9q + 3  16q  28  175 = 0
 25q  200 = 0
 25q = 200
 q = 8 [1]
 p = 3 and q = 8
7
Board Answer Paper : July 2016
ii. Let the cost of 1 dozen pencils, 1 dozen pens and 1 dozen erasers be ` x, ` y and ` z
respectively.
According to the given conditions,
4x + 3y + 2z = 60
2x + 4y + 6z = 90 i.e. x + 2y + 3z = 45
6x + 2y + 3z = 70 [1]
Matrix form of the given system of equations is,
é 4 3 2ù é xù é 60ù
ê ú ê ú ê ú
ê 1 2 3ú ê y ú = ê 45ú [1]
ê ú ê ú ê ú
ê 6 2 3ú êzú ê 70ú
ë û ë û ë û
Applying R1  R2
é 1 2 3ù é x ù é 45ù
ê ú ê ú ê ú
ê 4 3 2ú ê y ú = ê 60ú
ê ú ê ú ê ú
ê 6 2 3ú ê z ú ê 70ú
ë û ë û ë û
Applying R2  R2  4R1, R3  R3  6R1
é1 2 3ù é xù é 45ù
ê ú ê ú ê ú
ê 0 - 5 -10ú ê y ú = ê-120ú
ê ú ê ú ê ú
ê 0 -10 -15 ú êzú ê-200ú
ë û ë û ë û
Applying R3  R3  2R2,
é1 2 3ù é xù é 45ù
ê ú ê ú ê ú
ê 0 - 5 -10ú ê y ú = ê-120ú [1]
ê ú ê ú ê ú
ê0 0 5 úû êzú ê 40ú
ë ë û ë û
Hence, the original matrix is reduced to an upper triangular matrix.
 By equality of matrices, we get
x + 2y + 3z = 45 ....(i)
5y  10z = 120
i.e. y + 2z = 24 ....(ii)
5z = 40 ....(iii)
i.e. z=8
Substituting z = 8 in equation (ii), we get
y + 2(8) = 24
 y=8
Substituting z = 8 and y = 8 in equation (i), we get
x + 2(8) + 3(8) = 45
 x + 16 + 24 = 45
 x=5
 x = 5, y = 8, z = 8
Thus, the cost of pencils is ` 5 per dozen, that of pens is ` 8 per dozen and that of erasers is
` 8 per dozen. [1]
iii.
A
b c a

 b
B
D
P
O c C
Let a, b and c be the position vectors of points A, B and C respectively with respect to origin
O.
8
Mathematics

Complete the parallelopiped as shown in the figure with OA , OB and OC as its coterminous
edges.
AP is a perpendicular drawn to the plane of b and c . Let  be the angle made by AP with
OA. Volume of parallelopiped = (Area of parallelogram OCDB)  (height)
Now, area of parallelogram OCDB = b  c ….(i) [1]
Height of parallelopiped = l(AP) = l(OA) cos 
= OA cos 
= a cos  ….(ii) [1]
 From (i) and (ii), we get
volume of parallelopiped = a b  c cos 
= a   bc 
 Volume of parallelopiped = a b c  [1]
Let a = 2iˆ  5jˆ  4k,b
ˆ  5iˆ  7ˆj  5kˆ and c  4iˆ  5jˆ  2kˆ

2 5 4
 a b c  = 5 7 5
 
4 5 2
= 2(–14 – 25) – 5(–10 – 20) – 4(25 – 28)
= –78 + 150 + 12 = 84
 Volume of the parallelopiped is 84 cubic units. [1]

SECTION – II
Q.4. (A) Select and write the most appropriate answer from the given alternatives in each of the
following sub-questions:
i. (A)
7
  dy  3  3 d2 y
1 +    = 7 2
  dx   dx
7 3
  dy 3   d2 y 
Cubing on both sides, we get 1    = 73  2 
  dx    dx 
By definition of degree and order Degree: 3 ; Order: 2 [2]
ii. (C)
9
 1
9
1 9

1
 x2  9
Let I =  dx =  x 2 dx =    2  x 
x 1 4
4 4  
 2 4
= 2  
9  4 = 2(3  2)
 I=2 [2]
iii. (A)
x
x x
y2  y3  x3 1
F(x) =  f ( y )dy   dy =   =  [2]
1 1
3  9  1 9 9

9
Board Answer Paper : July 2016
(B) Attempt any THREE of the following:
i. y = sec x

dy d

dx dx
sec x  
= sec x tan x
d
dx
 x [1]
1
= sec x . tan x .
2 x
dy sec x tan x
 [1]
dx 2 x
x+1
ii. Let I =  dx
 x+ 2  x+3
x+1 A B
  
 x+ 2  x+3  x+ 2   x+3
 x + 1 = A(x + 3) + B(x + 2) ….(i)
 Putting x = 2 in equation (i) we get
1 = A
 A = 1
 Putting x = 3 in equation (i) we get
2 = B
 B=2 [1]
x+1 1 2
  
 x+ 2  x+3  x+ 2   x+3
 1 2 
 I =     dx
 x  2 x+ 3 
 I =  log x  2  2log x  3  c [1]

iii. Let A be the required area


Consider the equation, y2 = 4x
 y= 2 x
4
 A =  y dx
1
4
=  2 x dx [1]
1
4
2 3
= 2.  x 2 
3  1
4  32
 4   1 2 
3
= 
3 
4
= 8  1
3
28
A= sq. units. [1]
3

iv. sec2 x  tan y dx + sec2 y  tan x dy = 0


Dividing throughout by tan x tan y, we get
sec 2 x tan y sec 2 y tan x
dx + dy = 0
tan x tan y tan y tan x

10
Mathematics

sec 2 x sec 2 y
 dx + dy = 0 [1]
tan x tan y
Integrating on both sides, we get
sec 2 x sec 2 y
 tan x
dx +  tan y
dy = 0

 log |tan x| + log |tan y| = log |c|


 log |tan xtan y| = log |c|
 tan x tan y = c [1]

v. Given E(x) = 6, Var. (x) = 4.2


E( x) np
Now 
V( x) npq
6 1

4.2 q
4.2
 q=  0.7
6
p+q=1
p + 0.7 = 1
p = 0.3 [1]
then E(X) = np,
6 = n  0.3
6
n=
0.3
n = 20 [1]

Q.5. (A) Attempt any TWO of the following:


i. f is continuous at x = 0.
 f(0) = lim f(x) [1]
x 0

4  1
2 2
sin x
 4sin x 1  sin 2 x
  2
4  1 
sin x 2

x2  sin x  x
= lim = lim = lim [1]
x 0 x log 1  2 x  x 0 x  log 1  2 x  x 0 log(1 2 x)
2
x2 2x
2
 4sin x  1 sin x 
 lim  lim 
x  0 sin x x 0 x 
= 
log 1  2 x 
2lim
x 0 2x
 log 4 
2

 f(0) = [1]
2

1
ii. Let I =  3 + 2sin x + cos x dx
Put tan   = t
x
2
 x = 2 tan–1 t
2dt 2t 1- t 2
 dx = 2
and sin x = 2
, cos x = [1]
1+ t 1+ t 1 + t2

11
Board Answer Paper : July 2016
1 2
 I =  2t

1  t 2 1+ t 2
dt
3+ 2 +
1+ t 2 1+ t 2
2dt dt
=  =2  2
3 +3t + 4t +1  t
2 2
2t + 4t + 4
2 dt dt
=
2 t 2
+ 2t + 2
=  (t +1)
2
 1+ 2
[1]

dt
=  (t +1) + 12 2
= tan1 (t + 1) + c

éx æ ö ù
 I = tan1 ê tan ççç ÷÷÷ +1ú + c [1]
êë è 2ø úû
iii. ‘y’ is a differentiable function of ‘x’.
Let there be a small change x in the value of ‘x’.
Correspondingly, there should be a small change y in the value of ‘y’.
As x  0, y  0
x y
Consider,  =1
y x
x 1 y
 = , 0
y y x
x
Taking lim on both sides, we get
x  0

 x  1
lim   = [1]
 y 
x  0  y 
lim  
x  0 x
 
Since ‘y’ is a differentiable function of ‘x’
 y  dy
 lim   =
 x  dx
x  0

As x  0, y  0
 x  1
 lim   = ….(i)
 y 
y  0  y 
lim  
x  0 x
 
 limits on R.H.S. of (i) exist and are finite. [1]
Hence, limits on L.H.S. of (i) also should exist and be finite.
 x  dx
 lim   = exists and is finite.
 y  dy
y  0

dx 1 dy
 = , 0 [1]
dy  dy  dx
 
 dx 

(B) Attempt any TWO of the following: S

i. Let S be the position of source of light.


Let BD be the position of the man at a time t min. D
Let AB = x ft. be distance of man from lamp at time t min. 30
and BC = length of the shadow of man at time t min = y ft.
6
dx
= 100 ft/min
dt A x y C
B
ASC  BDC
12
Mathematics
AS AC
 =
BD BC
30 x+ y
 =
6 y
 5y = x + y
x
y= [1]
4
dy 1 dx
 = .
dt 4 dt
1
= 100 
4
= 25
 The shadow of the man is lengthening at the rate 25 ft/min. [1]
The tip of shadow is C.
Let AC = z
 z=x+y
dz dx dy
  = 100 + 25
dt dt dt
dz
 = 125 ft/min [1]
dt
 The tip of the shadow is moving at the rate 125 ft/min. [1]
ii. E(X) =  x  P( x )
i i

= 0(0.08) + 1(0.15) + 2(0.45) + 3(0.27) + 4(0.05)


= 0 + 0.15 + 0.9 + 0.81 + 0.2
= 2.06 [1]
E(X2) = 
xi2  P( xi )
= 02(0.08) + 12(0.15) + 22(0.45) + 32(0.27) + 42(0.05)
= 0(0.08) + 1(0.15) + 4(0.45) + 9(0.27) + 16(0.05)
= 0 + 0.15 + 1.8 + 2.43 + 0.8
= 5.18 [1]
Var (X) = E(X2)  [E(X)]2 [1]
= 5.18  (2.06)2
= 5.18  4.2436
= 0.9364 [1]
a
iii. Let I =  f  x  dx
0

Put x = a – t
 dx = – dt
When x = 0, t = a  0 = a
When x = a, t = a  a = 0
a 0
 I=  f ( x)dx =  f (a  t)(dt) [1]
0 a
0
=   f (a  t)dt
a

a  b a 
=  f (a  t)dt ….   f  x  dx    f  x  dx 
0  a b


13
Board Answer Paper : July 2016
a
 b b

=  f (a  x)dx ….   f  x  dx   f  t  dt  [1]
0  a a 
a a
  f ( x)dx =  f (a  x)dx
0 0

2 sin x
Let I =  …. (i)
0 sin x  cos x
 

sin   x 
2
2   a a

I=  dx ….   f ( x)dx   f (a  x)dx 
0       
sin   x   cos   x  0 0
2  2 

2 cos x
=  dx ….(ii) [1]
0 cos x  sin x
Adding (i) and (ii), we get

2 sin x+ cos x
2I =  dx
0 sin x  cos x

2
=  1dx
0

=  x 02

= 0
2

2I =
2

I=
4

2 sin x 
  dx  [1]
0 sin x  cos x 4

Q.6. (A) Attempt any TWO of the following:


i. Let y = etan x + (log x)tan x
Put u = etan x and v = (log x)tan x
 y=u+v
dy du dv
 = + ….(i) [1]
dx dx dx
u = etan x
Taking logarithm on both sides, we get
log u = tan xlog e = tan x
Differentiating w. r. t. x, we get
1 du
= sec2 x
u dx
du
 = usec2 x
dx
du
 = etan x  sec2 x ….(ii)
dx
v = (log x)tan x
Taking logarithm on both sides, we get
log v = tan xlog (log x)
14
Mathematics
Differentiating w.r.t. x, we get
1 dv 1 1
= tan x + log (log x)sec2 x
v dx log x x
dv é tan x ù
 = v êê + log (log x)sec2 xú
ú
dx ë x log x û
é tan x ù
= (log x)tan x êê + log (log x) ⋅ sec 2 xú
ú
….(iii) [1]
ë x log x û
From (i), (ii) and (iii), we get
dy  tan x 
= etan x  sec2 x + (log x)tan x    log (log x)sec 2 x  [1]
dx  x log x 

ii. Let X be the number of fluorescent lights that have a useful life of at least 800 hours.
P(a light has useful life of at least 800 hours) = p = 0.9, q = 1  0.9 = 0.1
Given n = 20
 X ~ B (20, 0.9)
The p.m.f. of X is given by
P(X = x) = p(x) = 20Cx (0.9)x (0.1)20x , x = 0,1,2, ……,20 [1]
P(at least 2 lights will not have a useful life) = P(at most 18 will have useful life)
= P(X  18) = 1  P(X > 18)
= 1  [P(X = 19) + P(X = 20)]
= 1  [20C19 (0.9)19 (0.1) + 20C20 (0.9)20] [1]
 19
9 9 20
 9 19

= 1   20    = 1   20  20  9  
 1020 1020   10 
 919  29 
=1  20 
 10 
29  919
Let M =
1020
 log M = log 29 + 19 log 9  20 log 10
= 1.4624 + 19 × 0.9542  20  1
= 1.4624 + 18.1298  20
= 19.5922  20
= 19.5922  19  1
= 1 .5922
 M = Antilog ( 1 .5922 ) = 0.3910
 P(at least two lights will not have a useful life) = 1  0.3910 = 0.6090 [1]
π π
iii. As function is continuous on [, ], it is continuous at x =  and x =
2 2
 lim f(x) =
-
lim f(x)
+
x - π x - π
2 2

 lim (2sin x) = lim ( sin x + )


- +
x - π x - π
2 2

æ -π ö÷ æ -π ö
 2sin ççç ÷÷ =  sin ççç ÷÷÷ + 
è 2 ø è 2 ø
 2(1) = (1) + 
 +=2 ….(i) [1]
15
Board Answer Paper : July 2016
π
Also, function is continuous at x =
2
 lim f(x) = lim+ f(x)
-
x π x π
2 2

 lim ( sin x + ) = lim+ cos x


-
x π x π
2 2

æπö π
  sin ççç ÷÷÷ +  = cos
è 2ø 2
 +=0 ….(ii) [1]
Solving (i) and (ii), we get
 = 1 and  = 1 [1]

(B) Attempt any TWO of the following:


dy
i. Since, represents the slope of tangent to a given curve at a point (x, y), the given equation is
dx
dy
+5=x+y [1]
dx
dy
  y = (x  5)
dx
dy
The given equation is of the form + Py = Q
dx
where, P = 1 and Q = (x  5)

I.F. = e  = e 
Pdx 1.dx
 = e x [1]
 Solution of the given equation is
y (I.F.) =  Q (I.F.) dx + c
y e x =  ( x  5) e
x
 dx + c

  x e  x dx  5  e  x dx + c

d  e x
= x  e  x dx     x   e  x dx  dx  5 +c
 dx  1
e x
=  x ex   1 dx  5e + c
x

=  x ex +  e  x dx + 5ex + c
=  x ex  ex + 5ex + c
 y ex =  x ex + 4ex + c
 y =  x + 4 + cex [1]
 x + y  4 = cex is the general solution.
Since the curve is passing through the point (0, 2)
 x = 0, y = 2
 0 + 2  4 = ce0
 c=2
 x + y  4 =  2ex
 y = 4  x  2ex is the required equation of the curve. [1]
16
Mathematics

ii. Let  vdx  w


dw
 =v
dx
d dw du
Consider, (uw) = u +w [1]
dx dx dx
du
= uv +  v dx  dx
du
= uv +
dx  vdx
Integrating on both sides w.r.t. ‘x’, we get
uw =  u vdx    
du
 dx
 vdx  dx [1]

 du 
 u  v dx=  uvdx     vdx  dx
 dx 
 d 
  u  v dx = u  vdx    dx (u)   vdx  dx [1]

Hence,  xe xdx  x .  e xdx    


d
x .  e x dx  dx
 dx 
= xex  1  e x dx
= xex  ex + c [1]
log e x
iii. Let f(x) = log10 x = = (log10 e)(loge x)
log e 10
= (04343) log x
0  4343
 f  (x) = [1]
x
x = 1016 = 1000 + 16 = a + h
Here, a = 1000 and h = 16
f(a) = f(1000) = log10(1000) = log10 (10)3
= 3log10 10 = 3 ….[ log10 mn = n log10 m]
f (a) = f (1000) = 0  4343 = 00004343 [1]
1000
f (a + h)  f(a) + hf (a)
 log10 (1016)  3 + 16(00004343) [1]
 3 + 00069488
 log10 (1016)  3006949 [1]

17
Mathematics

BOARD ANSWER PAPER : MARCH 2017


MATHEMATICS AND STATISTICS
SECTION – I
Q.1. (A) Select and write the most appropriate answer from the given alternatives in each of the
following sub-questions:
i. (C)
Let a , b , c be the position vectors of points A, B, C respectively.
 a = 2 î + ĵ + k̂ , b =  ĵ + 4 k̂ , c = k î + 3 ĵ  2 k̂
 AB = b  a
= ( ĵ + 4 k̂ )  (2 î + ĵ + k̂ )
=  2 î  2 ĵ + 3 k̂
and AC = c  a
= (k î + 3 ĵ  2 k̂ )  (2 î + ĵ + k̂ )
= (k 2) î + 2 ĵ  3 k̂
Since points A, B and C are collinear.
 AB and AC are collinear.
 AB  AC = 0
ˆi ˆj kˆ
 -2 -2 3 = 0
k - 2 2 -3

 î (0)  ĵ(3k) + k̂ ( 4 + 2k  4) = 0
 3 ĵ + ( 8 + 2k) k̂ = 0  î + 0  ĵ + 0  k̂
By equality of vectors, we get
 8 + 2k = 0
 k=4 [2]
ii. (A)
a b  1  d b 
If A =   , then A1 =
c d ad  bc   c a 
1  2 5
 A= [2]
13  3 1

iii. (B)
a + b + c 13  14  15
s= = = 21
2 2
A  s  b  s  c   21  14  21  15 1
sin   = = = [2]
2 bc 14  15 5

1
Board Answer Paper : March 2017
(B) Attempt any THREE of the following:
i. Let a = 2iˆ  3jˆ  4kˆ
b = 5iˆ  7ˆj  5kˆ
c = 4iˆ  5jˆ  2kˆ
be the coterminus edges of the parallelopiped.
volume of the parallelopiped = a b c 
2 3 4
V = 5 7 5 [1]
4 5 2
= 2  (– 39) –3  (– 30) – 4  (– 3)
= – 78 + 90 +12
= 24
 V = 24 cubic units [1]
ii. L.H.S. = a (b cos c – c cos B)
  a 2  b2  c2   a 2  c2  b2  
= a b    c   [1]
  2ab   2ac 
a 2
L.H.S. = [a + b2 – c2 – a2 – c2 + b2]
2a
1
= [2b2 – 2c2]
2
= b2 – c2 [1]
= R.H.S.
iii. The coordinates of the points A and B are (0, b, c) and (a, 0, c) respectively.
 OA  bjˆ  ckˆ , OB  aiˆ  ckˆ
The plane OAB passess through O  0  and is perpendicular to n  OA  OB
ˆi ˆj kˆ
 n  OA  OB  0 b c [1]
a 0 c
= bciˆ  acjˆ  abkˆ
 The equation of the plane OAB is
 r  0  n  0
 rn  0
  
r̂  bciˆ  acjˆ  abkˆ  0 [1]

iv. The equation of the line passing through A(x1, y1, z1) and B(x2, y2, z2) is
x  x1 y  y1 z  z1
  [1]
x2  x1 y2  y1 z 2  z1
 The quation of the line passing through (3, 4, –7) and (6, –1, 1) is
x3 y 4 z7
 
6  3 1  4 1  7
x3 y 4 z7
   [1]
3 5 8
2
Mathematics
v. Let p :  n  N, n2 + n is an even number.
q :  n  N, n2 – n is an odd number.
 Required symbolic form is p  q [1]
Now statement p is always True.
Statement q is always False.
 Truth value of p  q is
TF
F [1]
Q.2. (A) Attempt any TWO of the following:
i.
1 2 3 4 5 6
p q r pq pr (p  q)  (p  r)
T T T T T T
T T F T F T
T F T F T T
T F F F F F
F T T F F F
F T F F F F
F F T F F F
F F F F F F
In the above truth table, the entries in the last column are a combination of T and F.
 (p  q)  (p  r) is a contingency. [1]
[1 mark each for column 4 or 5 and column 6]
ii. Equations of the lines are
x 1 y  2 z  3
 
2 3 4
x 2 y 4 z 5
 
3 4 5
Here x1 = 1, y1 = 2, z1 = 3
x2 = 2, y2 = 4, z2 = 5
a1 = 2, b1 = 3, c1 = 4
a2 = 3, b2 = 4, c2 = 5
Shortest distance between the lines is
x2  x1 y2  y1 z 2  z1
a1 b1 c1
a2 b2 c2
= [1]
 b1c2  b2c1    c1a 2  c 2a1    a1b 2  a 2 b1 
2 2 2

1 2 2
2 3 4
3 4 5
= [1]
15  16   12  10    8  9 
2 2 2

115  16   2 10  12   2  8  9 
=
1 4 1
1  4  2
=
6
1
= units [1]
6

3
Board Answer Paper : March 2017
iii. sin 2x + sin 4x + sin 6x = 0
 (sin 2x + sin 6x) + sin 4x = 0
 6x  2x   6x  2x 
 2sin   cos   + sin 4x = 0
 2   2 
 2sin 4x cos 2x + sin 4x = 0 [1]
 sin 4x (2cos 2x + 1) = 0
1
 sin 4x = 0 or cos 2x =  [1]
2
    2 
 sin 4x = 0 or cos 2x =  cos   = cos     = cos  
3
   3   3 
Since, sin  = 0 implies  = n and cos  = cos  implies  = 2n   , n  Z.
2
 4x = n or 2x = 2m 
3
n 
 general solution is x = or x = m  , where n, m  Z. [1]
4 3
(B) Attempt any TWO of the following:
i. Given equations are
x–y+z=4
2x + y – 3z = 0
x+y+z=2
Matrix form of the given system of equations is
 1 1 1   x   4 
 2 1 3  y    0  [1]
     
 1 1 1   z   2 
 1 1 1   x 4
Where, A =  2 1 3 , X =  y  , B =
  0
 
 1 1 1   z   2 
Applying R2  R2 – 2R1, R3  R3 – R1
1 1 1   x   4 
 0 3 5  y    8 
     
 0 2 0   z   2 
2
Applying R3  R3 – R2
3
   
1 1 1   x   4 
    
  0 3 5   y    8  [1]
 10   z  10 
0 0   
 3 3
Hence, The original matrix A is reduced to an upper triangular matrix
 By equality of matrices, we get
 x–y+z=4 …(i)
3y – 5z = – 8 …(ii)
10 10
z …(iii) [1]
3 3
i.e., z = 1
4
Mathematics
Substituting z = 1 in equation (ii), we get
3y – 5 = – 8
 3y = – 3
 y=–1
Substituting y = –1 and z = 1 in equation (i), we get
x+1+1=4
 x=2
 x = 2, y = – 1, z = 1 is the required solution [1]
ii. The given combined equation of lines is ax2 + 2hxy + by2 = 0
Let m1 and m2 be the slopes of the lines represented by ax2 + 2hxy + by2 = 0
2h a
 m1 + m2 =  and m1m2 = , b  0
b b
a
If = 1, then m1m2 = 1.
b
 lines are perpendicular. [1]
a
So we assume that  1
b

Y y = m2x

y = m1x

X X
O

Y
Now, (m1  m2)2 = (m1 + m2)2  4m1.m2
2
4a 4h 2 4a
=    
2h
= 2 
 b  b b b
4h 2  4ab 4  h  ab 
2
2
 (m1  m2) = =
b2 b2
Taking square root on both the sides, we get
2 h 2  ab
m1  m2 =  [1]
b
Let  be the acute angle between the lines.
2 h 2  ab
m1  m 2 
 tan  = = b , a  1
1  m1 .m 2 a b
1
b
2 h 2  ab
 tan  = ,a+b0 [1]
ab
Lines are coincident (parallel), if and only if m1 = m2
i.e., if m1  m2 = 0
5
Board Answer Paper : March 2017

2 h 2  ab
i.e., if  =0
b
i.e., if h2  ab = 0
i.e., if h2 = ab
 Lines are coincident if and only if h2 = ab. [1]

iii. Let p, q, r be the position vectors of vertices P, Q, R respectively of PQR.


 p = 4 ĵ, q = 3 k̂ and r = 4 ĵ + 3 k̂ [1]
Now, PQ = q  p = 4 ĵ + 3 k̂
PR = r  p = 3 k̂
QR = r  q = 4 ĵ
Let x = QR , y = PR and z = PQ
 x = 4, y = 3, and z = 5. [1]
If H (h ) is the incentre of PQR, then
x p + y q + zr
h = [1]
x+ y+z
(4)(4ˆj) + (3)(3k)
ˆ + (5)(4ˆj + 3k)
ˆ
 h =
4 +3+5
16ˆj + 9kˆ + 20ˆj + 15kˆ 36ˆj + 24kˆ
= =
12 12
 h = 3 ĵ + 2 k̂
 H  (0, 3, 2)
 The incentre of PQR is (0, 3, 2) [1]
Q.3. (A) Attempt any TWO of the following:
i. Let p : The switch S1 is closed.
q : The switch S2 is closed.
 ~p : The switch S1 is closed or the switch S1 is open.
~q : The switch S2 is closed or the switch S2 is open. [1]
Consider the given statement
(p  q)  (~ p)  (p  ~ q)
p  q : represents that switches S1 and S2 are connected in series.
p  q : represents that switches S1 and S2 are connected in series.
Therefore (p  q)  (~ p)  (p  ~ q) represents that circuits corresponding to (p  q), (~ p) and
(p  ~ q) are connected in parallel with each other [1]
 Switching circuit corresponding to the given statement is

S1 S2

S1 [1]

S1 S2
L

6
Mathematics
ii. Given equation is 5x2 + 2xy  3y2 = 0.
Comparing with ax2 + 2hxy + by2 = 0, we get
a = 5, 2h = 2, b = 3
Let m1 and m2 be the slopes of the lines represented by 5x2 + 2xy  3y2 = 0
-2h 2 a -5
 m1 + m2 = = and m1 m2 = = [1]
b 3 b 3
Since, the required lines are perpendicular to these lines
1 1
 slopes of the required lines are - and - .
m1 m2
Required lines also pass through the origin, therefore their equations are
1 1
y= - x and y = - x
m1 m2
 x + m1y = 0 and x + m2y = 0
 the joint equation of the lines is
(x + m1y)(x + m2y) = 0 [1]
 x2 + (m1 + m2)xy + m1m2y2 = 0
2 5
 x2 + xy  y2 = 0
3 3
 3x + 2xy  5y2 = 0
2
[1]

4
iii. Let x = cos1  
5
4 
 cos x = and 0 < x <  sin x > 0
5 2
2
4 16 3
Now, sin x = 1 cos x = 1   = 1
2
=
5 25 5
 12 
Let y = cos1  
 13 
12 
 cos y = and 0 < y <
13 2
 sin y > 0
2
 12  144 5
Now, sin y = 1 cos 2 y = 1    = 1  = [1]
 13  169 13
4 12 3 5
But, cos (x + y) = cos x cos y  sin x sin y =    [1]
5 13 5 13
48  15 33
= =
65 65
 33 
 x + y = cos1  
 65 
4  12   33 
 cos1   + cos1   = cos1   [1]
5  13   65 

7
Board Answer Paper : March 2017
(B) Attempt any TWO of the following:
i. Let AB be a line with direction cosines l, m, n.
Z

B
A
 L
P(x, y, z)

Y
O

L

Consider a line L passing through the origin and parallel to the line AB.
Let P(x, y, z) be a point on the line L and l(OP) = r.
The position vector of point P is
OP = x î + y ĵ + z k̂
If , ,  are the direction angles of line OP, then
l = cos , m = cos , n = cos .
Consider, OP  î = (x î + y ĵ + z k̂ )  î
=x ….(i) [1]
Also, OP . î = OP  î  cos 
= r cos  ….(ii)
 x = r cos  ….[From (i) and (ii)]
Similarly, we have
y = r cos , z = r cos  [1]
 OP = r cos  î + r cos  ĵ + r cos  k̂
2
Now, OP  OP = OP
 (r cos )2 + (r cos )2 + (r cos )2 = r2
 r2 (cos2  + cos2  + cos2 ) = r2
 cos2  + cos2  + cos2  = 1
 l2 + m2 + n2 = 1
Given,  = 135,  = 45
Since, cos2  + cos2  + cos2  = 1 [1]
 cos2  + cos2 (135) + cos2 (45) = 1
2 2
 1   1 
 cos2  +     =1
 2  2
1 1
 cos2  + + =1
2 2
 cos2  = 0
 cos  = 0
  = 90
 the direction angle of the line with the X - axis is 90 [1]
8
Mathematics

ii Let a, b, c be the position vectors of the points A, B, C respectively.

 a = î + ĵ  2 k̂ , b = î + 2 ĵ + k̂ , c = 2 î  ĵ + k̂

B C

Now, AB = b  a = ĵ + 3 k̂

AC = c  a = î  2 ĵ + 3 k̂ [1]

n is perpendicular to AB and AC both.


 n = AB  AC
ˆi ˆj kˆ
= 0 1 3
1 -2 3

= î (3 + 6)  ĵ (0  3) + k̂ (0  1)

= 9 î + 3 ĵ  k̂ [1]

Vector equation of the plane is r.n  a.n [1]

 r   9iˆ + 3jˆ  kˆ  =  ˆi + ˆj  kˆ    9iˆ + 3jˆ  kˆ 


= 1(9) + 1(3)  2(1) = 9 + 3 + 2

 r   9iˆ + 3jˆ  kˆ  = 14

  xˆi + yˆj  zkˆ   9iˆ + 3jˆ  kˆ  = 14


 9x + 3y – z = 14 is the required equation of plane. [1]

iii. To draw the feasible region, construct table as follows:

Inequality x2 x+y3 2x + y  1


Corresponding equation (of line) x=2 x+y=3 2x + y = 1
Intersection of line with X-axis (2, 0) (3, 0)  1 
  ,0 
 2 
Intersection of line with Y-axis  (0, 3) (0, 1)
[1]
Region Origin side Origin side Origin side

Shaded portion OABCD is the feasible region, whose vertices are O(0, 0), A(2, 0), B, C and D(0, 1).
B is the point of intersection of the lines x = 2 and x + y = 3.
9
Board Answer Paper : March 2017
Putting x = 2 in x + y = 3, we get
Y
y=1
 B  (2, 1) 5
C is the point of intersection of the
lines x + y = 3 and 2x + y = 1. 4
Solving the above equations, we get 3
C  , 
2 7
2 7 2 7
x= ,y=  C=  ,  [1] 2 3 3
3 3 3 3
1 B(2, 1)
Here, the objective function is Z = 6x + 4y, (0, 1)D
A(2, 0)
 Z at O(0, 0) = 6(0) + 4(0) = 0 X X
4 3 2 1 O 1 2 3 4
Z at A(2, 0) = 6(2) + 4(0) = 12
1
Z at B(2, 1) = 6(2) + 4(1) = 12 + 4 = 16
2 7 2 7 2 x+y=3
Z at C  ,  = 6   + 4  
3 3 3 3 3
x=2
12 28 40 2x + y = 1
= + = = 13.33 4
3 3 3
Z at D(0, 1) = 6(0) + 4(1) = 4 Y [1]
 Z has maximum value 16 at B(2, 1)
 Z is maximum, when x = 2 and y = 1 [1]

SECTION – II
Q.4. (A) Select and write the most appropriate answer from the given alternatives in each of the
following sub-questions:
i. (B)
Let u = tan3 
du
 = 3 tan2  sec2 

and v = sec3 
dv
 = 3sec2  sec  tan  = 3sec3  tan 

du 3tan 2  sec 2  tan 
 = =
dv 3
3sec  tan  sec 

at  =
3

tan
du
= 3 = 3 [2]
dv  2
sec
3
ii. (A)
Equation of the curve is y = 3x2  x + 1
Differentiating w.r.t. x, we get
dy
= 6x  1
dx
 Slope of tangent at (1, 3) is
 dy 
  = 6(1)  1 = 5
 dx (1,3)

10
Mathematics

 dy 
Equation of tangent is y  y1 =   (x  x1)
 dx ( x1 , y1 )
Here, (x1, y1)  (1, 3)
 y  3 = 5(x  1)
 5x – y = 2 [2]
iii. (B)
Three coins are tossed
 S = {HHH, HHT, HTH, THH, TTH, THT, HTT, TTT}
 n(S) = 8
Thus, the probability distribution of X is as follows:
Here X= number of heads obtained
X 0 1 2 3
1 3 3 1
P(X = x)
8 8 8 8
Expected value = E(X) =  x  P( x )
i i

1  3  3 1
= 0   + 1   + 2   + 3   = 1.5 [2]
 
8 8
  8
  8
(B) Attempt any THREE of the following:
i. x sin y + y sin x = 0
Differentiating w.r.t. x, we get
d d d d
x (sin y) + sin y  x  + y  sin x  + sin x  y = 0 [1]
dx dx dx dx
dy dy
  xcos y + sin y + y cos x + sin x = 0
dx dx
dy
  (x cos y + sin x) =  (sin y + y cos x)
dx
dy  (sin y  y cos x)
 = [1]
dx ( x cos y  sin x)

1
ii. f(x) = x  , x  R, x  0
x
1
 f (x) = 1 + 2 [1]
x
Since x2 is always positive, x  0
 f (x) > 0 for all x  R, x  0
Hence, f(x) is an increasing function, for all x  R, x  0. [1]

sin x
iii. Let I =  x
dx

Put x  t
Differentiating w.r.t. x, we get
1
dx = dt [1]
2 x
11
Board Answer Paper : March 2017

 I =  sin t  2dt
= 2 sin t dt
= 2 (– cos t) + c
= –2 cos x  c [1]
iv. y = Ae5x + Be 5x ....(i)
Differentiating w.r.t. x, we get
dy
= 5Ae5x  5Be 5x [1]
dx
Again, differentiating w.r. t. x, we get
d2 y d d
2
= 5A (e5x)  5B (e 5x)
dx dx dx
= 5A(5e5x)  5B( 5e 5x)
= 25Ae5x + 25Be 5x
= 25(Ae5x + Be 5x) = 25y ....[From (i)]
d2 y
 = 25y
dx 2
d2 y
  25y = 0 [1]
dx 2
v. Let X be the number of bombs hitting the target.
P(bomb hits the target) = p = 0.8, q = 1  0.8 = 0.2
Given, n = 10
 X ~ B (10, 0.8)
The p.m.f. of X is given by
P(X = x) = p(x) = 10Cx (0.8)x (0.2)10x , x = 0, 1, 2, ….,10 [1]
 P(exactly 4 bombs will hit the target) = P(X = 4)
= 10C4 (0.8)4 (0.2)6
10  9  8  7
=  0.4096  6.4  10–5
4  3 2 1
= 210  0.4096  6.4  10–5
= 550.5024  10–5
–5
 required probability is 550.5024  10 . [1]
Q.5. (A) Attempt any TWO of the following:
dy
i. Given, = cos (x + y) …(i)
dx
Put x + y = v …(ii)
 y=v–x
dy dv
  –1 …(iii)
dx dx
Substituting (ii) and (iii) in (i), we get
dv
 1  cos v [1]
dx
dv
 = 1 + cos v
dx
dv v
 = 2 cos2
dx 2

12
Mathematics
1
 dv  2dx
v
cos 2
2
v
 sec2 dv = 2dx [1]
2
Integrating on both sides, we get
2 v
 sec 2 dv  2 dx
v
 2tan = 2x + c
2
v c
 tan = x +
2 2
 x y c
 tan   = x + c, where c = 2 [1]
 2 

ii. Let  v dx  w
dw
 =v
dx
d dw du
Consider, (uw) = u +w [1]
dx dx dx
du
= uv +  v dx 
dx
du
dx 
= uv + v dx
Integrating on both sides w.r.t. ‘x’, we get
 du 
uw =  u vdx     v dx  dx [1]
 dx 
 du 
 u  v dx=  uv dx     v dx  dx
 dx 
 d 
  u  v dx = u  v dx    dx (u)   v dx  dx [1]

iii. f is continuous at x = 0.
2
e x  cos x
 f(0) = lim f(x) = lim [1]
x 0 x 0 x2
2
e x  1  cos x +1  e x2  1 1  cos x 
= lim = lim  2  
x2 x0  
2
x 0
 x x 
x
2 2sin 2
ex  1 2
= lim + lim [1]
x 0 x2 x 0 4 2
x
4
2
 x
1  sin 2  1 2 1
= log e + lim   = log e + (1) = 1 +
2 x 0  x  2 2
 2 
3
 f(0) = [1]
2
13
Board Answer Paper : March 2017
(B) Attempt any TWO of the following:
i. ‘y’ is a differentiable function of ‘x’.
Let there be a small change x in the value of ‘x’.
Correspondingly, there should be a small change y in the value of ‘y’.
As x  0, y  0
x y
Consider,  =1
y x
x 1 y
 = , 0
y y x
x
Taking lim on both sides, we get
x  0

 x  1
lim   = [1]
x  0
 y  lim  y 
 
x  0 x
 
Since ‘y’ is a differentiable function of ‘x’,
 y  dy
lim   =
x  0 x
  dx
As x  0, y  0
 x  1
lim   = ….(i)
 y  lim  y 
y  0
 
x  0 x
 
 limits on R.H.S. of (i) exist and are finite. [1]
Hence, limits on L.H.S. of (i) also should exist and be finite.
 x  dx
 lim   = exists and is finite.
 y  dy
y  0

dx 1 dy
 = , 0
dy  dy  dx
 
 dx 
y = tan1 x, [1]
 x = tan y
Differentiating w.r.t. y, we get
dx
= sec2 y
dy
dy 1
 =
dx sec2 y
dy 1
 =
dx 1  tan 2 y
dy 1
 = [1]
dx 1  x2
ii. Number of subscribers = 5000
and annual rental charges per subscriber = ` 3000.
For every increase of 1 rupee in the rent, one subscriber will be discontinued.
14
Mathematics
Let the rent be increased by ` x.
 New rental charges per year = `(3000 + x)
and number of subscribers after the increase in rental charges = 5000  x.
Let R be the annual income of the company.
Then, R = (3000 + x)(5000  x) [1]
= 15000000  3000x + 5000x  x2
= 15000000 + 2000x  x2
dR d2R
 = 2000  2x and = 2 [1]
dx dx 2
dR
Now, R is maximum if =0
dx
 2000  2x = 0
 x = 1000 [1]
d R
2
  2  = 2 < 0 and hence, R is maximum when x = 1000.
 dx  x 1000 
Thus, the annual income of the company is maximum when the annual rental charges are
increased by ` 1000. [1]
a
ax
iii. Let I = 
a
ax
dx
a
ax ax
= 
a

ax ax
dx
a
ax
= 
a a 2  x2
dx [1]
a a
a x
= 
a a 2  x2
dx  
a a 2  x2
dx

 a a

  f ( x)dx  2  f ( x)dx if f ( x) is even 
a
1
 I = 2a  dx  0 ….   a 0 
a 2  x2  if f ( x)is odd 
0
=0
= 0 if f(x) is odd [1]
a
 x
= 2a sin 1  [1]
 a 0
= 2a [sin 1 – sin–1 0]
–1

 
= 2a   0 
 2 
 I = a [1]
Q.6. (A) Attempt any TWO of the following:
x
i. lim f ( x)  lim
x 0  x 0 x

x
= lim
x 0 x
= –1 [1]
15
Board Answer Paper : March 2017
x x
lim f ( x)  lim  lim
x  0 x  0 x x 0 x
=1 [1]
 lim f ( x)  lim f ( x)
x  0 x  0

 lim f ( x) does not exist


x 0

 f(x) is discontinuous at x = 0 [1]


ii. Let ‘x’ be the population of the country at time ‘t’.
dx
 x
dt
dx
 = kx
dt
dx
 = kdt
x
Integrating on both sides, we get
dx
 x 
= k dt

 log x = kt + c [1]
When t = 0, x = X0
 log X0 = k(0) + c
 c = log X0
 log x = kt + log X0
When t = 60, x = 2X0
 log (2X0) = 60k + log X0
 60k = log (2X0)  log X0
 2X 
 60k = log  0 
 X0 
1
 k= log 2 ....(i) [1]
60
When x = 3X0, we get
log(3X0) = kt + log X0
1
 log (3X0)  log (X0) = t log 2 ....[From (i)]
60
æ 3X ö t
 log çç 0 ÷÷÷ = log 2
çè X 0 ÷ø 60
log 3
 60 =t
log 2
60(1.0986)
 t= = 95.364  95.4 years
0.6912
 The population of the country will triple approximately in 95.4 years. [1]
iii. Let X be the number of heads out of 8 tosses.
1 1 1
P(getting head) = p = , q = 1  p = 1  =
2 2 2
Given n = 8
16
Mathematics

 1
 X ~ B (n, p)  X ~ B  8, 
 2
The p.m.f. of X is given by
x 8 x
1 1
P(X = x) = p(x) = 8 C x     , x = 0, 1, 2, ….,8 [1]
2 2
a. P(exactly 5 heads) = P(X = 5)
5 3
1 1 ! 1 8  7  6  5! 1 8 7 7
 P(X = 5) = 8C5     =  8=  = = = 0.21875 [1]
2 2
  5! 3! 2 3  2  1  5! 256 256 32

b. P(at least one head) = P(X  1)


0 8
1 1
 P(X  1) = 1  P(no head) = 1  P(X < 1) = 1  P(X = 0) = 1  8C0    
2 2
1
= 1  (1)(1) 
256
255
=
256
= 0.996 [1]
(B) Attempt any TWO of the following:
1
i. Let I =  d
sin   sin 2
1
=  d
sin  + 2sin  cos 
d sin  d
=  =  2
sin (1  2cos ) sin (1  2cos )
sin  d
=  (1  cos2
)(1  2cos )
[1]

sin  d
=  (1  cos )(1  cos )(1  2cos )
Put cos  = t
  sin  d = dt
 sin  d =  dt
 dt
 I = 
(1  t)(1  t)(1  2t)
dt
=  (1  t)(1  t)(1  2t)
1 A B C
Let =   [1]
(1  t)(1  t)(1  2t) 1  t 1  t 1  2t
 1 = A(1 + t)(1 + 2t) + B(1  t)(1 + 2t) + C(1  t)(1 + t) ….(i)
Putting in (i), t = 1, we get
1 = A(2)(3) + B(0)(3) + C(0)(2)
1
 A=
6
Putting t = 1 in (i), we get
1 = A(0)(1) + B(2)(1) + C(2)(0)
17
Board Answer Paper : March 2017
1
 B= 
2
1
Putting t =  in (i), we get
2
3 1
1 = A(0) + B(0) + C    
2 2
4
 C= [1]
3
1  1 4
1      
=  +
6 2 3
 +
(1  t)(1  t)(1  2t) 1  t 1  t 1  2t
 1   1   4  
 6    2   3  
 I =            dt
 1  t 1  t 1  2t 
 

1 1 1 1 4 1 1 log 1  t 1 4 log 1  2t
=  
6 1 t
dt  
2 1 t
dt  
3 1  2t
dt =  
6 1
+ log 1  t  
2 3 2
c

1 1 2
 I = log 1  cos   log 1  cos   log 1  2cos   c [1]
6 2 3

ii.
Y
x2 = 4ay

P(4a, 4a)

X X
O
(0, 0)

y2 = 4ax
Y
The equations of the parabolas are y2 = 4ax and x2 = 4ay
Solving the equations, we get
2
 x2 
  = 4ax
 4a 
 x4 = 64 a3 x
 x[x3 – (4a)3] = 0
 x = 0 or x = 4a [1]
When x = 0, y = 0 and when x = 4a, y = 4a
 The points of intersection of the parabolas are O (0, 0), P(4a, 4a) [1]
 the required area is,
18
Mathematics
A = (Area under parabola y2 = 4ax) – (Area under parabola x2 = 4ay)
4a 4a 2
x
=  4ax dx   dx [1]
0 0
4a
4a
2 3 1 1 4a
= 4a   x 2     x3 
3   0 4a 3 0

4 a 1
=  4a 4a   64a 3
3 12a
32 2 16 2
= a  a
3 3
16 2
= a sq. units. [1]
3

iii. Given the p.d.f. of a continuous random variable X as


x2
f(x) = , –1 < x < 2
3
= 0, otherwise.
 c.d.f. of X is given by
x
F(x) =  f ( y)dy
1
x
x
y2  y3 
=  dy   
1 3  9  1
x3 1
 F(x) =  ,  x  R. [1]
9 9
1 1 2
 P(X < 1) = F(1) =   [1]
9 9 9
 P(X > 0) = 1 – P (X  0)
1 8
= 1 – F(0) = 1 – = [1]
9 9
 P(1 < x < 2) = F(2) – F(1)
8 1 1 1
=   –  
9 9 9 9
2
=1–
9
7
= [1]
9

19
Mathematics

BOARD ANSWER PAPER : JULY 2017


MATHEMATICS AND STATISTICS
SECTION – I
Q.1. (A) Select and write the most appropriate answer from the given alternatives in each of the
following sub-questions:
i. (B)
a b  1  d b 
If A =   , then A1 =
c d ad  bc   c a 
1  3 1
 A= [2]
5  2 1
ii. (C)
3 1 4
a   b  c = 2 3 1
5 2 3
= 3(9 + 2) + 1 (6 – 5) + 4 (4 + 15)
= 33 + 1 + 76
= 110 [2]
iii. (D)
Let , ,  be the angles made by the line with positive directions of X, Y, Z axes respectively.
  = 90,  = 135,  = 45
 l = cos 90, m = cos 135, n = cos 45
Now, m = cos 135 = cos(180 – 45)
1
= cos 45 = 
2
1 1
 l = 0, m =  ,n=
2 2
1 1
 Direction cosines of the line are 0,  , [2]
2 2
(B) Attempt any THREE of the following:
i. b = 2iˆ  ˆj  2kˆ
n̂ = 3iˆ  2ˆj  pkˆ
Since, line is parallel to the plane
b  n̂ = 0 [1]
   
2iˆ  ˆj  2kˆ  3iˆ  2ˆj  pkˆ = 0
 6 – 2 + 2p = 0
 p=–2 [1]
ii. Consider cos 2 + cos 2 + cos 2
= (2cos2 1)+(2cos2 1)+(2cos2   1)
= 2(cos2  + cos2  + cos2 )  3 [1]
2 2 2
= 2(l)  3 …[  cos  + cos  + cos  = 1]
= 1
1
Board Answer Paper : July 2017
 cos 2 + cos 2 + cos 2 = –1.
i.e., cos 2 + cos 2 + cos 2 + 1 = 0 [1]
iii. (a)  n  N such that n + 7  6 [1]
(b) The kitchen is not neat or it is not tidy. [1]
iv. Let  be the acute angle between the lines whose direction ratios are 4, –3, 5 and 3, 4, 5.
Then,
a1a 2  b1b 2  c1c 2
cos  =
a  b12  c12  a 22  b 22  c 22
2
1

4  3   3 4   5  5 
 cos  = [1]
42   3  52  32  42  52
2

12  12  25
=
16  9  25  9  16  25
25 1
= =
50 2

 cos  = cos
3

 =
3

 The angle between the lines is . [1]
3

v. 3 a + 5 b – 8c = 0 ….(given)
 3 a = 8 c  5b
8c - 5b
 a= [1]
3
8c - 5b
 a=
8-5
 
A a divides BC externally in the ratio 8 : 5. [1]

Q.2. (A) Attempt any TWO of the following:



i. tan1 2x + tan1 3x =
4
  x  3x  
 tan1   = [1]
 1   2 x  3x   4
5x 
 = tan
1  6x 2
4
5x
 =1
1  6 x2
 5x = 1  6x2
 6x2 + 5x 1 = 0 [1]
 6x2 + 6x x  1 = 0
 6x(x + 1)  1(x + 1) = 0
 (x + 1)(6x  1) = 0
2
Mathematics
1
 x = 1 or x =
6

But x = 1 does not satisfy tan1 2x + tan1 3x =
4
1
 x= [1]
6
ii. Let p : It rains, q : the match will be cancelled.
 The symbolic form of the given statement is p  q.
Converse: q  p
i.e., If the match is cancelled then it rains. [1]
Inverse: ~p  ~q
i.e., If it does not rain then the match will not be cancelled. [1]
Contrapositive: ~q  ~p
i.e. If the match is not cancelled then it does not rain. [1]
iii. Given equation is px2  8xy + 3y2 + 14x + 2y + q = 0
Comparing with
ax2 + 2hxy + by2 + 2gx + 2fy + c = 0, we get
a = p, h =  4, b = 3, g = 7, f = 1, c = q.
The given equation represents a pair of lines perpendicular to each other
 a+b=0
 p+3=0
 p = 3 [1]
Also, the given equation represents a pair of lines
a h g
 h b f =0
g f c
-3 - 4 7
 -4 3 1 = 0 [1]
7 1 q
 3(3q  1) + 4(  4q  7) + 7( 4  21) = 0
 9q + 3  16q  28  175 = 0
 25q  200 = 0
 25q = 200
 q = 8 [1]
 p = 3 and q = 8
(B) Attempt any TWO of the following:
i. The equation of plane passing through the intersection of the planes 3x + 2y – z + 1 = 0
and x + y + z – 2 = 0 is
(3x + 2y – z + 1) + (x + y + z – 2) = 0 …. (i) [1]
It passes through the point (2, 2, 1)
 (6 + 4 – 1 + 1) +  (2 + 2 + 1 – 2) = 0 [1]
 10 + 3 = 0
10
 = [1]
3
3
Board Answer Paper : July 2017

 10 
Now, (3x + 2y – z + 1) +   (x + y + z – 2) = 0 .…[From (i)]
 3 
 9x + 6y – 3z + 3 – 10x – 10y – 10z + 20 = 0
 –x – 4y – 13z + 23 = 0
 x + 4y + 13z – 23 = 0 [1]
 The equation of plane is
x + 4y + 13z = 23

ii.
O

b
a r

A m n
R B

Consider a line segment AB.


Let R be any point on it such that point R divides AB internally in the ratio m : n.
OA = a, OR = r and OB = b are the position vectors of points A, R, B respectively.
Since point R divides AB internally in the ratio m : n,
AR m
 = [1]
RB n
 n(AR) = m(RB)
ARand RB are in the same direction.
 n( AR) = m (RB) [1]
 n (r  a) = m(b  r)
 nr  na = mb  mr
 r (mn) = mb na

mb  na
 r= ….(i) [1]
mn
This is the section formula for internal division.
Let P. V. of point A a = i  2j  k
P. V. of point B b = i  4j  2k
m 2
Given, 
n 1
2(i  4j  2k)
  1(i  2j  k)

Now, r = ....[From (i)]
2 1
3i  6j  3k
 r =
3
P. V. of R is r  i  2j  k [1]

4
Mathematics

iii. A, B, C are in A.P. and b : c = 3 : 2


 2B = A + C [1]
 2B = 180  B ….[ A + B+ C = 180]
 3B = 180
 B = 60 [1]
In ABC by sine rule, we have
sin B sin C
=
b c
sin B b
 =
sin C c
sin 60 3
 =
sin C 2
3 3
 =  sin C
2 2
1
 sin C = [1]
2
 C = 45
 A = 180  60  45 = 75
Thus, the angles of ABC are A = 75, B = 60, C = 45. [1]
Q.3. (A) Attempt any TWO of the following:
i. The vector equation of a line passing through the points having position vectors a and b is given
by r  a    b  a 
Here, a  3iˆ + 4 ˆj  7 kˆ and b  6 ˆi  ˆj  kˆ
 the vector equation of the line passing through A (3, 4, 7) and B (6, 1, 1) is
r  (3iˆ + 4 ˆj  7 kˆ )   [(6 ˆi  ˆj  kˆ )  (3iˆ + 4 ˆj  7 k)]
ˆ

 r  (3iˆ + 4 ˆj  7 k)
ˆ   ( 3iˆ  5 ˆj  8 k) ˆ ….(i) [1]
For cartesian form:
Putting r  x ˆi + y ˆj+ z kˆ in (i), we get
x ˆi + y ˆj  z kˆ  ( 3iˆ  4 ˆj  7 k)
ˆ   ( 3iˆ  5 ˆj  8 k)
ˆ

 xˆi + yˆj + zkˆ = (3 + 3)iˆ + (4  5)ˆj + (7  8)kˆ [1]


Equating the coefficients of ˆi, ˆj and k̂ , we get
x = 3 + 3, y = 4  5, z =  7 + 8
x 3 y4 z7
  ,  , 
3 5 8
x 3 y 4 z 7
   , which is the cartesian form of the equation of line. [1]
3 5 8
ii. cot x + tan x = 2cosec x
cos x sin x 2
 + =
sin x cos x sin x
cos x  sin x
2 2
2
 
sin x  cos x sin x
 cos2x + sin2 x = 2cos x
 2 cos x = 1
5
Board Answer Paper : July 2017
1
 cos x = [1]
2

 cos x = cos   [1]
3
Since, cos  = cos  implies  = 2n  , n  Z.

 the required general solution is x = 2n  , where n  Z. [1]
3
iii. Let p : The switch S1 is closed
q : The switch S2 is closed
p : The switch S1 is open
q : The switch S2 is open
The logical expression of the given switching circuit is (p  q)  (p  q) [1]
The switching table is:
p q p q p  q p  q (p  q)  (p  q)
1 1 0 0 1 0 1
1 0 0 1 1 0 1
0 1 1 0 0 1 1 [1]
0 0 1 1 1 0 1
From the last column of switching table we conclude that the current will always flow
through the circuit. [1]
(B) Attempt any TWO of the following:
é 1 -1 2ù
ê ú
i. A = ê3 0 - 2ú
ê ú
ê1 0 3ú
ë û
1 -1 2
 |A| = 3 0 -2 = 1(0) + 1(9 + 2) + 2(0)
1 0 3
= 0 + 11 + 0
 A = 11 [1]
0 -2
A11 = (1)1 + 1 M11 = 1 = 1(0  0) = 1  0 = 0
0 3
3 -2
A12 = (1)1 + 2 M12 = 1 = 1(9 + 2) = 11
1 3
3 0
A13 = (1)1 + 3 M13 = 1 = 1(0  0) = 0
1 0
-1 2
A21 = (1)2 + 1 M21 = 1 = 1(3  0) = 3
0 3
1 2
A22 = (1)2 + 2 M22 = 1 = 1(3  2) = 1
1 3
1 -1
A23 = (1)2 + 3 M23 = 1 = 1(0 + 1) = 1
1 0

6
Mathematics

-1 2
A31 = (1)3 + 1 M31 = 1 = 1(2  0) = 2
0 -2
1 2
A32 = (1)3 + 2 M32 = 1 = 1(2  6) = 8
3 -2
1 -1
A33 = (1)3 + 3 M33 = 1 = 1(0 + 3) = 3
3 0
Hence, matrix of the co-factors is
é A11 A12 A13 ù é 0 -11 0ù
ê úê ú
ê A 21 A 22 A 23 ú =
ê3 1 -1ú =  A ij 
ê úê ú 33
êA A 32 A 33 úûê2 8 3ú
ë 31 ë û
é 0 3 2ù
T ê ú
Now, adj A =  A ij  = ê -11 1 8ú [1]
33 ê ú
ê 0 -1 3úû
ë
é 1 -1 2ù é 0 3 2ù
ê ú ê ú
 A(adj A) = êê3 0 -2úú êê-11 1 8ú
ú
ê1 0 3úû êë 0 -1 3úû
ë
é 0 +11+ 0 3 -1- 2 2 - 8 + 6ù
ê ú
= êê 0 + 0 + 0 9 + 0 + 2 6 + 0 - 6ú
ú
ê 0 + 0 + 0 3 + 0-3 2 + 0 + 9 úû
ë
é11 0 0 ù
ê ú
= ê 0 11 0 ú .…(i) [1]
ê ú
ê 0 0 11ú
ë û
é 1 0 0ù é11 0 0 ù
ê ú ê ú
|A|  I = 11 êê0 1 0úú = êê 0 11 0 úú ….(ii)
ê 0 0 1ú ê 0 0 11ú
ë û ë û
From equations (i) and (ii), we get
A(adj A) = |A| I [1]
ii. Let x number of bicycles and y number of tricycles be manufactured by the company.
 Total profit Z = 180x + 220y
This is the objective function to be maximized. [1]
The given information can be tabulated as shown below:

Bicycles (x) Tricycles (y) Maximum availability of time (hrs)


Machine A 6 4 120
Machine B 3 10 180

 The constraints are 6x + 4y  120, 3x + 10y  180, x  0, y  0


 Given problem can be formulated as
Maximize Z = 180x + 220y
Subject to, 6x + 4y  120, 3x + 10y  180 , x  0, y  0. [1]
7
Board Answer Paper : July 2017
To draw the feasible region, construct the table as follows:
Inequality 6x + 4y  120 3x + 10y  180
Corresponding
6x + 4y = 120 3x + 10y = 180
equation (of line)
Intersection of line with X-axis (20, 0) (60, 0)
Intersection of line with Y-axis (0, 30) (0, 18)
Region Origin side Origin side

Shaded portion OABC is the feasible region, whose vertices are O  (0, 0), A  (20, 0), B and C
 (0, 18)
B is the point of intersection of the lines 3x + 10y = 180 and 6x + 4y = 120.
Solving the above equations, we get Y
B  (10, 15)
40
Here the objective function is,
Z = 180x + 220y 30
 Z at O(0, 0) = 180(0) + 220(0) = 0 [1]
20
Z at A(20, 0) = 180(20) + 220(0) = 3600 B(10, 15)
C(0,18)
Z at B(10, 15) = 180(10) + 220(15) = 5100 10
A(20, 0)
Z at C(0, 18) = 180(0) + 220(18) = 3960 X X
O 10 20 30 40 50 60
 Z has maximum value 5100 at B(10, 15) 3x + 10y = 180
 Z is maximum when x = 10, y = 15

Y 6x + 4y = 120

Thus, the company should manufacture 10 bicycles and 15 tricycles to gain maximum profit
of `5100. [1]
iii. The given combined equation of lines is
ax2 + 2hxy + by2 = 0
Let m1 and m2 be the slopes of the lines represented by ax2 + 2hxy + by2 = 0
2h a
 m1 + m2 =  and m1m2 = , b  0
b b
a
If = 1, then m1m2 = 1.
b
 lines are perpendicular. [1]
a
So we assume that  1
b
Y
y = m2x
y = m1x

X X
O

Y
8
Mathematics
Now, (m1  m2)2 = (m1 + m2)2  4m1.m2
2
4a
=    
2h
 b  b
4h 2 4a
= 2 
b b
4h 2  4ab 4  h  ab 
2

 (m1  m2)2 = =
b2 b2
Taking square root on both the sides, we get
2 h 2  ab
m1  m2 =  [1]
b
Let  be the acute angle between the lines.
2 h 2  ab
m1  m 2  a
 tan  = = b ,  1
1  m1 .m 2 a b
1
b
2 h 2  ab
 tan  = ,a+b0 [1]
ab
The given pair of lines are x2 – 4xy + y2 = 0
Comparing with ax2 + 2hxy + by2 = 0, we get
a = 1, h = –2, b = 1
2 h 2  ab 2 4 1
tan  = 
ab 11

2 3
=
2

 tan  = 3

  = 60 or [1]
3

SECTION – II
Q.4. (A) Select and write the correct answer from the given alternatives in each of the following
sub-questions:
i. (D)
 2x , x < 0
f(x) = 
 0, x  0
lim f(x) = lim 2x = 0
x 0  x 0

lim f(x) = lim 0 = 0


x  0  x 0

and f(0) = 0
 lim f(x) = lim f(x) = f(0)

x 0 x  0

Hence, f(x) is continuous at x = 0.


Now we find left hand derivative and right hand derivative of f(0) at x = 0
Right hand derivative at x = 0
9
Board Answer Paper : July 2017

f  0 + h   f  0 00
i.e., f (0+) = lim = lim =0
h 0 h h 0 h
Left hand derivative at x = 0
f  0 + h   f  0 h 0
i.e., f (0) = lim = lim
h 0 h h 0 h
h
= lim = 1
h  0 h

 f (0+)  f (0)
Hence, f(x) is not differentiable at x = 0. [2]
ii. (B)

  3x 
2
 2 x  1 dx = 14
0

  x3  x 2  x  = 14
 0
 3 + 2 +  – 14 = 0
 ( – 2) (2 + 3 + 7) = 0
But 2 + 3 + 7 = 0 does not have real roots
 =2 [2]

iii. (A)
f (x) = x3 – 3x2 + 3x – 100, x  R
 f (x) = 3x2 – 6x + 3
= 3(x2 – 2x + 1)
= 3(x – 1)2
Since, (x – 1)2 is always positive x ≠ 1
 f (x) > 0 for all x  R, x ≠ 1
Hence, f (x) is an increasing function, for all x  R, x ≠ 1 [2]

(B) Attempt any THREE of the following:


i. Let u = 3x
du
  = 3x log 3
dx
log x
Let v = log3 x =
log 3
dv 1 d
 = (log x)
dx log 3 dx
1 1 1
=  = [1]
log 3 x x log 3
æ du ö÷
çç ÷
du çè dx ÷ø 3x log 3 x
 = = = 3 x (log 3)2 [1]
dv æ dv ö÷ 1
çç ÷
çè dx ÷ø x log 3

10
Mathematics
ii. f(x) = (x  1)(x  2)(x 3), x  [1, 3]
3 2
= x  6x + 11x  6
As f(x) is a polynomial function, it is continuous and differentiable everywhere on its domain.
Thus,
a. f(x) is continuous on [1, 3]
b. f(x) is differentiable on (1, 3) [1]
Further, f(1) = 0 and f(3) = 0
 f(1) = f(3)
Thus, all the conditions of Rolle’s theorem are satisfied. [1]

tan x
tan x cos 2 x
iii. Let I =  dx =  sin x  cos x dx
sin x  cos x
cos 2 x
sec 2 x tan x
=  tan x dx [1]

sec 2 x
=  tan x
dx

 f (x) 
 I = 2 tan x + c ….  

 f ( x)
dx  2 f ( x )  c 

[1]

iv. Y

y=6

y=2

X X
O
Y
y varies from y = 2 to y = 6.
Equation of parabola x2 = 16y
 x=4 y
b
Required area =  xdy
a
6
=  4 y dy [1]
2
6
 3
 y2 
= 4 
3
 
 2  2

2  32 3

=4  (6)  2 2

3 
8  32 3

=  6  2 2
 sq. units [1]
3 
11
Board Answer Paper : July 2017
v. Given, n = 10, p = 0.4
q = 1 – p = 1 – 0.4 = 0.6
Now, E(X) = np = 10  0.4 = 4 [1]
Var(X) = npq = 10  0.4  0.6 = 2.4 [1]

Q.5. (A) Attempt any TWO of the following:


i. f is continuous at x = 0.
 f(0) = lim f(x) [1]
x 0

5  1
2 2
sin x
 5sin x 1  sin 2 x
 5  1
sin x 2
   2
x 2
 sin x  x
 f(0) = lim = lim = lim [1]
x  0 x log 1  2 x  x  0 x  log 1  2 x  x0 log(1 2 x)
2
x 2 2x
2
 5sin x  1 sin x 
 lim  lim 
x  0 sin x x 0 x 
= 
log 1  2 x 
2 lim
x 0 2x
 log 5 
2

 f(0) = [1]
2

ii. E(X) =  x  P( x )
i i

= 0(0.08) + 1(0.15) + 2(0.45) + 3(0.27) + 4(0.05)


= 0 + 0.15 + 0.9 + 0.81 + 0.2
= 2.06 [1]

E(X2) = x 2
i  P( xi )
= 0 (0.08) + 12(0.15) + 22(0.45) + 32(0.27) + 42(0.05)
2

= 0(0.08) + 1(0.15) + 4(0.45) + 9(0.27) + 16(0.05)


= 0 + 0.15 + 1.8 + 2.43 + 0.8
= 5.18 [1]
Var (X) = E(X2)  [E(X)]2
= 5.18  (2.06)2
= 5.18  4.2436
= 0.9364 [1]
iii. X = Number of families who own a television set.
P = Probability of families who own a television set.
80 4
P = 80% = =
100 5
4 1
q=1–p=1– =
5 5
 4
Given, n = 5, X  B  5, 
 5
The p.m.f. or X is given as
P(X = x) = nCx px qn  x
= nCx (p)x (q)5  x [1]
12
Mathematics
a. P(families own television set)
= P(X = 3)
3 53
4 1
= 5C3    
 5 5
128
=
625
= 0.2048 [1]
b. P(At least two families own television set)
= P(X  2) = 1  P(X < 2)
= 1 – [P(X = 0) + P(X = 1)]
 5  4 0  1 5 5  4 1  1 4 
= 1   C0      C1      
  5 5  5   5  
 1 20 
=1–   
 55 55 
21 3104
=1–  [1]
3125 3125
(B) Attempt any two of the following:
i. Let f(x) = cos x
 f (x) = – sin x [1]

1
x = 60 30 = 60 + =a+h
2

Here, a = 60 =
3

1 0.0175
and h = = = 0.00875
2 2
  1
f(a) = f   = cos   = = 0.5
3 3 2
 
f (a) = f    = –sin   = – 0.8660 [1]
3 3
f(a + h)  f(a) + hf(a) [1]
 cos(60 30)  0.5 + (0.00875) (– 0.8660)
 0.5 – 0.0075775
 0.4924 [1]
ii. Let ‘N’ be the number of bacteria at time’t ’
dN
 N
dt
dN
 = kN
dt
dN
 = kdt
N
Integrating on both sides, we get
dN
 N
= k  dt
 log N = kt + c [1]
when t = 0, N = 1000
c = log 1000 [1]
13
Board Answer Paper : July 2017
 log N = kt + log 1000
 N 
 log   = kt
 1000 
 N = 1000 ekt ….(i)
when t = 1 hour, N = 2000
 ek = 2 [1]
 N = 1000  (2)t ….[From(i)]
1
when t = 2 hours, we get
2
5
N = 1000   2  2
= 1000  4  2 = 4000  1.414
N = 5656
1
Number of bacteria present after 2 hours is 5656. [1]
2
iii. Since, – a < 0 < a
a 0 a
  f ( x)dx =
a
 f ( x)dx +  f ( x)dx
a 0
[1]
a a
  f ( x)dx = I +  f ( x)dx
a 0
….(i)
0
Now, I =  f ( x)dx
a

Put x = t
 dx =  dt
When x = a, t = a and when x = 0, t = 0
0 0
 I =  f ( t)(dt) =   f ( t)dt
a a
a
 b a

=  f ( t)dt ….   f ( x)dx    f ( x)dx 
0  a b 
a
 b b

=  f ( x)dx ….   f ( x)dx   f (t)dt 
0  a a 
 Equation (i) becomes
a a a


a
f ( x)dx =  f ( x)dx +  f ( x)dx
0 0
a
=  f ( x)  f ( x) dx
0
….(ii) [1]

Case 1: If f(x) is an even function, then f(x) = f(x).


Thus, equation (ii) becomes
a a a

 f ( x)dx =
a
 f ( x)  f ( x) dx = 2  f ( x)dx
0 0
[1]

Case 2: If f(x) is an odd function, then f(x) = f(x).


Thus, equation (ii) becomes
a a

 f ( x)dx =
a
  f ( x)  f ( x) dx = 0
0
[1]

14
Mathematics
Q.6. (A) Attempt any TWO of the following:
i. Since f is continuous on [4, 2],
 f is continuous on x = 2
 lim f  x   lim f  x  [1]
x 2 x 2

 lim 6b  3ax  lim 4 x +1


x 2 x 2

 6b  3a(2) = 4(2) + 1 [1]


 6b + 6a = 7
 6(a + b) = 7
7
 a+b= [1]
6
ii. Let  v dx  w
dw
 =v
dx
d dw du
Consider, (uw) = u +w [1]
dx dx dx
du
= uv +  v dx 
dx
du
dx 
= uv + v dx
Integrating on both sides w.r.t. ‘x’, we get
 du 
uw =  u vdx     v dx  dx [1]
 dx 
 du 
 u  v dx=  uv dx     v dx  dx
 dx 
 d 
  u  v dx = u  v dx    dx (u)   v dx  dx [1]

iii. By definition cummulative distribution function at x is


P(x ≥ 2) = 0.3 + 0.4 + 0.2 = 0.9 [1]
f (xi) = P1 + P2 + P3 + ……. + Pi where, i = 1, …, x
Thus f (x1) = P1 = 0.1
f (x2) = P1 + P2 = 0.1 + 0.3 = 0.4
f (x3) = P1 + P2 + P3 = 0.1 + 0.3 + 0.4 = 0.8
f (x4) = P1 + P2 + P3 + P4 = 0.1 + 0.3 + 0.4 + 0.2 = 1 [1]
4
 f (x4) = p
i 1
i =1

 Cumulative distribution function of X is


X=x 1 2 3 4
F(x) 0.1 0.4 0.8 1
[1]
(B) Attempt any TWO of the following:
dy
i.  y = ex
dx
dy
The given equation is of the form + Py = Q
dx
Where, P = –1 and Q = ex
I.F. = e   e 
pdx 1 dx
 e x [1]
15
Board Answer Paper : July 2017
 Solution of the given equation is
y (I.F.) =  Q (I.F.) dx + c
 y. e  x =  e x .e x dx  c
 y e x = x + c [1]
put x = 0 and y = 1, we get
c=1 [1]
y. e  x = x + 1
y = (x + 1) ex is a particular solution of D.E. [1]
ii. ‘y’ is a differentiable function of ‘x’.
Let there be a small change x in the value of ‘x’.
Correspondingly, there should be a small change y in the value of ‘y’.
As x  0, y  0
x y
Consider,  =1
y x
x 1 y
 = , 0
y  y x
x
Taking lim on both sides, we get
x  0

 x  1
lim   = [1]
x  0
  lim  y 
y
 
x  0 x
 
Since ‘y’ is a differentiable function of ‘x’,
 y  dy
lim   =
x  0 x
  dx
As x  0, y  0
 x  1
lim   = ….(i)
 y  lim  y 
y  0
 
x  0 x
 

 limits on R.H.S. of (i) exist and are finite. [1]


Hence, limits on L.H.S. of (i) also should exist and be finite.
 x  dx
 lim   = exists and is finite.
y  0 y
  dy
dx 1 dy
 = , 0 [1]
dy  dy  dx
 
 dx 
 
y = sin1 x, 1 ≤ x ≤ 1,   y 
2 2
 x = sin y
Differentiating w.r.t. y, we get
dx
= cos y
dy

16
Mathematics
dy 1
 =
dx cos y
dy 1
 =
dx  1  sin 2 y
dy 1
 =
dx  1  x2
 
Since   y  , y lies in I or IV quadrant.
2 2
 cos y is positive.
dy 1
 = , x < 1 [1]
dx 1  x2

8
iii. Let I =   x  2  x 2
 4
dx

8 A Bx + C
Let =  2 [1]
 x  2  x 2
 4  x  2  x  4
 8 = A (x2 + 4) + (Bx + C) (x + 2)
 8 = Ax2 + 4A + Bx2 + 2Bx + Cx + 2C
 8 = (A + B) x2 + (2B + C) x + (4A + 2C)
Comparing the coefficients of x2 , x and the constant term, we get
A + B = 0, 2B + C = 0 and 4A + 2C = 8
On solving these equations, we get
A = 1, B = –1, C = 2 [1]
8 1 x + 2
 =  2
 x  2  x  4  x  2  x  4
2

 1 x  2 
I =   2  dx
x  2 x  4
1 1 2x 1
=  dx   2 dx  2 x dx [1]
x2 2 x 4   2
2 2

1  x
= log x  2 – log x 2  4 + tan–1   + c
2 2
x2  x
= log  tan 1    c [1]
x 4
2
2

17
Biology

BOARD ANSWER PAPER : MARCH 2016


BIOLOGY
SECTION – I
[BOTANY]
Q.1. Select and write the most appropriate answer from the given alternatives for each sub-
question:
i. (B) 1 : 2 : 1 [1]

ii. (C) 68 [1]

iii. (B) Saccharomyces [1]

iv. (D) Anthocyanin [1]

v. (A) UAG [1]

vi. (D) Acetyl - Co – A [1]

vii. (B) 5th June [1]

Q.2. (A) Answer in ‘One’ sentence only :


i. When F1 hybrid is back crossed with its recessive parent, it is called ‘test cross’. [1]
ii. The pathogenic fungi used to selectively kill weeds or inhibits the growth of weeds is known
as ‘mycoherbicide’. [1]

iii. A set of three exposed nitrogen bases present on anticodon loop of tRNA is called
anticodon. [1]

iv. The process of formation of dark coloured amorphous substance known as humus, which
is a partially decomposed organic matter, is known as humification. [1]

v. The bubbles of CO2 trapped in glutein makes idlies puffy. [1]

vi. Ecological succession is the gradual (and predictable) change in the species composition
of a given area. [1]
(B)
Ribosome
Outer membrane
. Inner membrane Peristromium
. . .. . . . .
. .. . .. . Stroma
. DNA
. . . . . . . ... Stroma lamellae
. . .. Thylakoids
. Granum
Chloroplast [Proportionate diagram] [½]
[Any three labels: ½ mark each] [1½]
[Labels: Outer membrane, inner membrane, grana, stroma, stroma lamellae, thylakoids]
1
Board Answer Paper : March 2016
(C) Answer the following (Any TWO):
i. a. The process of induction and utilization of mutation for development of new crop
varieties having desirable traits is known as mutational breeding. [½]
b. Mutational breeding helps in producing disease resistant varieties. [½]
c. Plants are produced by inducing mutations. Chemicals or physical mutagens are used
for bringing about mutation. [½]
d. Varieties of moong beans resistant to yellow mosaic virus and powdery mildew have
been developed by the technique of mutational breeding. [½]
ii. Following are the advantages of Biogas:
a. Biogas is a cheap, safe and renewable source of energy.
b. Biogas can be burnt in gas stoves to provide heat.
c. It can be used for cooking, domestic lighting, street lighting.
d. It burns with a blue flame and without smoke.
e. It is eco-friendly and does not cause pollution.
f. It can be used for driving engines.
g. It helps to improve sanitation of the surrounding.
h. It can be easily generated, stored and transported.
i. The residue left after biogas production can be used as manure.
[Any four points: ½ mark each] [2]
iii. Carbon cycle:
i. The carbon enters the biotic components of ecosystem through green plants,
photosynthetic bacteria and cyanobacteria in terrestrial ecosystem and through
phytoplanktons and hydrophytes in the aquatic ecosystems.
These are called producers and carry out photosynthesis by taking in the atmospheric
CO2 and make carbohydrates and oxygen.
ii. Carbohydrates are used as source of food by animals. Thus, carbon fixed by producers
enters the food chain and keeps moving through different living organisms. They are
returned to the soil in the form of detritus.
iii. The decomposers breakdown the larger organic compounds into simpler ones and then
into inorganic molecules. In this process, small amount of CO2 is returned back to
atmosphere during enzyme catalyzed breakdown of dead matter by decomposers.
iv. CO2 is released into atmosphere by respiration of producer and consumers.
v. Burning of fossil fuels in automobiles and machineries to produce energy, burning of
wood, organic debris also release CO2 in atmosphere.
vi. Volcanic eruption and hot springs also release CO2 into atmosphere. [1]

CO2

Sugars,
Starches
Fossil
Fuels

Decomposition

Carbon Cycle
[1]
2
Biology
iv. a. Flowers are large, stout enough so that bats can hold on to the flowers.
b. Flowers open during night.
c. Flowers emit rotten fruit odour.
d. Flowers have large number of stamens.
Flower produce a considerably large quantity of pollen grains.
[Any four points: ½ mark each] [2]

Q.3. (A) Answer the following (Any TWO):


i. a. When a single gene controls two (or more) different traits, it is called pleiotropic gene
and this phenomenon is called pleiotropy or pleiotropism.
The ratio is 2:1 instead of 3:1. [½]
b. According to Mendel’s principle of unit character, one gene (factor) controls one
character (trait), but sometimes single gene produces two related or unrelated
phenotypic expressions.
c. For example, the disease, sickle cell anaemia is caused by a gene Hbs. Normal or
healthy gene is HbA and is dominant. [½]
A S
d. The carriers (heterozygotes – Hb /Hb ) show signs of mild anaemia as their RBCs
become sickle-shaped (half-moon shaped) in oxygen deficiency. They are said to have
sickle-cell trait and are normal in normal conditions.
e. The homozygotes with recessive gene Hbs however, die of fatal anaemia.

Phenotype of parents Sickle-cell carrier  Sickle-cell carrier [1]


Genotype A
Hb Hb S A
Hb Hb S

Gametes HbA HbS HbA HbS

Genotype of offsprings HbAHbA HbAHbS HbSHbA HbSHbS

Phenotype of offsprings Normal Sickle-cell carriers Sickle-cell Anaemics


1 2 1(dies)
Graphical representation of pleiotropy

f. Thus, the gene for sickle-cell anaemia is lethal in homozygous condition and produces
sickle cell trait in heterozygous carrier.
g. Two different expressions are produced by a single gene.
h. A marriage between two carriers will produce normal, carriers and sickle-cell anaemic
children in 1:2:1 ratio. But, sickle-cell anaemics who are homozygous for gene HbS
will die, as HbS is a lethal gene causing death of the bearer. [1]
Thus the sickle cell anemics die leaving carriers and normals in the ratio 2:1.

3
Board Answer Paper : March 2016
ii.
Pyruvic acid (3C)
NAD
CO2 NADH2
Acetylation Acetyl (2C)

Co-A

Acetyl Co-A (2C)


H2O
Co-A

Citric acid (6C)


H2O
Oxaloacetic acid (4C)
NADH2 Cis-aconitic acid (6C)
H2O
NAD

Malic acid (4C)


H 2O Isocitric acid (6C)
NAD

NADH2
Fumaric acid (4C)

FADH2 Oxalosuccinic acid (6C)

FAD
Succinic acid (4C) CO2
Co-A
H2O -ketoglutaric acid (5C)
Co-A
GTP GDP
(1ATP) Succinyl Co-A CO2
ADP ATP
NADH2
NAD
TCA Cycle/ Krebs Cycle
[Correct schematic representation] [3]
iii.
No. Cyclic Photophosphorylation Non-cyclic Photophosphorylation
i. Electrons emitted by chlorophyll return The electrons emitted by chlorophyll do not
back to the same chlorophyll. return back to the same chlorophyll.
ii. NADPH2 is not formed. NADPH2 is formed.
iii. Does not involve photolysis of H2O. Involves photolysis of H2O.
iv. First electron acceptor is FRS. First electron acceptor is CO-Q.
v. No evolution of O2. There is evolution of O2.

4
Biology
vi. Less efficient and less significant. More efficient and significant process.
vii. Only Photosystem-I (P700) is involved in Both Photosystem PS-I (P700) as well as PS-II
this cycle. (P680) are involved.
viii. It operates under low light intensity, It takes place under optimum light, aerobic
anaerobic conditions, poor availability of conditions and in the presence of sufficient CO2.
CO2.
ix. Found in photosynthetic bacteria. Absent in photosynthetic bacteria.
[Any six points:½ mark each] [3]
(B) Perfect pairing
m-RNA
G U U
C A A

t-RNA
[1]
5 G C
C
A

Wobble pairing

G U C G U A G U G
m-RNA
C A A C A A C A A

t-RNA
5 G C 5 G C 5 G C
C C C
A A A [2]

Wobble Hypothesis
Q.4. Definition:
The fusion of one male gamete with egg and that of another male gamete with secondary
nucleus is called as double fertilization. [1]
It is the characteristic feature of angiosperms. It was discovered by Nawaschin (1897) in Lilium
martagon plant.
It consists of two processes:
a. Syngamy: [½]
It is the fusion of first male gamete with egg. It results in the formation of diploid zygote
which develops to form embryo. It is also called generative fertilization.
b. Triple Fusion: [½]
It is the fusion of second male gamete with secondary nucleus. It results in the formation
of triploid PEN (Primary Endosperm Nucleus) which develops to form endosperm.
Since both male gametes participate in fertilization, it is called double fertilization.
Process of double fertilization is described as follows:
i. After pollination, the intine of the pollen grain forms pollen tube and passes through the germ
pore.
ii. The growth of pollen tube is stimulated by the sugary substance produced on the stigma.
iii. The pollen tube with two male gametes and tube nucleus runs through the style and
finally turns towards the micropylar end of the ovule in the cavity of the ovary.
5
Board Answer Paper : March 2016
iv. The length of the pollen tube depends on the length of style.
v. When the pollen tube enters through the micropylar end of the ovule for fertilization, it is
called porogamy (sometimes it may enter through integuments and called as mesogamy
or sometimes through chalaza and called as chalazogamy). [½]
vi. Filiform apparatus of synergids attract the pollen tube towards egg apparatus.
vii. As the pollen tube elongates, it carries with it two haploid, non-motile male gametes and
hence, the fertilization is also called siphonogamy (siphon = tube). [½]
viii. On piercing the nucellus, the pollen tube penetrates the embryo sac. Its tip penetrates the
embryo sac and reaches the egg apparatus passing either between the egg and synergids
or between one synergid and wall of embryo sac.
ix. Ultimately, the tip of the pollen tube bursts and two male gametes are released.
x. The tube nucleus degenerates before bursting of the pollen tube.
xi. One of these male gametes fuses with the egg cell or oosphere causing fertilization, as a
result of which diploid oospore or zygote is formed. This is called first fertilization or
syngamy. [½]
xii. The other male gamete fuses with the secondary nucleus forming the triploid endosperm
nucleus which later on gives rise to endosperm. This is called as triple fusion or second
fertilization. [½]
xiii. Thus, this process of fertilization which occurs twice in the same embryo sac at a time by
two male gametes (syngamy and triple fusion) is called double fertilization.
Pollen grain (Germinating)

Stigma

Pollen tube
Antipodal cells

Style Male gamete


Polar nuclei
Ovary
Male gamete
Chalaza
Egg
Antipodal cells
Synergids
Secondary nucleus
Egg Pollen tube
Synergids
Male gametes Micropyle Embryo sac
[Proportionate diagram: 1 mark]
[Any four labels: ½ mark each] [3]
Double fertilization [Labels: Pollen tube, micropyle, male
gamete, secondary nucleus, egg
apparatus, antipodal cells, chalaza]

OR
a. It is defined as the technique of manipulating the genome of a cell or organism so as to
change the phenotype desirably. [1]
b. Manipulation of the genome involves addition, removal, replacement or repair of a part of
genetic material that results into the desirable phenotype change.
The following steps are involved in genetic engineering or r-DNA technology:
i. Isolation of desired gene:
The donor individual having desired gene is selected. [½]

6
Biology
ii. Fragmentation of desired gene:
From the DNA of this donor, desired gene is selected and isolated with the help of restriction
endonuclease enzyme. The donor DNA containing the desired gene is called passenger DNA. [½]
iii. Selection of vector:
A vector DNA (usually plasmid DNA or Phage DNA) is selected. [½]
iv. Formation of recombinant DNA:
The vector DNA is cleaved at a specific point using restriction endonuclease enzyme. The cut
ends of vector DNA are sticky, i.e. cohesive.
The desired gene is now ligated with the vector DNA using ligase enzyme.
The vector DNA containing a new introduced gene is called recombinant DNA (r-DNA) or
chimeric DNA (Chimeric vector or chimeric plasmid). [½]
v. Gene transfer to the host:
This chimeric plasmid is introduced into bacterial cell (Host cell). e.g. E.coli for cloning.
Bacillus subtilis and Saccharomyces cerevisiae can also be used as host.
Such a bacterial cell with chimeric or r-DNA is called transformed host.
The incorporation of r-DNA into bacterial cell is generally done by electroporation. [½]
vi. Cloning:
The transformed bacterial cell is now allowed to grow on the nutrient medium where it multiplies
rapidly.
It results in the formation of a large number of transformed bacterial cells.
All these cells have a copy of recombinant or chimeric DNA.
Generally, after introduction of r-DNA or chimeric plasmid in the host cell, amplification is done.
It is a process in which the number of r-DNA in a bacterial cell is increased. [½]
Therapeutic products made by recombinant DNA technique
No. Therapeutic product Examples
i. Blood proteins Erythropoietin; Factors VII, VIII, IX, Tissue plasminogen
activator, Urokinase
ii. Human hormones Epidermal growth factor, Follicle stimulating hormone,
Insulin, Nerve growth factor, Relaxin, Somatotropin
iii. Immune modulators Interferon, interferon, Colony stimulating factor,
Lysozyme, Tumor necrosis factor
iv. Vaccines Cytomegalovirus, Hepatitis B, Measles, Rabies.
[Any three points: 1mark each] [3]
SECTION – II
[ZOOLOGY]
Q.5. Select and write the most appropriate answer from the given alternatives for each sub-
question:
i. Metacentric [1]
ii. Tissue Growth Factor. [1]
iii. Rh–ve [1]
iv. atherosclerosis [1]
v. intra-specific struggle [1]
vi. Volant adaptation [1]
vii. in-breeding [1]
7
Board Answer Paper : March 2016
Q.6. (A) Answer in ‘One’ sentence each:
i. Visit of veterinary doctor is mandatory for checking health problems of cattles, diseases of
cattles and their rectification.
[Any two points: ½ mark each] [1]

ii. Aquatic animals can afford to be ammonotelic because large quantity of water is required to
eliminate ammonia and water is available in excess to them. [1]

iii. PUC is mandatory for all vehicles in order to check air pollution. [1]

iv. Organic evolution is a slow, gradual, continuous and irreversible changes through which
the present day complex forms have descended from their simple, pre-existing forms of the
past. [1]

v. The genotype of Turner’s syndrome is 44 + XO where 44 is the autosome number. [1]

vi. R.F.L.P. stands for restriction fragment length polymorphism. [1]


(B)
Tunica interna

Lumen

Tunica media
Tunica externa

T.S. of Vein [Neat and Proportionate diagram: ½ mark]


[Any three labels: ½ mark each] [2]
(C) Attempt any TWO of the following:
i. Following factors are responsible for genetic variation:
a. Gene mutation:
Changes in the chemical make up of a gene is called gene mutation or point mutation.
These mutations are the cause for new alleles that introduce variations in the gene pool
and it accounts for the change in gene frequency.
b. Gene flow:
Transfer of genes between populations that differ genetically from one another is called
gene flow.
c. Genetic recombination:
In sexually reproducing organisms, during gametogenesis, the homologous
chromosomes exchange genetic material by the process of crossing over.
This produces new combinations and the phenomenon is called genetic recombination.
It adds variability to individuals.
d. Genetic drift:
Any alteration in allele frequency, in the natural population by pure chance is called
genetic drift. For e.g. elimination of a particular allele from a population due to events
like accidental death prior to mating of an organism, i.e. the sole possessor of particular
allele. Smaller populations have greater chances for genetic drift. It is also called
Sewall Wright effect.
8
Biology
e. Chromosomal aberrations:
It refers to the structural alterations in a chromosome causing changes in the gene
arrangement. e.g., deletion, duplication, translocation and inversion.
They also cause variations in the Mendelian populations.
[Any two points: 1 mark each] [2]
ii. A: Acrosome [½]
Function: It secrets an enzyme hyaluronidase which helps on the penetration of egg during
fertilization. [½]
B: Tail piece or flagellum [½]
Function: It provides motility to the sperm (helps its movements in the female genital tract). [½]
iii. Artificial acquired active immunity:
a. Artificial acquired active immunity is acquired artificially by vaccination.
b. Vaccines contain dead or live the but attenuated pathogens or toxoids consisting of
microbial components or toxins secreted by pathogens.
c. Vaccines are introduced into the body to stimulate the formation of antibodies by the
immune system.
d. Example: polio vaccine, BCG vaccine.
[Any two points:1 mark each] [2]
iv. Economic importance of fisheries:
a. It is a source of employment for many people.
b. It provides nutrient food, as fishes are rich in proteins, vitamins (A, D and E),
carbohydrates, fats and minerals.
c. It promotes allied business like manufacturing of crafts and gears and also provides raw
material to other industries.
d. Oil extracted from the body of fishes has medicinal as well as commercial value. e.g.
Shark liver oil, cod liver oil (medicinal value) and oil extracted from sardine and
mackerel (commercial value).
e. The waste parts of fishes are used to prepare the fertilizers and fish manure.
f. Fishes yield number of byproducts such as fish meal, Isinglass, fish glue, fish flour, etc.
g. Fishery is a good source of foreign exchange. Prawns and lobsters have market all over
the world.
[Any four points: ½ mark each] [2]
Q.7. (A) Attempt any TWO of the following: (6)[9]
i. Sex Determination in human being:

Parents:

Gametes:

Chromosomes:

Offsprings:
[Chart: ½ mark]
[Labels: 1 mark] [11/2]

9
Board Answer Paper : March 2016
Human diploid cell has 46 chromosomes, i.e. 23 pairs in each cell.
Out of these, 22 pairs of chromosomes are called autosomes.
Autosomes determine all body characters like colour of hair, skin, colour of eyes, height, etc.
Out of 23 pairs, one pair of chromosomes is called sex chromosomes.
They are X and Y chromosomes. [½]
Every individual gets one set of chromosomes from his mother and one from his father.
A human male thus has 44 +XY chromosomes, whereas a female has 44 + XX chromosomes.
During gamete formation; meiosis or reductional division takes place and a gamete gets only
one set of chromosomes and thus it is haploid.
e.g. Female gamete (ovum) 22 + X, Male gamete (sperm) 22 + X or 22 + Y. [½]
When the male and female gametes unite to form a zygote, the chromosomes again become
diploid. Thus, the offspring gets the same number of chromosomes as his parents. Sex of the
baby is determined by the nature of sperm (X or Y) that fertilizes the ovum. Thus, in human
being, it is the male which determines the sex of the baby. [½]
ii. Steady population
The relative proportion of various age groups in population is called the age structure. It
determine the reproductive status of a population. [½]
The three major age groups are:-
a. Pre  reproductive group ( 0 to 14 years)
b. Reproductive group (15 to 59 years)
c. Post  reproductive group ( age 60 years and above) [½]
The age structure determines the trend of the population. A population is said to be steady,
when it has same number of pre  reproductive and post  reproductive age groups. It is a
stable population. [½]

Post
reproductive

Reproductive

Pre 
reproductive

[Pyramid :1½ mark]


Pyramid of steady population
iii. a. Tissue plasminogen activator (TPA) is used to prevent or reverse blood clots.
b. Human Growth Hormone producer gene helps treat pituitary dwarfism.
c. Human blood clotting factor VIII is used to treat hemophiliacs.
d. Human insulin (HUMULIN) to treat diabetes (insulin dependent diabetes)
e. Tissue growth factor  Beta promotes epidermal growth and formation of new blood
vessels. Used in treatment of burns and for wound healing.
f. DNase is used to treat cystic fibrosis.
g. Bovine growth hormone helps to increase cattle and dairy yield.
h. Recombinant vaccines for prophylaxis of human and animal viral diseases (hepatitis
B).
i. Genetically engineered bacteria helps in enhanced production of industrial enzymes,
citric acid and ethanol.
[Any three points: 1 mark each] [3]
10
Biology
(B) L.S. of Kidney:

Renal cortex Renal capsule


Renal medulla
Renal pyramids

Renal column
of Bertini Renal pelvis
Major calyx Ureter
Minor calyx

V.S. of Human kidney [Proportionate diagram: 1 mark]


[Any four labels: ½ mark each] [3]

Q.8. Gametogenesis is the process of formation of gametes in sexually reproducing animals. [1]
It includes two process: spermatogenesis and oogenesis.
Spermatogenesis:
i. Spermatogenesis is the process of formation of haploid, microscopic and motile male
gametes, called spermatozoa, from the diploid spermatogonia of the testis of male organism.

Spermatogonium (46)

Proliferation

Growth Primary spermatocyte (46)

Secondary spermatocyte (23)

Maturation

Spermatid (23)

Transformation
Sperm (23)

Spermatogenesis
[Diagrammatic representation] [11/2]

11
Board Answer Paper : March 2016
Each testis has seminiferous tubules which are lined by cuboidal epithelium called germinal
epithelium.
Germinal cells in testes are known as primary germinal cells. Primordial cells passes through
three phases, namely:
i. Multiplication phase: Primordial cells undergo repeated mitotic divisions to produce
large number of spermatogonia.
Each spermatogonium is diploid (2n). [½]
ii. The Growth phase: Spermatogonium cell accumulates food and grows in size. Now, it
is called primary spermatocyte. [½]
iii. The Maturation phase: The primary spermatocyte undergoes first meiotic or
maturation division. The homologous chromosomes start pairing.
Each homologous chromosome splits longitudinally. Chiasma formation results in
exchange of genetic material.
At the end of 1st meiotic division, two haploid, secondary spermatocytes are formed.
Each secondary spermatocyte undergoes 2nd meiotic division and produces spermatids.
So, at the end of maturation phase, each spermatogonium produces four haploid
spermatids.
Spermatid is non-motile, so it has to undergo spermiogenesis to become functional,
motile male gamete, i.e. spermatozoan. [½]

Oogenesis is the process of formation of haploid female gametes, called ova, from the diploid
oogonia of the ovary of female organism. It is completed in three stages:
i. Multiplication phase
ii. Phase of Growth
iii. Maturation Phase

Germ Cell (2n)

Mitosis

Oogonia

Primary
Oocyte
Meiotic Division

Secondary
oocyte

1 Ovum (n) 3 Polar Bodies (n)


Oogenesis [Diagrammatic representation] [1½]
12
Biology
i. Multiplication phase: Germinal cells undergo mitosis to form large number of
oogonia. Oogonia in human beings are formed in the ovary of a female baby even
before her birth. [½]
ii. Phase of Growth: Just before puberty, under the influence of follicle stimulating
hormone, one of the oogonium grows in size. Growth in size of oogonium is larger
than that seen in spermatogenesis. This grown up cell is called primary oocyte. [½]
iii. Maturation phase: Primary oocyte undergoes maturation or meiotic division.
Meiotic I division of primary oocyte shows equal nuclear division but unequal
cytoplasmic division.
So, at the end of meiosis I division, large sized haploid secondary oocyte and haploid
small sized polar body are formed.
Unequal division is meant for sufficient supply of food for developing embryo.
Secondary oocyte and polar bodies undergo 2nd meiotic division.
2nd meiotic division is arrested at metaphase stage and secondary oocyte is released
from ovary. This division is also unequal. Remaining part of division is completed at
the time of fertilization.
This ovum is ready for fertilization. [½]
OR
Cerebrum (Telencephalon):
Frontal lobe
Central sulcus
Cerebrum Parietal lobe
Lateral sulcus
Parieto
occipital sulcus
Occipital lobe
Temporal lobe
Pons Varolii
Medulla
oblongata Cerebellum
Spinal cord

[Neat and Proportionate diagram: 1 mark]


[Any four labels: ½ mark each] [3]
It is the largest part of the brain accounting for 80-85% of its weight.
It is divided into cerebral hemispheres (right and left) by a median longitudinal cerebral
fissure.
Two cerebral hemispheres are connected by a single thick bundle of nerve fibres called
corpus callosum.
It is the largest commissure in human brain.
It connects both the hemispheres and helps in co-ordination.
The outer part of cerebrum is called cortex, while the inner part is called medulla. [1]
Cerebral cortex is formed by neurons cell bodies which appear grey and hence are called grey
matter.
Cerebral medulla is mainly formed of white matter (axons of nerve cells).
Deep within the white matter, certain masses of grey matter are located which are called basal
nuclei.
Thick dorsal wall (roof) of cerebrum is called pallium and the ventrolateral wall is known as
corpora striata.
Cerebral cortex shows number of ridges called gyri and depressions called sulci.
The gyri increase the surface area of cerebral cortex.
13
Board Answer Paper : March 2016
There are three deep sulci on the cerebrum, namely Central sulcus, Lateral sulcus and
Parieto-occipital sulcus. [1]
These sulci divide each cerebral hemisphere into four lobes, viz. anterior frontal lobe,
middle parietal lobe, posterior occipital lobe, lateral temporal lobe.
The frontal and parietal lobes are separated by central sulcus, the parietal and temporal lobes
are separated by lateral sulcus, while the parieto-occipital sulcus separates parietal lobe from
occipital lobe. [1]
Functions of Cerebrum:
i. It controls all voluntary activities.
ii. It perceives sensory stimuli through vision, taste, smell, sound, touch and speech.
iii. It is a centre of memory, will power, intelligence, reasoning and learning.
iv. It is a centre for micturition, defaecation, weeping and laughing.
v. It is also a centre for emotions, thoughts and feelings like pain, pleasure, fear,
fatigue, pressure, temperature, etc.
[Any two functions : ½mark each] [1]

14
Biology

BOARD ANSWER PAPER : JULY 2016


BIOLOGY
SECTION – I
[BOTANY]
Q.1. Select and write the most appropriate answer from the given alternatives for each sub-
question:
i. (C) ii [1]
ii. (B) Bacillus thuringiensis [1]
iii. (C) Datura stramoneum [1]
iv. (B) Vitamin B [1]
v. (D) 250 [1]
vi. (B) unidirectional [1]
vii. (C) triploid [1]
Q.2. (A) Answer in ‘One’ sentence only :
i. Bacteriophage is virus that infects the bacteria. [1]
ii. Emasculation is removal of anthers from young bisexual flower before it releases pollen
grains. [1]
iii. The branched vesicles forming the arbuscles of endomycorrhizae is known as VAM. [1]
iv. Decomposers act on dead organic matter releasing inorganic nutrients back in
environment by decomposing the complex compounds of the dead or living protoplasm. [1]
v. Edible mushroom are:
a. White button mushroom (Agaricus bisporus)
b. Paddy straw mushroom (Volvariella volvacea)
c. Oyster mushroom (Pleurotus florida)
[Any two names: ½ mark each] [1]
vi. Ecological succession:
The gradual (and predictable) change in the species composition of a given area that may replace
one population of species by another leading to change in physical environment is called
ecological succession. [1]
(B) Ultrastructure of mitochondrion.

F1 particles or oxysomes
Ribosome
Outer membrane
Matrix DNA
Cristae
Inner membrane

Mitochondrion [Proportionate diagram] [½]


[Any three labels: ½ mark each] [1½]
1
Board Question Paper : July 2016
(C) Answer the following (Any TWO):
i. Floral adaptations for anemophily:
a. Flowers are small, numerous, inconspicuous and not showy.
b. Inflorescence or flowers may be pendent so that they may swing.
c. Stamens possess long filaments and anthers are versatile.
d. Pollen grains are smooth, dry and light in weight.
e. Flowers bear well exposed stamens so that they can be easily dispersed by wind
currents.
f. Pollen grains are produced in large numbers to compensate their wastage.
g. To catch pollen grains, the stigmas become sticky, hairy, feathery or branched.
h. Flowers are devoid of scent, nectar, etc.
i. In unisexual flowers, male flowers are more than female flowers and are situated at
higher levels.
[Any four points: ½ mark each] [2]
ii.
No. Antibiotic Microbial source
i. Chloromycetin Streptomyces venezuelae
ii. Erythromycin Streptomyces erythreus
iii. Penicillin Penicillium chrysogenum
iv. Streptomycin Streptomyces griseus
[Name of any two antibiotics: ½ mark each] [1]
[Name of any two organisms: ½ mark each] [1]
iii. Helobial type of endosperm:
a. It is an intermediate between nuclear and cellular type of endosperm.
b. First division of primary endosperm nucleus is followed by (division of cytoplasm)
wall formation.
b. The wall formation results in the central cell being divided into a large micropylar and
a small chalazal chamber.
c. In both the chambers the development occurs as seen in nuclear type.
e. This type is generally seen in Helobiae series of monocots, hence the name ‘Helobial
Endosperm’.

[Description] [1½]
Helobial endosperm [Proportionate diagram] [½]
iv. Use of certain agrochemicals have been associated with some major environmental and
ecological damages. Such as:
a. The pesticides and weedicides are toxic, not only to target organisms but also to
many other non-target organisms, which are important components of the soil
ecosystem.
b. Continuous use of inorganic fertilizers changes the chemical nature of soil and
reduces its fertility.
c. In addition, the run off of agrochemical fertilizers into streams, lakes and ponds can
cause an increased productivity of those aquatic ecosystems causing eutrophication
and algal blooms in the water bodies.
d. The pesticide residues can remain in the soil for long and can enter the food chain
and undergo biomagnification.
2
Biology
e. Pesticides also enter the aquatic food chain. They get accumulated in the fatty tissues
of fishes as well as in birds which feed on them. This process is called as
bioaccumulation.
f. People handling pesticides in industries also suffer from various disorders like
respiratory diseases, nervous disorders, skin diseases, blindness, etc.
[Any four points: ½ mark each] [2]
Q.3. (A) Answer the following (Any TWO):
i. Incomplete dominance:
a. Incomplete dominance can be defined as a phenomenon in which neither of the alleles
of a gene is completely dominant over the other and hybrid is intermediate between the
two parents.
b. Incomplete dominance is a deviation of Mendel’s law of dominance which states that
out of two contrasting allelomorphic factors, only one expresses itself in an individual
in F1 generation called as dominant, while other which has not shown its effect is called
as recessive, however this recessive hidden character reappeared, unchanged in F2
generation.
c. Thus, according to incomplete dominance, F1 phenotype is intermediate between the
parental traits. Incomplete dominance is demonstrated in Mirabilis jalapa (four o’clock
plant) as given below:
Phenotype of Parents Red flower White flower
Genotype RR rr

Gametes R r

F1 generation Rr
Pink flower
Selfing of F1 Rr  Rr
generation
R r R r
Gametes
F2 generation
R r
R RR Rr
Red Pink
r Rr rr
Pink White [Graphical representation] [1]

Phenotypic ratio  1:2:1 (1 Red : 2 Pink : 1 White) [1/2]


Genotypic ratio  1:2:1 (1 RR : 2 Rr : 1 rr) [1/2]
This indicates the following facts:
a. Pink is the phenotype of the heterozygous genotype (Rr).
b. This pattern of inheritance is due to non-blending of the characters, because one-fourth
of the F2 progeny are red-flowered and another one-fourth are white-flowered, which
are the parental combinations.
c. The phenotypic and genotypic ratios are the same.
d. This type of observation has resulted from incomplete dominance of the alleles.
[Explanation] [1]
ii. The plants which are genetically engineered to carry desirable traits like disease resistance,
insect resistance, herbicide resistance are called as transgenic plants. [1]
Examples: Bt cotton, Golden rice, Flavr Savr tomato
3
Board Question Paper : July 2016
a. Bacillus thuringiensis (Bt) is a soil bacterium that produces a protein with insecticidal
property.
Bt toxin proteins occur as inactive protoxin.
The ‘cry’ gene codes for this inactive protoxin.
Crops have now been genetically engineered using rDNA technology, by inserting the
‘cry’ gene in plants.
After the insects ingest the transgenic crops, they are killed because the inactive
protoxin is converted into the active form due to alkaline pH of the insect gut.
Bt toxin gene has been cloned and introduced in many plants to provide resistance to
insects without the need of insecticides.
Bt cotton is commercially available to control the disease affecting the cotton balls.
b. Golden rice is genetically engineered rice with greater pro-vitamin A [ – carotene] content.
c. In Flavr Savr variety of tomato, the expression of a native gene that codes for the
enzyme polygalacturonase, has been blocked. The enzyme is responsible for
degradation of pectin during fruit ripening.
In the absence of this enzyme, the fruit ripening or softening is delayed and hence, the
fruit remains fresh for longer periods and flavour is saved.
[Any two examples: 1mark each] [2]
iii. Micropropagation:
It is a type of tissue culture technique by which large number of plants propagules are produced.
In this technique, shoot apical meristem is used as explants. Using the proper proportion of
growth hormones and tissue culture techniques many shoot apices can be produced.
These shoot apices are called Micropropagules. These are genetically identical and from
them, individual plants can be obtained. eg : Potato, banana and orchids. [11/2]
Somatic hybridization:
The protoplast from two different plants can be made to fuse by using fusogenic agent as
polyethylene glycol (PEG). From the resultant combined protoplast by tissue culture, a new
plant variety can be produced.
e.g. Pomato = Potato + Tomato
Raphanobrassica = Raphanus + Brassica [11/2]
(B) Degenerating Developing
megaspores Embryo sacs

Functional
megaspore
Functional 1st nuclear division 2nd nuclear division 3rd nuclear division
megaspore (2 nucleate embryo sac) (4 nucleate embryo sac) (8 nucleate embryo Sac)
Synergids
Egg (n) Egg
apparatus
Embryo sac Female
Polar nuclei Polar nuclei gametophyte

Antipodals (n)
Mature embryo sac
Shifting of nuclei
(Organisation of 7 celled and 8 nucleate embryo sac)
Development of the embryo sac or Female gametophyte [3]
4
Biology
Q.4. Give the central dogma of protein synthesis. Explain the process of translation.
i. Central dogma of molecular biology can be defined as unidirectional or one way flow of
information from DNA to mRNA (Transcription) and from mRNA to protein (Translation).
This can be represented as:

DNA  
Transcription
 mRNA 
Translation
 Protein

However, in some retroviruses, reverse transcription takes place due to which DNA is
synthesized from RNA. This can be represented as:
Transcription
 mRNA   Protein
Translation

DNA   [1]
Re verse
Transcription

Translation:
Translation is the process in which the sequence of codons on the mRNA strand is used
(read/decoded) and accordingly the amino acids are joined to each other to form a
polypeptide chain that makes protein.
The process involves the following steps:
a. Activation of amino acids and formation of AA-tRNA complex:
In presence of an enzyme aminoacyl tRNA synthetase, the amino acid (AA) molecule is
activated and each amino acid is attached to the specific tRNA molecule at 3/CCA end to
form aminoacyl-tRNA complex. The reaction requires ATP. This process is called charging
of tRNA or aminoacylation of tRNA.

b. Formation of the polypeptide chain: It is the actual translation which involves the
following steps:
Initiation:
It begins with the formation of initiation complex which requires the mRNA having codons
for a polypeptide, the smaller (30S) and larger (50S) sub-units of ribosome, the initial
AA1tRNA complex and ATP and GTP as source of energy.
The process of initiation needs initiation factors.
In prokaryotes, the first AA1tRNA complex has amino acid, N-formyl-metheonine (f-met);
In eukaryotes, it is metheonine (met).
The process starts with binding of mRNA on the smaller 30S sub-unit of ribosome.
The start codon AUG is positioned properly.
The AA1-tRNA complex, i.e. f-met-tRNA complex now gets attached to the start codon
AUG. This is done with the help of anticodon UAC of tRNA.
Small and large subunits of ribosome join to form 70S ribosome.
The ribosome has three sites namely, Aminoacyl site (A), Peptidyl site (P) and Exit site (E).
The empty tRNA leaves from E site.
Only the AA1-tRNA complex binds at P site directly, while all the other incoming tRNA
complexes get attached first at A site and then are shifted to P site.
Polypeptide chain is released from P-site.
In eukaryotes, 40S (smaller sub unit) and 60S (larger sub unit) combine to form 80S type of
ribosome. [2]
5
Board Question Paper : July 2016

Polypeptide chain

G U Next t-RNA-AA-complex
Empty t-RNA G U
A A

Ribosome (large unit)

START STOP
CGU CAA
A U G U C U C U U G G G U C C G C A G U U A A U U U C U A U C C C U A A
5 3
Ribosome (small unit)

[Proportionate diagram with correct labelling] [2]


Elongation:
This is done by formation of peptide linkages/bonds in between the successive amino acid
molecules (AA1, AA2, AA3 and so on). The elongation activity is catalyzed by the enzyme
peptidyl transferase.
Each tRNA complex brings a specific amino acid. Due to complementary nature of
anticodons and codons, the amino acids are placed to their proper positions.
During elongation, the ribosome moves along the mRNA in a step wise manner from start to
stop codon (5  3), one codon ahead each time. This movement is called translocation.
In every step of translocation, one amino acid is added in the polypeptide chain causing
elongation. [1]
Termination:
When the mRNA reaches the last termination codon, i.e. either UAA, UAG or UGA,
termination occurs.
In identifying the stop/termination codon and in releasing the polypeptide chain from the site,
the release or termination factors R1, R2 and S play an important role. After termination, the
smaller (30S) and larger (50S) sub units of ribosome get separated from each other.
In order to increase the cellular efficiency of protein synthesis, many ribosomes may bind to
the mRNA strand and form the polypeptide chain for synthesis of protein molecule. Such a
structure with many ribosomes bound to mRNA is called polysomes or polyribosome. [1]
OR
Q.4. Dark Reaction:
It is the second phase of photosynthesis in which CO2 is fixed or reduced to glucose. It takes
place in stroma of chloroplast. The reactions take place independent of light hence it is
known as dark reaction.
It is also known as ‘Blackman’s reaction’, ‘Calvin cycle’ or C3 pathway. [1]
The cycle is divided into the following phases:
a. Carboxylation phase: [1]
RuBP (Ribulose-1,5-bisphosphate) accepts atmospheric CO2 in the presence of enzyme RuBP
carboxylase (RuBisCO) and forms a 6-carbon unstable compound. This unstable compound
soon splits into two molecules of 3-carbon compound called PGA (phosphoglyceric acid) in
presence of the same enzyme:

RuBP + CO2 Mg
RuBP Carboxylase
 Unstable compound
(5C) (1C) (6C)
6
Biology

Unstable compound + H2O Mg
RuBP Carboxylase
 2 molecules of 3-PGA
(6C) (3C)

b. Reduction phase (utilization of assimilatory power): [1]


i. The phosphoglyceric acid molecules are first phosphorylated by using ATP to produce
1,3 di-phosphoglyceric acid.
ii. It is then reduced by using NADPH2 to produce phosphoglyceraldehyde (PGAL) and
inorganic phosphate is released.
3PGA + ATP  1,3-diphosphoglyceric acid + ADP
1,3-diphosphoglyceric acid + NADPH2  3PGAL + NADP + Pi.
iii. Some molecules of 3 PGAL are converted into its isomer dihydroxyacetone phosphate
(DHAP) in presence of enzyme triose phosphate isomerase.

c. Synthesis of sugar and Regeneration phase: [1]


Synthesis of sugar:
i. For the synthesis of one molecule of glucose, six molecules of RuBP and six molecules
of CO2 are required, i.e. six turns of Calvin cycle are required for the synthesis of one
molecule of glucose.
ii. Out of 12 molecules of PGAL, 2 molecules (i.e. 1/6th part) are used for the synthesis of
glucose.
iii. A molecule of PGAL and DHAP combine together to form a molecule of fructose 1, 6-
diphosphate as shown below:
3 PGAL + DHAP  Fructose-1,6-diphosphate
(3C) (3C) (6C)
[Note: Fructose-1,6-diphosphate is the first carbohydrate formed during this process]
iv. Fructose-1,6-diphosphate undergoes dephosphorylation to form fructose-6-phosphate
which on isomerization forms glucose-6-phosphate.
v. Glucose-6-phosphate undergoes dephosphorylation to form glucose. Glucose, thus
formed is either utilized or stored as starch.
Regeneration phase:
i. RuBP gets regenerated through many biochemical reactions called sugar phosphate
interconversions.
ii. All the intermediate compounds formed like erythrose-4-phosphate, xylulose-5-
phosphate, ribose-5- phosphate, sedoheptulose-7-phosphate, etc. are sugar phosphates.
iii. Out of 12 molecules of PGAL, 10 molecules are utilized for the regeneration of 6
molecules of RuMP (ribulose monophosphate) which on phosphorylation form RuBP
as shown below:
12 PGAL  6 RuMP ….(1)
6 RuMP + 6 ATP  6 RuBP ….(2)
iv. Thus, RuBP which is necessary for the reduction of CO2 is regenerated to keep the
process going.
7
Board Question Paper : July 2016

RuBP
6CO2
Carboxylase
Ribulose
Bisphosphate Unstable
(5C) Compound (6C)

6 ATP Carboxylation
6 ADP 3-Phosphoglyceric acid
(3C)
Ribulose
Monophosphate 12 ATP
(5C)
Regeneration Reduction 12 ADP
1, 3-Diphosphoglyceric
Sedoheptulose Acid (3C)
phosphate (7C)
12 NADPH2
12 NADP
Erythrose
3-phosphoglyceraldehyde
Phosphate (4C)
(3C)

3-phosphoglyceraldehyde

Fructose-1, 6-diphosphate
(6C)
Pi

Fructose-6-phosphate
(6C)

Glucose-6-phosphate
(6C)
Pi
Glucose
(6C)
Calvin cycle
[Diagram with correct sequential labels] [3]
SECTION – II
[ZOOLOGY]
Q.5. Select and write the most appropriate answer from the given alternatives for each sub-
question:
i. (A) vestigial [1]
ii. (D) Down’s syndrome [1]
iii. (C) haemophilliacs [1]
iv. (C) polyuria [1]
v. (B) retina [1]
vi. (C) AIDS [1]
vii. (A) Banded Krait [1]
8
Biology
Q.6. (A) Answer in ‘One’ sentence each:
i. Two example of commensalism are:
a. An orchid plant growing as an epiphyte on a mango tree. [1/2]
b. Sea anemone and hermit crab [1/2]
ii The common name for Apis mellifera is European Bee. [1]
iii. Bovine somatotropin is used to enhance dairy productivity. [1]
iv. The tendency of the genes on the same chromosome to inherit together is called linkage. [1]
v. Archaeopteryx is the connecting link between birds and reptiles. [1]
vi. Natality is defined as the number of births per unit time, per unit area, per 1000 individuals of
a population. [1]
(B) Afferent arteriole
Efferent arteriole

Glomerulus
Parietal layer

Visceral layer
Bowman’s capsule
Podocyte
Capsular space

Neck
Proximal convoluted tubule

Malpighian body [Proportionate diagram: 1/2 Mark]


[Any three labels: 11/2 mark each] [2]
(C) Attempt any TWO of the following:
i. Propliopithecus

Dryopithecus

Ramapithecus

Australopithecus

Homo habilis
Java Man
Homo erectus
Peking Man
Homo neanderthalensis
(Neanderthal man)

Homo sapiens fossilis


(Cro - Magnon man)

Homo sapiens sapiens


(Modern man)
[Any four sequential evolutionary names: ½ mark each] [2]
9
Board Question Paper : July 2016
ii. Significance of Transgenic animals for betterment of life is as follows:
i. To understand the normal physiology and development:
Specifically designed transgenic animals help us to understand the mechanism of gene regulation.
It is used to study how genes affect the normal functions and development of the body. For e.g.
study of biological role of insulin, role of growth factor in regulating body’s growth.
ii. To test vaccine safety:
Transgenic animals are used as laboratory animals to test efficiency of newly discovered
vaccines before it is used on human beings. Mice are used to test polio vaccines.
iii. To test chemical or toxin safety:
For this, transgenic animals with foreign genes are produced so that the transgenic animals
become more sensitive to toxic chemicals than the non-transgenic animals. These animals are
exposed to toxic chemicals and their effects are observed.
iv. To serve as a model for study of diseases:
Transgenic animals help us to study how genes contribute to development of disease and for
investigation of new treatments for diseases. Transgenic animals are now available to study
cancer, rheumatoid arthritis, cystic fibrosis, Alzheimer’s disease, etc.
v. To obtain biological products
Transgenic animals are created to produce biological compounds or molecules that are useful
in various ways. It is accomplished by introducing the gene coding for a particular molecule,
into the animal. For e.g. Rosie, first transgenic cow was produced which produced milk
containing human protein, alpha lactalbumin.
[Any four points: ½ mark each] [2]
iii. Economic importance of lac:
a. It is used in the preparation of sealing wax, paints, varnish, electrical goods.
b. It is used in the preparation of bracelets, buttons, toys and in filling hollow gold
ornaments.
c. It is used in artificial leather and pottery.
d. It is also used in gramophone industry.
e. It is used for coating fruits and vegetables
[Any four points: ½ mark each] [2]
iv. Mesoderm forms all types of muscles, connective tissue, dermis of skin, adrenal cortex,
kidney, circulatory system including heart, blood vessels and blood, lymphatic vessels,
middle ear and dentine of teeth. [2]
Q.7. (A) Attempt any TWO of the following:
i. Sex determination in birds
a. In birds, male is homogametic, whereas female is heterogametic.
b. Male has genotype ZZ and female has genotype ZW.
c. Sperms carry Z chromosome, but egg carries Z and W chromosome approximately in
equal number.
d. Sex of the zygote depends on which egg gets fertilized.
Male Female [Description] [11/2]
Parents:
ZZ ZW
Gametes: Z Z Z W

F1: ZZ ZW
Male Female
[Chart] [11/2]
ii. Unique features of acquired immunity are as follows:
a. Specificity:
Acquired immunity is specific for each type of pathogen. It has the ability to
differentiate between various foreign molecules.
10
Biology
b. Diversity:
It has the ability to recognize vast variety of diverse pathogens or foreign molecules.
c. Discrimination between self and non-self:
It is able to differentiate between own cells (self) and foreign cells or molecules
(non-self).
d. Memory:
When the immune system encounters a specific pathogen for the first time, it generates
immune response and eliminates the invader. The immune system retains the memory
of this encounter. As a result, a second encounter with same pathogen stimulates a
stronger immune response.
[Any two features: 1½ mark each] [3]

iii. Growing population.

Post
reproductive
Reproductive

Pre  reproductive

Pyramid of growing population [Pyramid] [11/2]


Growing population:
The population having larger number of individuals of the pre  reproductive age groups
shows a very rapid growth rate. Thus is known as growing population.
Young individual in the age group (0-14 years) are included in pre-reproductive group,
individuals in age group (15-59) are included in reproductive group and individuals in age
group (60 yrs and above) are included in post reproductive group.
[Description] [11/2]
(B)
Sensory neuron

Receptor organ (skin)

Connector or Adjust
neuron Effector organ (muscle)

Motor neuron
[Proportionate diagram : 1 mark]
Reflex arc [Any four labels : 1/2 mark each] [3]
Q.8. Process of early cleavage.
Definition: The process of rapid mitotic division of single celled zygote to form hollow, spherical,
multicellular blastula is called cleavage. [1]
Cleavage occurs during its passage through the Fallopian tube to the uterus.
In humans, cleavage is complete or holoblastic and equal.
11
Board Question Paper : July 2016

1st 2nd 2nd 3rd


Cleavage Cleavage Cleavage Morula
Cleavage
complete
Trophoblast
Inner
mass of
cells
Two 3 Cells Four Eight
blastomeres blastomeres blastomeres

Cleavage
[Proportionate diagram] [1]
[Correct labelling] [1]
1st Cleavage:
It is completed about 30 hours after fertilization.
It is vertical, i.e. it takes place from animal pole to vegetal pole.
Two daughter cells or blastomeres are formed. [1]
2nd Cleavage:
It is completed about 60 hours after fertilization.
The second cleavage is also vertical, but at right angle to the first one.
The second cleavage occurs first in one blastomere resulting in transitory three-celled stage
followed by four cell stage. [1]
3rd Cleavage:
It takes place about 3 days (72 hours) after fertilization.
It is longitudinal and horizontal and forms 8 cells.
The cleavage continues and results in the formation of a solid mass of cells known as Morula. [1]
Morula:
Morula is about 16 to 32 celled stage, which looks like a small mulberry. It reaches the uterus 4  6
days after fertilization. It is still surrounded by Zona pellucida. Zona pellucida prevents
implantation at abnormal site. It does not expose trophoblast cells till it reaches implantation site. [1]
OR
Leucocytes are colourless, nucleated, amoeboid and phagocytic cells.
Leucocytes are of two types: Granulocytes and Agranulocytes. [½]
i. Granulocytes: They are produced in red bone marrow and contain large sized granules in the
cytoplasm.
Granulocytes are of three types:
a. Neutrophils (neutro = neutral, philic = affinity)
They constitute about 70% of total WBCs.
Nucleus is multilobed containing 3 to 5 lobes.
The granules in cytoplasm of these cells take up neutral stain. [½]
Functions: These are chief phagocytic cells.
They protect the body against invasion of bacteria.
Dead neutrophils along with damaged tissue are removed from the body in the form of
pus. [½]

Multilobed
nucleus

Granules
[½]
Neutrophil

12
Biology
b. Eosinophils (acidophils):
They constitute about 3% of total WBCs.
The nucleus is bilobed.
The granules in the cytoplasm of these cells take up acidic stain. [½]
Functions:
They are non-phagocytic and their number increases during allergic reactions.
They show anti-histamine property. [½]

Bilobed nucleus

Eosinophil
c. Basophils: [½]
They are the smallest white blood cells which constitute about 0.5% of the total WBCs.
They show twisted nucleus (‘S’ or comma shaped).
The granules in the cytoplasm of these cells take up basic stain. [½]
Functions:
They are non-phagocytic.
They secrete heparin, histamine, thus play an important role in local anticoagulation
and formation of ground substance. [½]

S shaped nucleus

[½]
Basophil
ii. Agranulocytes: They are produced in spleen and do not contain granules in the cytoplasm.
Agranulocytes are of two types:
a. Lymphocytes: They form 30% of total WBCs.
Nucleus is large, spherical and surrounded by thin layer of cytoplasm.
Functions: They produce antibodies and opsonins to neutralize the harmful effects of
foreign matter and their toxins. [½]

Cytoplasm

Nucleus

Lymphocytes [½]
b. Monocytes: They are the largest leucocytes and constitute 1 to 3% of total WBC.
They have large amount of cytoplasm and kidney shaped nucleus.
Functions: They are phagocytic in action. They engulf foreign particles. e.g. bacteria.
They also remove the damaged and dead cells, hence are referred to as scavengers. [½]

Kidney
shaped nucleus [½]
Monocyte

13
Biology

BOARD ANSWER PAPER : MARCH 2017


BIOLOGY
SECTION – I
[BOTANY]
Q.1. Select and write the most appropriate answer from the given alternatives for each sub-
question:
i. (B) IBi [1]
ii. (A) fragmentation [1]
iii. (B) Pusa Gaurav [1]
iv. (D) Streptomyces venezuelae [1]
v. (C) restriction endonuclease [1]
vi. (C) 0.9 [1]
vii. (C) CFCs [1]
Q.2. (A) Answer in One sentence each :
i. The bacterium, Thermus aquaticus is the source of thermostable enzyme DNA polymerase. [1]
ii. Toad stools are non-edible or poisonous mushrooms. [1]
iii. Vincristin and vinblastin are the secondary metabolites in Catharanthus roseus. [1]
iv. The gradual (and predictable) change in the species composition of a given area is called
ecological succession. [1]
v. Yeast and group of enzymes collectively called as zymase can bring about alcoholic
fermentation of sucrose. (Name of organism ½ mark + enzyme ½ mark) [1]
vi. Floral adaptations in Salvia:
a. Flowers are bisexual and protandrous, i.e. anthers mature earlier than stigma.
b. It shows lever mechanism, to favour insect pollination. [1]
(B)
CO2

Sugars,
Starches
Fossil
Fuels

Decomposition

Carbon Cycle

(Correct schematic representation with correct labels) [2]

1
Board Question Paper : March 2017
(C) Answer the following (Any TWO):
i. The cross between F1 hybrid and the recessive parent is called test cross. [1]
Significance of test cross:
a. It helps to determine whether individuals exhibiting dominant character are
genotypically homozygous or heterozygous.
b. Purity of the parents can be determined.
c. It helps to determine the genotype of the individual.
d. It has wide application in plant breeding experiments.
(Any two significance) [½  2] [1]
ii. Wobble Hypothesis:
a. According to Wobble hypothesis, in codon-anticodon pairing the third base may not be
complementary.
b. The third base of the codon is called wobble base and this position is called Wobble
position.
The actual base pairing occurs at first two positions only.

Perfect pairing Wobble pairing

G U U G U C G U A G U G
C A A C A A C A A C A A

5 G C 5 G C 5 G C 5 G C
C C C C
A A A A
val val val val
Wobble Hypothesis
c. In the above example, though the codon and anticodon do not match perfectly, then
also the required amino acid is brought perfectly.
d. This enables the economy of tRNA. GUU, GUC, GUA and GUG code for amino acid –
Valine. So, a single tRNA can interact with all the four codons which code for amino
acid Valine. (Explanation - 1 mark + Correct diagram - 1 mark) [2]
iii. a. A biopatent is a patent granted by the government to the inventor for biological entities,
processes and for products obtained from them. [1]
b. Examples:
1. Basmati rice known for its unique aroma and flavour has been grown in India for
centuries.
2. Turmeric and margosa plants.
(Any one example) [1]
iv. (W)
Suspensor
(Y)
Radicle
(Z)
Cotyledons
(X)
Plumule
(Correct labels to W, X, Y, Z - ½ mark each) [2]

2
Biology
Q.3. (A) Answer the following (Any TWO):
i. Replication of bacteriophages, (lytic cycle) inside the specific host/bacterial cell takes place in
the following steps:
a. Attachment: Bacteriophages attach to specific receptors on the surface of bacteria. As
phages do not move independently, they rely on random encounters with the right
receptors.
b. Penetration: After attachment, the tail fibres bring the base plate closer to the surface
of the cell. Once attached completely, the tail contracts, injecting genetic material
(DNA) through the bacterial membrane. (Capsid – protein coat remains outside and is
called ‘ghost’)
Attachment

Penetration
Lysis of host cell
and release of
virions
Ghost (Capsid)
Assembly of Viral DNA
virions within
host cell
Degradation of host DNA

Synthesis of viral DNA


and protein coat
Lytic cycle
(Schematic representation) [1]
c. Degradation of host DNA: Once the viral DNA enters the host cell, the degradation of
host DNA starts.
d. Synthesis of proteins and nucleic acid: The host’s normal synthesis of proteins and
nucleic acids is disrupted, and it is forced to manufacture viral DNA and proteins
instead. These products are the parts of new virions within the cell or proteins involved
in cell lysis.
e. Virion assembly: The base plates are assembled with the tails first. The head (capsids)
are constructed separately and then are joined with the tails. The DNA is packed
efficiently within the head. The whole process takes about 15 minutes.
f. Release of virions: Phages are released via. lysis of cell. It is achieved by an enzyme
called endolysin, which breaks down the cell wall. Released virions are capable of
infecting a new bacterium.
(Any four correct steps) [½  4] [2]

ii. The biofertilizers are mostly nitrogen fixing microbes which enrich soil with nutrients.
Examples of biofertilizers:
a. Bacterial biofertilizers: Rhizobia are symbiotic nitrogen fixing bacteria associated
with nodules of leguminous plants. Azotobacter and Azospirillum are free-living
nitrogen fixing bacteria used as biofertilizers.
3
Board Question Paper : March 2017
b. Cyanobacterial biofertilizers: Cyanobacteria are autotrophic microbes, also known as
blue green alga which are used as biofertilizers. Some species of Anabaena are
symbiotic. Anabaena is associated with aquatic fern Azolla, whereas Nostoc, Aulosira
are cyanobacteria which carry out nitrogen fixation with the help of heterocysts.
c. Fungal biofertilizers: Mycorrhiza is a fungus and forms symbiotic association with
the roots of higher plants. They are of two types, Ectomycorrhiza and Endomycorrhiza.
(Any two examples) [1  2] [2]

iii.
No. Anemophily Entomophily
i. Wind pollinated flowers Insect pollinated flowers.
ii. Size of flowers is small. Size of flowers is large or present in
groups to show conspicuous appearance.
iii. Not brightly coloured. Brightly coloured.
iv. Odourless. Usually odour/fragrance present.
v. Nectar is not produced. Nectar or edible pollen usually present.
vi. Number of pollen grains produced in Number of pollen grains are less.
large quantity.
vii. Stigma is branched or hairy. Stigma is usually unbranched and sticky.
viii. Pollination is non-directional. Pollination is directional and highly
specific.
ix. Pollen grains are light and smooth. Pollen grains are heavier and sticky.
x. e.g. grass, maize, etc. e.g. jasmine, rose, etc.

(Any six correct differences) [½  6] [3]


(B) Chalaza

Raphe
Nucellus

Integuments
Antipodal cells
Definitive nucleus
Embryo-sac
(secondary nucleus)
Egg-cell
Hilum
Funiculus Synergids
Micropyle

V.S. of mature Anatropous ovule

(Proportionate diagram) [1]


(Any four correct labels) [½  4] [2]

4
Biology
Q.4. Photophosphorylation:
Formation of ATP molecules from ADP and inorganic phosphate (Pi) in presence of light and
chlorophyll during the photochemical phase of photosynthesis is called photophosphorylation or
photosynthetic phosphorylations. [1]
It can be represented as follows:
light
ADP + Pi Chl.a ATP

Non-cyclic photophosphorylation involves the following steps:


i. Photoexcitation of PS-II:
PS-II absorbs light and gets excited, which results in ionisation of chlorophyll-a and a high
energy electron is expelled from PS-II.
ii. Electron Acceptors:
The expelled energy rich electron is first accepted by CO-Q (co-enzyme quinone).
Electron from CO-Q moves down through various electron carriers and releases energy.
From CO-Q, electrons are transferred to plastoquinone (PQ) (it is an iron containing protein).
From PQ, electrons are transferred to cytochrome complex (cytochrome-b and cytochrome-f).
From cytochrome complex, the electrons are transferred to plastocyanin (PC) (it is a copper
containing protein). From PC, the electrons are finally accepted by ionized chlorophyll of PS-I.
iii. Synthesis of ATP :
One ATP is synthesized when electron passes from cytochrome b6 to cytochrome f.
iv. Photoexcitation of PS-I:
The light energy absorbed by PS-I is transferred to reaction centre P700. It gets excited and
expels energy rich electrons. The electrons are first accepted by unknown electron acceptor
called FRS (Ferredoxin Reducing Substance).
Electron from FRS moves down through various electron carriers and release energy.
From FRS, electrons are transferred to ferredoxin (it is an iron containing protein).
The reduced ferredoxin transfers electron to NADP to form NADP 
in the presence of
enzyme Fd-NADP-reductase.
v. Photolysis of water:
Splitting of water into H+ and OH in presence of light and chlorophyll is called photolysis of
water.
Manganese, calcium and chloride ions present in PS-II play an important role in photolysis of
water.
Photolysis of water occurs in order to satisfy the electron need of PS-II and proton need of
NADP .
vi. Assimilatory power:
ATP and NADPH2 are together called as assimilating power by Calvin, as it is required for
assimilation of CO2.
(Explanation - Any two points) [1  2] [2]

5
Board Question Paper : March 2017

Primary
Acceptor CO-Q
FRS

4e 4e

PQ Fd

4e 4e
Cyt. b6 2NADP
ADP + Pi
Photoexcitation

4e ATP
4e 4e
Cyt. f 2NADP 

4e

PC

4e
P-680 P-700
PS-II 4e PS-I

4 photons 4 photons

LIGHT LIGHT

4H+
2NADPH2

CO-Q: Co-enzyme quinone


Mn  ,Cl FRS: Ferredoxin Reducing Substance
4H2O  4e + 4H + 2H2O + O2 
 +

Photolysis of water PC: Plastocyanin


PQ: Plastoquinone
Cyt: Cytochrome
Non-Cyclic Photophosphorylation Fd: Ferredoxin
(Schematic representation) [3]
Significance of non-cyclic photophosphorylation:
i. ATP molecules as well as NADPH2 are produced. [½]
ii. Photolysis of water takes place to release O2. [½]

OR
Q.4. RNA (Ribose Nucleic Acid) is a type of nucleic acid found in the nucleus as well as in the
cytoplasm.
Types of RNA: There are two main types of RNA:
i. Genetic RNA: It acts as genetic material in some viruses.
ii. Non-genetic RNA: It is mainly involved in protein synthesis. There are three different types
of non-genetic RNA:
a. Messenger RNA (m-RNA) b. Ribosomal RNA (r-RNA)
c. Transfer RNA (t-RNA) or Soluble RNA (s-RNA)
a. Messenger RNA (m-RNA) or Informational RNA:
1. It is called messenger RNA because it carries the message for protein synthesis
from DNA to the ribosomes (site for protein synthesis) in the form of codons.
6
Biology
2. It is produced on the DNA strand inside nucleus by a process called transcription
and then transferred to the cytoplasm.
3. It constitutes about 3-5% of the total RNA content of the cell.
4. It is long RNA and the molecular weight of an average sized m-RNA is about
5,00,000.
5. It is always single stranded, linear and straight (unfolded).
6. It has two ends  5 end and 3 end.
7. A triplet of nucleotides on m-RNA is called codon.
8. Each codon on m-RNA specifies one amino acid. This is called m-RNA language
or genetic code or cryptogram.
9. The codon present at 5 end of mRNA is called initiation codon or start codon.
The common initiation codon is AUG or in some cases GUG. AUG and GUG
specify amino acids methionine and valine respectively.
10. The codon present at 3 end is called termination codon or stop codon or non-
sense codon (as they do not specify any amino acid). The termination codon
may be UAA (Ochre) or UAG (Amber) or UGA (Opal).
11. m-RNA is short lived and is degraded soon after protein synthesis.
5 3
AUG UAA

Initiation Triplet Termination


codon codons codon
Structure of m-RNA
Functions of m-RNA:
1. It carries genetic information from DNA to ribosomes during protein synthesis.
2. The genetic code of m-RNA gets translated into the sequences of amino acids to
form proteins.

Unpaired
bases

Paired Coiled region


bases
Uncoiled region
Structure of r-RNA
b. Ribosomal RNA (r-RNA):
1. It is present in ribosomes, hence the name ribosomal RNA.
2. It consists of a single strand.
The single strand is folded upon itself in certain regions.
3. In folded regions, complementary base pairing occurs, while in unfolded regions,
it is absent. Hence, r-RNA does not show purine-pyrimidine equality.
4. It constitutes about 80% of the total RNA content of the cell.
5. The molecular weight ranges from 40,000 to 100,000.
Functions of r-RNA:
1. It provides proper binding site for m-RNA on the ribosome.
2. It orients m-RNA molecule in such a way that all the codons are properly read.
3. It releases t-RNA molecule after transfer of activated amino acid to polypeptide
chain.
4. It also protects the protein molecule under construction.
5. It also protects m-RNA from Rnase enzyme.
7
Board Question Paper : March 2017
c. Transfer RNA (t-RNA) or Soluble RNA (s-RNA) or Supernatant RNA or adapter
RNA.
1. It is the smallest of the three types of non-genetic RNA.
2. It transfers activated amino acids to the site of protein synthesis. Hence, it is
called transfer RNA.
3. It is also called soluble RNA as these molecules cannot be separated from
cytoplasm even by ultra centrifugation technique.
4. The t-RNA molecule consists of a single strand folded upon itself.
5. It constitutes about 10-20% of the total RNA content of the cell.
6. It is made up of 73-93 nucleotides with molecular weight of about 23,000-30,000.
7. There are more than 20 different types of t-RNA.
8. Structure of t-RNA can be explained by two models as:
1. Hairpin model:
The t-RNA molecule is folded in such a manner that there is formation of one loop
having a triplet of unpaired bases called anticodon.
The 5 end has G-nucleotide, while at 3 end, there is a sequence of CCA nucleotides.
2. Clover leaf model (trifoliate leaf model):
i. The clover leaf model of t-RNA shows presence of three arms, namely – DHU
arm, middle arm and TC arm.
ii. These arms have loops at their ends such as amino acyl binding loop, anticodon
loop and ribosomal binding loop respectively.
iii. The anticodon loop has anticodon which is a triplet of unpaired nucleotides.
iv. The anticodons present on t-RNA are complementary to codons present on the
m-RNA (anticodons are also referred to as nodoc).
v. In addition, it also shows a small lump called variable arm or variable lump.
vi. Like the hair-pin model of t-RNA, it has G nucleotide at 5 end and CCA
nucleotides at 3 end.
Functions of t-RNA:
It carries specific type of amino acid at CCA end to the ribosomes during protein synthesis.
It places the required amino acid properly in the sequence. (This becomes possible because of
complementary nature of codons and anticodons).
3 3
A A
C Acceptor end C
C C
5 5
G TC arm G
DHU Loop TC Loop

DHU arm Variable arm/lump


Middle arm
or Anticodon loop
Anticodon arm
Anticodon
Clover-leaf model Hairpin model
Structure of t-RNA

(Correct definition - 1 mark + Correct explanation of each type of RNA - 1  3


+ Correct diagram with labels - 1  3) [7]
8
Biology
SECTION – II
[ZOOLOGY]
Q.5. Select and write the most appropriate answer from the given alternatives for each sub-
question:
i. (C) XC Xc [1]
ii. (B) Y chromosome [1]
iii. (D) progesterone [1]
iv. (C) Seminal vesicles [1]
v. (B) ANF [1]
vi. (A) Diabetes mellitus [1]
vii. (D) Gene therapy [1]

Q.6. (A) Answer in One sentence each:


i. Blood, semen, hair roots or other cells of the body are used for isolation of DNA in
fingerprinting technique. [1]
ii. Podocytes produce foot like processes, which form intimate contact with glomerulus and the
gaps between these processes are filtration slits which help in ultrafiltration. [1]
iii. Commensalism is an interspecific interaction in which one species is benefited and the other
one is neither benefited nor harmed. [1]
iv. Acrosome secretes an enzyme hyaluronidase which helps in the penetration of egg during
fertilization. [1]
v.

No. X Chromosome Y Chromosome


i. These chromosomes are metacentric, These chromosomes are acrocentric,
hence appear X shaped. hence appear Y shaped.
ii. They are longer than ‘Y’ chromosomes.They are shorter than ‘X’ chromosomes.
iii. It contains large amount of euchromatin
It contains large amount of
and small amount of heterochromatin. heterochromatin and small amount of
euchromatin.
iv. Found in both male and females. Found only in males.
v. Non-homologous part shows more genes Non-homologous part contains few
than Y chromosome. genes as compared to X chromosome.
vi. X – linked genes are present on X Y linked genes (Holandric genes) are
chromosome. present on Y chromosome.
vii. Genes present on X chromosome show Genes present on Y chromosome show
criss-cross inheritance. straight inheritance.
(Any two points) [½  2] [1]
vi. Endangered species:
i. Plant species
a. Psilotum nudum b. Osmunda regalis
ii. Animal species
a. Asiatic wild ass b. White-eyed duck c. Red Panda
d. Crocodile
(Any two examples) [½  2] [1]
9
Board Question Paper : March 2017
(B)
Glycoprotein

Lipid membrane
Single Stranded RNA
Reverse transcriptase

Outer and Inner


protein coats
Structure of HIV
(Proportionate diagram) [1/2]
(Any three labels) [1/2  3] [11/2]

(C) Attempt any TWO of the following:


i. RBC (Red Blood Corpuscles) or Erythrocytes:
Erythrocytes are circular, biconcave and non-nucleated cells.
Their diameter is 7 and are 2.5 thick.
In adult male, RBC count is 5.1 to 5.8 million per cubic millimeter, while in female, it is 4.3
to 5.2 million per cubic millimeter.
Normal life span of a single RBC is about 120 days.
Formation of RBCs is called erythropoiesis.
Erythropoiesis in foetus, takes place in yolk sac, kidney, spleen and liver while in adults, it
takes place in red bone marrow.
Stroma of RBC contains haemoglobin which helps in carrying respiratory gases.
Old and worn out RBCs are destroyed in spleen and liver.
(Explanation) [1½]

RBC (Diagram) [½]

ii. a. Vaccine is an antigenic preparation used to stimulate the production of antibodies.


b. Vaccines induce immunity against several diseases.
c. Vaccines stimulate immune system to act against genuine toxins.
d. Vaccines or vaccination programme is used to eradicate a particular disease.
e.g. Now, the disease smallpox is totally eradicated by vaccination programme.
(Any two applications) [1  2] [2]

iii. a. Budding is a type of asexual reproduction seen in Coelenterates (e.g. Hydra) and
Ascidians.
b. These multicellular animals form a small bud which grows gradually.
c. The bud grows slowly into a young animal and then detaches itself from the parent
body.

10
Biology
d. This bud then grows into an independent new organism.
(Explanation) [1]

Parent Parent Parent Parent


body body body Hydra

Young
bud Growing Mature Daughter
bud bud Hydra
Budding in Hydra [1]

iv. Bombyx mori is the species used in sericulture. [1]

Adult Male Adult Female

Cocoon cut open

Cocoon
Eggs

Mature caterpillar
Stages of life cycle of silkworm

(Cyclic representation with arrows) [1]


Q.7. (A) Attempt any TWO of the following:
i. On the surface of plasma membrane of RBC’s, certain glycoprotein molecules called antigens are
present. These antigens differ in different persons and give blood grouping properties to them.
i. ABO blood group system
ii. Rh-blood group system
i. a. ABO blood group system was discovered by Karl Landsteiner.
b. In ABO system, blood groups are determined by the presence or absence of
antigen A and antigen B.
c. Antigen: Blood groups are based on two antigens present on the membranes of
RBC’s, namely antigen A and antigen B. Person may have neither of them or one
of them or both of them.
d. Antibody: It is γ-globulin protein present in blood plasma, so is called plasma
factor. There are two types of antibodies a and b. A person may have neither of
them, one of them or both of them.
e. Antigen A and antibody a and antigen B and antibody b are incompatible to each
other and causes self-clumping. On the basis of this, four blood groups are
recognized.
Blood Group Antigen Antibody Can give blood to Can receive blood from Genotype
A A b A, AB A,O IAIA or IAi
B B a B, AB B,O IBIB or IBi
AB A,B (Both) None AB All (Universal recipient) IAIB
O None a,b (Both) All (Universal donor) O ii

11
Board Question Paper : March 2017
f. Person with blood group O is called Universal Donor as it has no antigen and can
donate blood to any person.
g. Person with blood group AB is called Universal Recipient as it has no antibody
in their plasma, so can receive blood from any blood group.
(Explanation - 1½ marks + Chart - 1 ½ marks) [3]
ii. a. The relative proportion of individuals of various age groups in the population is called
age structure of the population.
b. Age structure of a population is the percentage of individuals of different age groups
such as young (0-14 years), adults (15-59 years) and old (60 years and above).
c. The distribution of these age groups determine the trend of population.

Post-
reproductive
Reproductive

Pre-
reproductive

Age structure showing declining population

d. In declining population, number of post reproductive age group is large whereas the
number of pre reproductive age group is small.
e. Thus, the age structure shows declining population.
(Explanation - ½ mark + Diagrammatic representation - 1½ marks) [3]
iii. Reflex arc: The pathway of nerve fibres along which the reflex impulse travels is known as
reflex arc.
It is always unidirectional from receptor organs to the effector organ, via. CNS.
Reflex arc is the structural and functional unit of reflex action.
Components of simple reflex arc:
Simple reflex arc is formed of five components as given below:
i. Receptor organ:
It is a specialized part of body called sense organ that receives the stimulus and
converts it into the impulse.
e.g. skin, eye, tongue, nose and ears.

Synapse
Sensory neuron

White matter Receptor organ (skin)


Connector or
Adjustor neuron
Grey Matter
Effector organ (muscle)

Motor neuron
Reflex Arc

12
Biology
ii. Sensory or Afferent neuron:
It carries sensory nerve impulse from receptor organ to CNS.
Its cyton is located in dorsal root ganglion.
Its dendron is long and connected to receptor, while the axon enters in the grey matter
of spinal cord to form a synapse.
iii. Associated or Intermediate neuron:
It is present in the grey matter of spinal cord.
It receives sensory impulse, interprets it and generates motor impulse.
iv. Motor or effector neuron:
Its cyton is present in the ventral horn of grey matter and axon travels through ventral
root.
It conducts motor impulse from spinal cord to effector organ.
v. Effector organ:
It is a specialized part of the body which is excited by receiving the motor impulse.
It gives proper response to the stimulus.
e.g. Muscles and glands.
(Explanation - ½ marks + Diagrammatic representation - 1½ marks) [3]
(B)
Urinary bladder
Ureter

Vas-deferens
Seminal vesicle
Ejaculatory duct
Prostate gland Prostatic urethra
Cowper’s gland
Epididymis
Testis
Gubernaculum Penile urethra
Scrotal-sac
Urethra
Penis

Prepuce Glans Penis


External urethral orifice
Human Male Reproductive System
(Diagram) [1]
(Any four labels) [½  4] [2]
Q.8. Human endocrine glands:
Hypothalamus, pituitary, parathyroid, thymus, adrenal, pancreas, testis and ovary are the human
endocrine glands. (Any four names) [½  4] [2]
Histological structure of thyroid gland:
Thyroid gland is externally covered by connective tissue sheath or capsule.
From the capsule, number of septa called trabeculae arise, which divide the thyroid gland into
several lobules.
The lobules contain about 3 million thyroid follicles.
Each thyroid follicle is oval in shape and varies in size.
Larger follicles are present towards periphery, whereas smaller ones are present in the interior.
The follicles are surrounded by a connective tissue called interfollicular tissue which contains blood
vessels and nerve fibres.

13
Board Question Paper : March 2017
Each follicle is lined by a single layer of cuboidal glandular epithelium which rest on a very thin
basement.
The follicular cavity or acinus is filled by dense amorphous semisolid substance called colloid,
which is thyroglobulin, a precursor of thyroid hormone, thyroxine.
Other cells bigger than follicular cells are also present singly or in groups in the connective tissue.
These are called parafollicular or ‘C’ cells.
They secrete hormone thyrocalcitonin. (Explanation) [1½]

Basement membrane
Cuboidal epithelium
Colloid

Para follicular cells

Thyroid follicle

Inter follicular connective tissue


T.S. of Thyroid Gland
(Diagram with three labels) [1½]
Deficiency of thyroxine:
a. Cretinism:
In childhood, deficiency of thyroxine causes cretinism.
It leads to retardation of physical and mental growth of the child.
Patient has low I.Q. (mentally retarded), delayed puberty, dwarfism and sterility.
b. Myxoedema (Gull’s disease):
In adults, deficiency of thyroxine causes myxoedema.
It causes thickening and puffiness of the skin and subcutaneous tissue.
Patient has low BMR, low body temperature, reduced heart rate, low pulse rate and BP, low
blood sugar and iodine level, increased body weight.
It also causes mental dullness (loss of memory), falling of hairs, dry skin and intolerance of
cold.
c. Simple goiter (Iodine deficiency goiter or endemic goiter):
Deficiency of iodine in diet or drinking water causes simple goiter.
It causes enlargement of thyroid gland (15 times or more) for synthesis of thyroxine hormone.
It is commonly found in hilly regions.
(Any two disorders) [2]
OR
Evolution:
Organic evolution is a slow, gradual, continuous and irreversible changes through which the
present day complex forms have descended from their simple, pre-existing forms of the past. [1]
Principles of Darwinism:
Darwin’s theory of organic evolution by natural selection is based on the following principles:
i. Over production or prodigality of production:
All organisms have a natural tendency to over produce.
If this tendency is not checked, then even a single species of a plant or animal will occupy
the entire space available on the earth. [1]
ii. Struggle for existence: Organisms multiply in geometric ratio, but space and food remain
constant leading to competition for survival. Increase in the number of species leads to a
competition called struggle for existence. [1]
14
Biology
iii. Variations and Heredity:
The differences which occur between the closely related organisms are called variations.
It is universal law of nature. Variations may be favourable or unfavourable. [1]
iv. Survival of the fittest or natural selection:
According to Darwin, in the struggle for existence, the fittest individuals survive and
reproduce, while the unfit individuals perish. [1]
v. Origin of new species:
According Darwin, useful variations appear in every generation and are inherited from one
generation to another. [1]

Objections to Darwin’s natural selection theory:


i. Natural selection theory explained “survival of the fittest” but not “arrival of the fittest”.
ii. Darwin did not take into account the hereditary principles.
iii. He could not provide a satisfactory explanation for the cause, origin and inheritance of
variations.
iv. Certain useless characters are also inherited. He could not explain the inheritance of useless
variations.
v. He was unable to differentiate variations as hereditary and environmental variations.
(Any one objection) [1]

15
Biology

BOARD ANSWER PAPER : JULY 2017


BIOLOGY
SECTION – I
[BOTANY]
Q.1. Select and write the most appropriate answer from the given alternatives for each sub-question:
i. (C) endonucleases [1]
ii. (D) rice [1]
iii. (C) DNA [1]
iv. (A) CFCs [1]
v. (D) humification [1]
vi. (B) 140 [1]
vii. (C) Acetobacter aceti [1]
Q.2. (A) Answer in One sentence each :
i. The Blue Green Algae (BGA) have some specialized colourless cells called heterocysts
which are the sites of nitrogen fixation. [1]
ii. Plasmids are small, circular, double stranded, self-replicating DNA molecules. [1]
iii. Chl-a (Chlorophyll-a) absorbs and converts solar energy into chemical energy, hence it is
called as essential pigment. [1]
iv. Global warming is an increase in global mean temperature caused by green house effect. [1]
v. Nif gene is a Nitrogen (N2) fixing gene. [1]
vi. The aquatic fern commonly used in paddy field as biofertilizer is Azolla. [1]
(B)
Outer membrane
Inner
membrane
DNA
Stroma
1 to 2 

Grana

Stroma lamellae
Ribosome

4 to 6 

Ultrastructure of Chloroplast
(Diagram - ½ mark + Any three labels  outer membrane, inner membrane,
grana, stroma, stroma lamellae, DNA, ribosome - 1½ marks) [2]
(C) Answer the following (Any TWO):
i. Following are the advantages of biogas:
a. Biogas is a cheap, safe and renewable source of energy.
b. Biogas can be burnt in gas stoves to provide heat.
1
Board Question Paper : July 2017
c. It can be used for cooking, domestic lighting, street lighting.
d. It burns with a blue flame and without smoke.
e. It is eco-friendly and does not cause pollution.
f. It can be used for driving engines.
g. It helps to improve sanitation of the surrounding.
h. It can be easily generated, stored and transported.
i. The residue left after biogas production can be used as manure.
(Any four points) [½  4] [2]
ii. Pusa Gaurav, Pusa Sawni, Pusa A-4 are insect-resistant crop varieties.
(Any two varieties) [1  2] [2]
iii. The causes of deforestation are:
a. Intensive collection of firewood.
b. Clearing of land for agriculture.
c. Developing pastures for grazing animals.
d. Carrying out mining activities.
e. Obtaining land for construction of roads, dams and buildings.
f. Selling wood for earning foreign exchange.
g. Meeting the increasing demand of timber wood.
h. Natural calamities like wildfire, floods, storms, volcanic eruptions, earthquakes, etc.
(Any four points) [½  4] [2]
iv. Floral adaptations for entomophily:
a. Flowers are large and attractive. When small, they are clustered into an inflorescence
(e.g. sunflower).
b. Flowers have attractive bright colors, with pleasant fragrance and nectar gland.
c. In some plants, additional modifications are made to attract insects. e.g. Corona in
passion flower and petalloid bracts in Bougainvillea.
d. Pollen grains possess spiny or rough outer wall.
e. Stigma has rough and sticky surface.
f. To favour insect pollination, some plants develop special mechanism e.g. lever
mechanism in Salvia. e.g. Jasmine, Rose, Salvia, Bougainvillea.
(Any four points) [½  4] [2]
Q.3. (A) Answer the following (Any TWO):
i. Wobble Hypothesis:
According to Wobble hypothesis, in codon-anticodon pairing the third base may not be
complementary.
The third base of the codon is called wobble base and this position is called Wobble position.
The actual base pairing occurs at first two positions only.

Perfect pairing Wobble pairing

G U U G U C G U A G U G
C A A C A A C A A C A A

5 G C 5 G C 5 G C 5 G C
C C C C
A A A A
val val val val
Wobble Hypothesis

2
Biology
In the above example, though the codon and anticodon do not match perfectly, then also the
required amino acid is brought perfectly.
This enables the economy of tRNA. GUU, GUC, GUA and GUG code for amino acid –
Valine. So, a single tRNA can interact with all the four codons which code for amino acid Valine.
(Meaning and explanation - 1 mark + Diagram - 2 marks) [3]
ii. a. Each pyruvic acid molecule formed in glycolysis undergoes oxidative decarboxylation
(i.e. oxidation by dehydrogenation and decarboxylation by loss of CO2), to form Acetyl
group (2C), which combines with Co-A to form Acetyl Co-A.
b. Thus, pyruvic acid is converted to Acetyl Co-A. This is called Acetylation of Pyruvate.
c. It involves several reactions, many enzymes and co-factors.
d. Hydrogen released combines with NAD to form reduced co-enzyme NADH2, which
has the potential to form 3ATP in ETS.
e. In this process, CO2 is released.
f. Acetyl Co-A is a connecting link between glycolysis and Krebs cycle.
g. Summary is represented as:
Enzyme Complex ( Pyruvic dehydrogenase )
2PA + 2Co-A + 2 NAD             2 Acetyl Co-A + 2 CO2
(3C) (2C)
+ 2 NADH2
(Explanation) [1½]
Schematic representation of formation of Acetyl Co-A:
2NAD
(3C) Pyruvic acid (2M)
2NADH2 2H 2CO2

Acetyl + Coenzyme - A

(2C) Acetyl Co-A (2M) Co -A

(Schematic representation) [1½]


iii. ROUTE II
Succinic acid Fumaric acid
2H

ATP FAD ATP ATP


FADH2
ROUTE I
NAD 1
AH2 FMN Co Q Fe3+ Fe3+ Fe3+ Fe3+ Fe3+ O2
Reduced 2
Substrate 2e 2 Cyt b 2 Cyt c1 2 Cyt c 2 Cyt a 2 Cyt a3 2e

NADH2 FMNH2 Co QH2 Fe2+ Fe2+ Fe2+ Fe2+ Fe2+ O


A H 2O
Oxidised
Substrate O + 2eO 
ADP + iP 2H+ ADP + iP ADP + iP 2H+ + O 2H2O

Diagrammatic Representation of Electron Transport Chain

(Diagram of ETS - 1 mark + Any four labels – ½ 4) [3]

3
Board Question Paper : July 2017
(B)
Attachment

Penetration
Lysis of host cell
and release of
virions
Ghost (Capsid)
Assembly of Viral DNA
virions within
host cell
Degradation of host DNA

Synthesis of viral DNA


and protein coat
Lytic cycle
(Diagram) [1]
(Any four correct labels) [½  4] [2]

Q.4. Neo Mendelism:


a. Different patterns of inheritance or gene interactions were discovered based on Mendelian
ratios.
b. These interactions can be explained by modifying Mendel’s laws thus, this is called Post
Mendelian genetics or Neo Mendelism.
c. These gene interactions are of two types intragenic and intergenic.
d. Quantitative inheritance is also known as polygenic inheritance in which characters are
determined by two or more gene pairs which have additive or cumulative effect. These genes
are called cumulative gene or polygenes.
e. This quantitative inheritance can be explained by wheat kernel colour which is controlled by
two pairs of genes Aa and Bb.
f. H. Nilsson Ehle crossed red kernelled variety of wheat with white kernelled variety.
g. Gene A and B determine the red colour of kernel (responsible for producing red pigment)
and are dominant over their recessive allele a and b which do not produce red colour pigment
and the expression is white, if dominant gene is not present. Thus, genotype of a parent with
red kernels is AABB and that of parent with white kernels is aabb. Genotype of F1 hybrids is
AaBb.
h. Continuous variations in expressions is observed as each dominant gene produces specific
amount of pigment and the shade of red colour, depends on number of dominant gene present
as shown in Punnett square.
4
Biology

Phenotype of Parents Red kernels White kernels

Genotype AABB  aabb

Gametes AB ab

AaBb
F1 generation
Intermediate Red
Selfing of F1 generation AaBb  AaBb

Gametes AB Ab aB ab

AB AABB AABb AaBB AaBb


Red darkest Medium Medium Intermediate
F2 generation Ab AABb AAbb AaBb Aabb
Medium Intermediate Intermediate Light
aB AaBB AaBb aaBB aaBb
Medium Intermediate Intermediate Light
ab AaBb Aabb aaBb aabb
Intermediate Light Light White
i. In F2 generation, five different phenotypic expressions appeared in the ratio 1 : 4 : 6 : 4 : 1
(Meaning of Neo-Mendelism - 1 mark + Meaning of quantitative inheritance - 1 mark
+ Example and parental cross - 1 mark + F1 hybrids and gametes - 1 mark
+ Checker board of F2 generation with phenotypes - 2 marks
+ Ratio obtained in F2 generation - 1 mark) [7]
OR

Q.4. Advantages of cross pollination:


i. Offsprings produced are healthier and are well adapted to the environment.
ii. Seeds are produced in large numbers with higher viability and of better quality.
iii. Possibility of variations results in new desired varieties of plants, hence favours the process
of evolution.
iv. The offsprings show better vigour (hybrid vigour) and vitality.
Disadvantages of cross pollination:
i. The plants have to depend upon various external agencies.
ii. There is expenditure of energy due to the adaptations such as bright colour, nectar, fragrance,
etc. in attracting insects.
iii. There is considerable wastage of pollen grains.
iv. Since cross pollination result in the formation of new genotypes, it may also develop
undesirable characters in the progeny. Some desirable characters may get eliminated.
v. Genetic purity is not maintained.

Pollination mechanism in Salvia:


i. In Salvia, flower is bisexual and protandrous, i.e. anthers mature earlier than stigma.
ii. The two stamens of flower have long bifurcated connective.
iii. Upper branch of connective bears fertile anther lobe, while lower has sterile anther lobe.
iv. When an insect enters the flower, it pushes the lower sterile lobes. As a result, the upper fertile anther
lobe bends down.
5
Board Question Paper : July 2017
v. The fertile anther lobe comes in contact with back side of insect body and pollen grains are dusted there.
vi. When the same insect visits another flower with matured gynoecium, the pollen grains are picked up
by the receptive stigma.
vii. This mechanism is called lever-mechanism or turn-pipe mechanism.

Fertile anther lobe


Connective
Sterile anther lobe
B

C
Pollination is Salvia flower
(Three points of advantages - ½  3 + Three points of disadvantages - ½  3
+ Explanation of pollination mechanism in Salvia - 2 marks
+ Labelled diagram of pollination in Salvia flower - 2 marks) [7]

SECTION – II
[ZOOLOGY]
Q.5. Select and write the most appropriate answer from the given alternatives for each sub-question:
i. (B) Theory of organic evolution [1]
ii. (A) complete sex-linkage [1]
iii. (B) Sporozoite [1]
iv. (D) acetylcholine [1]
v. (B) tunica vaginalis [1]
vi. (B) against concentration gradient [1]
vii. (D) Human growth hormone producer gene [1]
Q.6. (A) Answer in One sentence each:
i. The first vaccine was developed against small pox. [1]
ii. The two types of maps generated by H.G.P. are genetic linkage maps and physical maps. [1]
iii. Appearance of different types of symptoms is called syndrome. [1]
iv. Killer-T cells secrete perforins. [1]
v. The full form of IVF is In vitro fertilization. [1]
vi. The origin of life on the earth is called protobiogenesis. [1]

6
Biology
(B)
Afferent arteriole
Efferent arteriole

Glomerulus
Parietal layer

Visceral layer
Bowman’s capsule
Podocyte
Capsular space

Neck
Proximal convoluted tubule

Malpighian body
(Proportionate diagram) [1/2]
(Any three labels) [1/2  3] [11/2]
(C) Attempt any TWO of the following:
i. Organisms showing characters of two groups (kingdoms, phyla, classes, etc.) are called connecting
links between the two groups.
Archaeopteryx shows characters of birds as well as of reptiles.
Avian characters of Archaeopteryx:
a. It has skull bones completely fused: cranium rounded with large orbits and a single condyle.
b. Jaws modified into beak.
c. Forelimbs modified into wings.
d. Limb bones have their first toe opposable.
e. Foot with clawed digits.
f. Feathery exoskeleton
Reptilian characters of Archaeopteryx:
a. Jaw with homodont teeth.
b. Ribs with single head.
c. Abdominal ribs somewhat like those seen in crocodiles.
d. Forelimbs modified into wings and three digits ending in distinct claws.
e. The hind limb had four digits ending with claws.
f. Long tail.
Thus, Archaeopteryx is called connecting link between reptiles and birds.
(Any two Avian characters - 1 mark + Any two Reptilian characters - 1 mark) [2]
ii. a. Production of vaccines involves growing the organism in a growth medium to produce
large amounts of toxins. [½]
b. These toxins are then altered to give toxoids which are no longer toxic. These toxoids
become antigens which the immune system recognises, and hence produces antibodies
to act against genuine toxins. [½]
c. The live organisms in the culture are inactivated, and the resulting mixture is diluted
and mixed with an adjuvant. [½]
d. The blended vaccine is then filled and packed. [½]
7
Board Question Paper : July 2017
iii. a. Lac is a pink coloured resin which hardens on coming in contact with air forming lac.
b. Tacchardia lacca is most commonly used for lac culture.
c. Lac insects feed on succulent twigs of certain plants like Ber, peeple, palas, kusum,
babool, etc.
d. Natural lac is always contaminated, so washing and filtering is needed to obtain shellac
in pure form.
e. India contributes upto 85% of total lac production in world.
f. Lac is the byproduct of lac culture which is used in bangles, toys, woodwork, polish
inks, silvering mirrors etc.
(Any four points) [½  4] [2]
iv. Commensalism:
Commensalism is an interspecific interaction in which one species is benefited and the other
one is neither benefited nor harmed. [1]
e.g. a. An orchid plant growing as an epiphyte on a mango tree.
In this association, the orchid plant is benefited (gets its shelter), while the mango
tree is neither benefited nor harmed.
b. The cattle egrets and grazing cattle.
The egrets forage close to where the cattle are grazing because as the cattle
move, they stir out the insects from the grass, that otherwise might be difficult for
the egrets to find and catch (the egrets are benefited and the cattle are
unaffected). (Any one example) [1]

Q.7. (A) Attempt any TWO of the following:


i. Chromosomes can be classified into four types on the basis of position of centromere:
a. Metacentric:
Centromere is in the middle of the chromosome and hence, the two arms are nearly equal.
Chromosome looks ‘V’-shaped in anaphase.
b. Submetacentric:
Centromere is not in the center but some distance away from the centre of
chromosomes and therefore arms are unequal in length.
The chromosome appears ‘L’ shaped in anaphase.
c. Acrocentric:
Centromere is near one end of the chromosome. One arm is very short, while other is long.
d. Telocentric:
Centromere is at the extreme tip of the chromosome.
The arm is only on one side.
Chromosome remains rod shaped even in anaphase.
Chromosome appears ‘i’ shaped.
Centromere Centromere

Centromere Centromere

Metacentric Sub-metacentric Acrocentric Telocentric

(Explanation - 2 marks + Diagram - 1 mark) [3]


ii. a. Characteristic symptom is slowly progressing fever as high as 40C or 104 F, profuse
sweating and gastroenteritis.
b. Headache, general weakness is common among the patients.
8
Biology
c. Abdominal pain with either constipation or diarrhoea.
d. Leukopenia, eosinopenia and relative lymphocytosis is observed.
e. Bradycardia (slow heart rate) in 1st week of infection.
f. In some cases, in the 2nd week, a rash of flat, rose coloured spots called “rose spots”
appear on the front of the chest.
g. Ulceration of intestine, lesion of intestinal mucosa and haemorrhage, delusion and
confusion are also possible in 3rd week.

‘O’ antigen

Flagella
with antigen
‘H’

Salmonella typhi
(Any four symptoms) [½  4] [2]
(Diagram) [1]
iii. The relative proportion of individuals of various age groups in the population is called age
structure of the population.
Role of age structure:
Age structure (distribution of age groups) determines the trend of the population.
a. Growing population:
Population having larger number of individuals of the pre-reproductive age groups
shows a very rapid rate of growth.
b. Steady population:
Population having same number of pre-reproductive and post-reproductive age groups
is stable.
c. Declining population:
Large number of post-reproductive and small number of pre-reproductive groups make
the population decline.

Post
Post-reproductive
reproductive
Reproductive
Post Reproductive Pre-
reproductive reproductive
Reproductive Pre -
Pre - reproductive reproductive

Age structure showing declining


Pyramid of growing population population
Pyramid of steady
population
(Explanation - 1½ marks + diagram - 1½ marks) [3]

9
Board Question Paper : July 2017
(B)

Basement membrane
Cuboidal epithelium
Colloid

Para follicular cells

Thyroid follicle

Inter follicular connective tissue


T.S. of Thyroid Gland
(Proportionate Diagram) [1]
(Any four labels) [½  4] [2]
Q.8. Histology of Testis:
i. Externally, each testis is covered by three layers. These are:
a. Tunica vaginalis:
It is the outermost incomplete peritoneal covering made up of connective tissue and
epithelium.
b. Tunica albuginea:
It is the middle layer formed by fibrous connective tissue.
c. Tunica vasculosa:
It is the innermost layer formed of delicate connective tissue, supporting a network of
blood capillaries.
Tunica albuginea
Interstitial cells / cells of Leydig

Sperm bundle Seminiferous tubule


Germinal epithelium
Sertoli cell

Basement membrane
Connective tissue
Different stages of
spermatogenesis
T.S. of testis
ii. Each testis contains about 200-300 tubules called seminiferous tubules.
iii. These are lined by a single layer of cuboidal germinal epithelium which undergo
spermatogenesis.
iv. In the germinal epithelium, various stages of spermatogenesis such as spermatogonia,
primary spermatocytes, secondary spermatocytes, spermatids and sperms are seen.
v. Between these cells, few large and pyramidal cells called nurse cells or sertoli cells are
present.
vi. Bundles of sperms are seen attached to Sertoli cells.
10
Biology
vii. These cells provide nourishment to the sperms till maturation.
viii. In between seminiferous tubules, connective tissue containing blood vessels, nerves, lymph vessels
and groups of interstitial cells (Cells of leydig) is present.
ix. Interstitial cells produce male hormone testosterone.
Spermatogenesis:
Spermatogenesis is the process of formation of haploid, microscopic and motile male gametes, called
spermatozoa, from the diploid spermatogonia of the testis of male organism.

Primordial cell

Mitosis

Growth phase Primay spermatocyte

Secondary spermatocyte
Meiotic division
Spermatid

4 Spermatozoa
Spermatogenesis

Each testis has seminiferous tubules which are lined by cuboidal epithelium called germinal epithelium.
Germinal cells in testes are known as primary germinal cells. Primordial cells passes through three
phases, namely:
a. Multiplication phase: Primordial cells undergo repeated mitotic divisions to produce large
number of spermatogonia.
Each spermatogonium is diploid (2n).
b. The Growth phase: Spermatogonium cell accumulates food and grows in size. Now, it is
called primary spermatocyte.
c. The Maturation phase: The primary spermatocyte undergoes first meiotic or maturation
division. The homologous chromosomes start pairing.
Each homologous chromosome splits longitudinally. Chiasma formation results in exchange
of genetic material.
At the end of 1st meiotic division, two haploid, secondary spermatocytes are formed. Each
secondary spermatocyte undergoes 2nd meiotic division and produces spermatids. So, at the
end of maturation phase, each spermatogonium produces four haploid spermatids.
Spermatid is non-motile, so it has to undergo spermiogenesis to become functional, motile
male gamete, i.e. spermatozoan.
(Description of histology of testis - 2 marks + Diagram of T.S. of testis - 1 mark
+ Three phases of spermatogenesis - 3 marks + Diagram of spermatogenesis - 1 mark) [7]
11
Board Question Paper : July 2017
OR
Working of Heart:
Human heart is composed of cardiac muscles which are capable of alternate contraction (systole)
and relaxation (diastole).
Human heart is myogenic as its contraction is initiated and conducted by the modified muscle
plexuses called nodal tissues. [1]
It consists of the following components:
a. SinoAuricular or SinuAtrial node (SA node or SAN)
b. Atrioventricular node (AV node of AVN)
c. Bundle of His
d. Purkinje fibres
a. Sinoauricular or sinuatrial node or SA node or SAN:
A specialised patch of cardiac muscle present in the wall of right atrium near the right side of
right atrium [opening of precaval vein] is known as SA node or SAN.
It is also called pacemaker, as it is the first to originate the cardiac impulses and maintain the
pace (or duration) between first and second contraction.
It brings about atrial systole and this wave of contraction generated by SA node is conducted
to both the atria and brings about their contractions. [1]
b. Atrioventricular node (AV node):
It lies in the right atrium near the junction of inter-auricular and inter-ventricular septum,
close to groove which separates atria and ventricles.
It is stimulated by the wave of contraction initiated by SA node.
It generates the cardiac impulses which are conducted to the muscles of the ventricles through
Bundles of His and Purkinje fibres. [1]
c. Bundle of His:
1. It arises from AV node and spreads the wave of contraction at the speed of 5 mts/sec.
2. It divides into two branches which descends along two sides of inter-ventricular septum
and supply the wall of the ventricle of their own side by a network of fine fibres called
Purkinje fibres which penetrate in myocardium of the ventricles.
3. SA node, AV node, Bundles of His and Purkinje fibres collectively form the
conducting system of the heart. It is responsible for autorhythmicity of heart. [2]

Sino-auricular node Atrio-ventricular node

Right atrium Inter-atrial septum


Left atrium
Annular pad
Right Atrio-ventricular valve

Semilunar valve Bundle of His

Purkinje fibre
Inter-ventricular septum
Right ventricle
Left ventricle

Conducting System of Human Heart [2]

12

S-ar putea să vă placă și